explanations to aamc practice test iiidocshare02.docshare.tips/files/14524/145247660.pdfaamc mcat...

65
Explanations to AAMC Practice Test III © Kaplan, 2001. All rights reserved. * MCAT is a registered trademark of the Association of American Medical Colleges.

Upload: others

Post on 09-Mar-2021

58 views

Category:

Documents


4 download

TRANSCRIPT

Page 1: Explanations to AAMC Practice Test IIIdocshare02.docshare.tips/files/14524/145247660.pdfAAMC MCAT PRACTICE TEST III VERBAL REASONING EXPLANATIONS Passage I (Questions 1-6) This passage

Explanations toAAMC Practice Test III

© Kaplan, 2001. All rights reserved.

* MCAT is a registered trademark of the Association of American Medical Colleges.

Page 2: Explanations to AAMC Practice Test IIIdocshare02.docshare.tips/files/14524/145247660.pdfAAMC MCAT PRACTICE TEST III VERBAL REASONING EXPLANATIONS Passage I (Questions 1-6) This passage

©2001 KAPLAN, INC.

All rights reserved. No part of this book may be reproduced in any form, byphotostat, microfilm, xerography, or any other means, or incorporated into anyinformation retrieval system, electronic or mechanical, without the writtenpermission of Kaplan, Inc.

Page 3: Explanations to AAMC Practice Test IIIdocshare02.docshare.tips/files/14524/145247660.pdfAAMC MCAT PRACTICE TEST III VERBAL REASONING EXPLANATIONS Passage I (Questions 1-6) This passage

Copyright © 2001, Kaplan, Inc. 1None of this material may be reproduced by any means or incorporated into any information retrieval system, without written permission.

AAMC MCAT PRACTICE TEST III VERBAL REASONING EXPLANATIONS

Passage I (Questions 1-6)

This passage addresses the issue of drug legalization in the United States. The author writes a lengthy passagethat attempts to persuade the reader that legalization of presently-illicit drugs are in the best interest ofsociety. One initial note: the passage contains multiple ellipses (. . .) within and between sentences. Do notlet this distract you. Ellipses are occasionally placed in Verbal Reasoning to hinder the reader. Just read rightthrough them.

The author introduces the topic – and his position on this issue – in the first two paragraphs. Here, the authoradvocates increased public debate on the issue of legalization, as the author believes that such discussionwould improve public sentiment toward legalization. In the third paragraph, the author outlines the three majorpoints to be addressed throughout the remainder of the passage. The first point is that present drug controlpolicies have failed; the second is that drug-control efforts are "costly and counter-productive"; and the third isthat "repealing drug laws would not lead...to a dramatic rise in drug abuse."

The author proceeds to discuss the point that drug control policies have proven costly and ineffective. Hebegins in the fourth, fifth, and sixth paragraphs by discussing that drug control policies have consumed anincreasing amount of money and resources from law enforcement agencies over the last decade.

In the next four paragraphs, numbers seven through ten, the author elaborates on the connection between illicitdrug use and crime. The author even discusses corruption among police with regard to the drug trade. Theauthor presents an interesting analogy here – the author compares crime and police corruption associated withthe modern drug trade with similar activities during Prohibition of the 1920s. The author argues thatProhibition suppressed an "illicit market" that cannot be eliminated, and thus, the "drug war" suppresses amarket that cannot be eliminated as well.

In the eleventh paragraph, the author highlights some of the potential benefits of drug legalization, includingnew drug tax revenue. The author also states that a legalized drug trade would result in increased quality ofurban life, as well as a decrease in many felonies, including homicide and robbery.

In the twelfth paragraph, the author outlines the evidence supporting his belief that legalization of drugs wouldnot lead to increased addiction rates. The author bases this hypothesis on two assumptions: most drugs are notas dangerous as suspected, and most people would not accept the dangers behind use of many drugs. Finally,the author concludes the passage in paragraph thirteen by once again calling for increased public discussion ofdrug legalization.

1. Difficulty = Moderate; Question Type = Application;The correct answer is choice D. This is an application question that requires you to assess a quote,

"tobacco shortens one's life, cocaine debases it; nicotine alters one's habits, cocaine alters one's soul." Thequestion then asks you to deduce the beliefs of this quote's orator. Obviously, this quote reflects the beliefthat, in general, drugs are hazardous, but cocaine – and other illicit drugs – are far more destructive anddangerous to individuals than legalized drugs such as tobacco. Choice D, "...both cocaine and tobacco [are]harmful but cocaine more so," clearly reflects this belief and is correct.

Choice A implies the opposite of the question stem's quote. This quote does not support thelegalization of cocaine. Similarly, choice C represents the opposite of the orator's tone by implying thatcocaine and tobacco are not dangerous. Choice B seems to make sense, but as we saw in the passage,someone can recognize that drugs are harmful and also see that abolition is fraught with problems as well.Again, choice D is correct.

2. Difficulty = Moderate; Question Type = Application;The correct answer is choice C. The question stem asks you to select a federal law with which the

author would most likely agree. The author advocates legalization of drugs, as well as most programs thatwould support responsible drug use. Choice C, dispensing free sterile needles to drug addicts, would mostlikely earn the author's strong support.

Choice A is misguided, as the requirement of mandatory drug-treatment programs to convicted drugusers contradicts the author's tolerance of free drug use. Also, the word "convicted" should have lead you to

Page 4: Explanations to AAMC Practice Test IIIdocshare02.docshare.tips/files/14524/145247660.pdfAAMC MCAT PRACTICE TEST III VERBAL REASONING EXPLANATIONS Passage I (Questions 1-6) This passage

Kaplan Explanations to AAMC Practice Test III Verbal Reasoning

2 Copyright © 2001, Kaplan, Inc.None of this material may be reproduced by any means or incorporated into any information retrieval system, without written permission.

immediately eliminate this choice, as this word implies illegality of drug use – a position that the authorobviously opposes. B is wrong because the author would vigorously oppose granting tax-exempt status to thedrug trade--one of the benefits he expects from legalization is "new tax revenues from legal drug productionand sales" (lines 58-59). Choice D advocates the use of the military to intercept foreign drug traffic. Thisstatement contradicts the point of the passage. The author advocates free drug trade, not elimination of drugtrade. Again, choice C is correct.

3. Difficulty = Moderate; Question Type = Implied Detail;The correct answer is choice D. This question requires you to assess the validity of three statements

as they pertain to the author's analogy of present-day drug laws to the prohibition of alcohol. The analogy ofpresent drug laws to prohibition of alcohol is discussed in paragraphs 9 and 10 of the passage. It is best,however, to address each statement in the question stem and indicate which ones are valid.

Option I suggests that such drug laws will ultimately fail as prohibition of alcohol did in the 1920s.This is true, as the author regards Prohibition of alcohol as having been futile and believes that present druglaws are similarly senseless. Option II implies that present drug laws encourage police corruption similar tothat of the 1920s. This is also true, and the author details corruption in law enforcement in paragraphs 9 and10. Option III implies that drug laws would be "far less costly" to society if drugs were no longer illegal butrather controlled. This is true as well. The author elaborates on the societal benefits of drug legalization inparagraph 11. As options I, II, and III all work, the correct answer is choice D.

4. Difficulty = Moderate; Question Type = Detail;The correct answer is choice C. This question is similar to question 3, as it requires the evaluation of

three statements. In this question, each statement must be evaluated regarding its support of the theory thatpublic treasuries would "enjoy a net benefit as a result of drug legalization." Again, it is best to assess eachstatement individually.

Option I notes that income earned form the drug trade would be taxed. This is true: according to theauthor, taxation of revenue would certainly increase public treasuries. Option II declares that fewer lawenforcement personnel would be needed. There is no evidence presented in the passage regarding theelimination of law enforcement once drugs are legalized. Rather, the author only implies that existing lawenforcement officials would devote more time to other duties, not be reduced or eliminated altogether.Therefore, option II is not true. Option III states that fewer public funds would be spent on drug enforcement.The author states in paragraph 11 that fewer funds would be spent on drug enforcement after legalization ofdrugs. Thus, options I and III are valid, and choice C is the correct answer.

5. Difficulty = Moderate; Question Type = Tone;The correct answer is choice B. The question stem asks which answer choice best supports the

author's claim that drug legalization would not result in "a dramatic increase in drug abuse." The authorpresents his rationale for this assumption in paragraph 12. The author claims that many people would not bewilling to accept the dangers and risks associated with drug use, and therefore, most people would elect not touse drugs despite their legal status. Choice B, "Most Americans are unlikely to engage in a dangerousactivity," agrees completely with the author's reasoning and would, if true, support the author's originalcontention. Thus, choice B is correct.

Choice A is a tricky answer choice. While hostility toward drug legalization might indicate adisinclination to use drugs among most Americans, there is no direct correlation presented in the passage tosupport this inference. Choice A is not a strong answer choice and is incorrect. Similarly, choice C does notthoroughly satisfy the question stem. This answer choice implies that most Americans do not consider thelegality of a substance before deciding to ingest it, so simply legalizing drugs would have no effect on thelevel of abuse. This is a weak hypothesis--one that is never mentioned in the passage. This answer choicedoes not completely support the author's assumption and is certainly not the strongest answer choice. Thus,choice C is incorrect. Finally, choice D is also not correct. In fact, choice D would support the opposite ofthe author's entire argument. If the consumption of alcohol increased after the repeal of prohibition, then onewould expect drug use to increase after the repeal of drug laws. Obviously, this contradicts the authors'assumption as stated in the question stem, making choice D incorrect. Again, choice B is correct.

Page 5: Explanations to AAMC Practice Test IIIdocshare02.docshare.tips/files/14524/145247660.pdfAAMC MCAT PRACTICE TEST III VERBAL REASONING EXPLANATIONS Passage I (Questions 1-6) This passage

Kaplan Explanations to AAMC Practice Test III Verbal Reasoning

Copyright © 2001, Kaplan, Inc. 3None of this material may be reproduced by any means or incorporated into any information retrieval system, without written permission.

6. Difficulty = Easy; Question Type = Implied Detail;The correct answer is choice A. The question stem asks which answer choice is likely to reflect the

relationship of "enforcement of drug-prohibition laws and the street prices of illicit drugs" as outlined by thepassage. In paragraph eight, the author states that drugs would be significantly cheaper if they were legalized.In this paragraph, the author also says that drug addicts presently spend "hundreds and even thousands ofdollars a week" for their drugs, as an underground market can typically demand a premium price for illicitsubstances. So, what do we expect? The most plausible relationship between enforcement and drug price isthat increasing enforcement would make drugs harder to buy and would raise prices. True. In actuality, thisissue is a simple exercise in supply-and-demand economics: supply is limited due to a drug's illicit nature, sodemand is high and prices are high. Once drugs are legalized, supply increases to resemble demand, andprices fall. Answer choice A directly states this: "The more strict the enforcement, the higher the streetprice." Thus, choice A is correct.

Choice B states that more strict the enforcement, the lower the price. This is the opposite of what weexpect to happen, and certainly, according to the passage, has not occurred. Thus, choice B is incorrect.Choice C is similarly misguided. This answer choice says that prices will rise with less drug enforcement.Again, we anticipate that less enforcement means easier availability of drugs, thus lowering prices. Thus,choice C states the opposite of what is expected, and is not correct. Finally, choice D states that there is noconnection between drug enforcement and street price. This is obviously not true, as the author draws a directcorrelation in paragraph eight. Therefore, choice D is also incorrect, and, again, choice A is the correctanswer choice.

Passage II (Questions 7-12)

This is an interesting natural sciences passage that addresses the myths and truths surrounding wolves. Theauthor spends a great deal of the passage discussing the complexity and intrigue surrounding wolves and theirbehavior. He is also concerned with the mysticism and the misconceptions about wolves. The author tries tounderscore the message that wolves are complicated and mysterious creatures, often maligned but rarelyclearly understood.

The first paragraph is simply a basic introduction to the general passage topic – the wolf. Here the author setsthe stage for his main theme, our misconceptions about the wolf.

The author introduces the Nunamiut Eskimos in the second paragraph. The Nunamuits, as presented by theauthor, have a deep understanding of the wolf. The author then elaborates on Nunamiut beliefs concerning thewolf throughout the second paragraph. The important aspect here is that the Nunamiut understand the complexinteractions of wolves and wolf packs. The Nunamiut appreciate the fact that wolves are not simple animalsacting only on instinct and primal urges. The author underscores this point by citing the inability of theNunamiut to generalize wolf behavior. This theme is continued throughout the third and fourth paragraphswhere the author highlights the myriad of wolves' social behaviors, hunting methods, coloration, andcommunications.

At this point, the author turns the discussion toward human opinions of wolves, and the lack of understandinginherent within. The author discusses the influence wolves can have on man, and how wolves' mysticism haslead to both wondrous admiration and fictitious beliefs among man. As the author states in the sixthparagraph, "Everything we have been told about wolves in the past...should have been said with the prefacethat it is only a perception in a particular set of circumstances, that in the end it is only an opinion." Thisdiscussion continues through the fifth, sixth, and seventh paragraphs.

In the eighth and ninth paragraphs, the author discusses his personal quest to better understand wolves, as wellas his pursuit to locate "a wolf different from that ordinarily given us in the course of learning about animals."The author concludes this discussion – and the passage – by stating that after all his elaborate research, hebelieves that man created his own impressions about the wolf, some factually-based, some pure imagination.

Page 6: Explanations to AAMC Practice Test IIIdocshare02.docshare.tips/files/14524/145247660.pdfAAMC MCAT PRACTICE TEST III VERBAL REASONING EXPLANATIONS Passage I (Questions 1-6) This passage

Kaplan Explanations to AAMC Practice Test III Verbal Reasoning

4 Copyright © 2001, Kaplan, Inc.None of this material may be reproduced by any means or incorporated into any information retrieval system, without written permission.

7. Difficulty = Easy; Question Type = Main Idea;The correct answer is choice A. This questions asks for the passage's "main idea." As discussed in

the above description of this passage, the author's main focus is the wolf's complexity and themisunderstandings surrounding the wolf. This idea is reflected in choice A, making it correct.

Choice B, wolves exhibiting a "wide and contradictory range of behaviors," is true based on evidencepresented in the passage. But this fact is too specific, being only one aspect of the characteristics of wolvespresented in the passage, not the passage's central theme. Choice C is likewise too narrow. Man's fascinationwith wolves, as stated in choice C, is only one aspect of the overall passage. True, man is fascinated bywolves, but the passage goes on to elaborate on the result of that fascination – man's misapprehensions aboutwolves – and this seems closer to the central idea. Choice D presents only part of the central argument. Theauthor does admit that the truth about wolves probably lies between the beliefs of wolf lovers and wolf haters,but this is only part of the main theme. The author also devotes a significant portion of the passage to themystery and wonder of wolves, not just man's perception of these animals. Thus, choice D is an insufficient,and choice A is the correct answer.

8. Difficulty = Moderate; Question Type = Implied Detail;The correct answer is choice B. An important note: the line reference in this question stem is

incorrect. The actual line references should be lines 32-35, not 42-45. That said, let's look at the question.This question asks what point the Bella Coola Indian legend makes. This legend states that someone tried tochange all animals into people, and they only succeeded with the eyes of the wolf. Paragraph five relates thecontext of this legend – individuals claim that a wolf can take your stare and return it to you, and the wolf'sstare elicits feelings of respect, fear, hatred, and curiosity. All of this supports the author's point that the "wolfexerts a powerful influence on the human imagination." Choice B paraphrases this, and is correct.

Choice A is not relevant to this legend. According to this answer choice, this story demonstrates thatpeople know very little about wolves. Well, it may be true that people know little about wolves, but that facthas no pertinence to this story. Also, the Bella Coola legend did not evolve out of ignorance, but out ofexperience with the wolf's stare. Thus, choice A is wrong. Choice C states that the Bella Coola legend provespeople revere the wolf. The passage does hint that some individuals treat the wolf with reverence, but that hasnothing to do with the story about the wolf's eyes. Choice D is nonsensical. Although there may be somepersonification in the Indian legend, that is not its central purpose. The legend is not trying to show howwolves resemble humans in certain ways. Thus, choice B is correct.

9. Difficulty = Moderate; Question Type = Implied Detail;The correct answer is choice C. An important note: the line reference in this question stem is

incorrect. The actual line reference should be line 45, not 59. Nonetheless, this question asks what the wordrigorous means in the context of this passage. The use is, "To be rigorous about wolves – you might as wellexpect rigor of clouds." The author has preceded this quote with an elaborate discussion of how wolves arecomplex creatures both in their physical attributes and their behavior. In this quote, the author implies thatone cannot expect clouds to be well-defined, and similarly, one cannot expect wolves to be well-definedeither. Thus, choice C, "precise" shares this meaning and is correct.

Choice A is a good distracter choice, but is incorrect. This choice presents one of the dictionarydefinitions of rigorous, "unyielding." But, the word rigorous is being used metaphorically here. As seen inlines 45-46, clouds are not "unyielding" but rather always changing. A good test-taking strategy: when askedfor the meaning of a word in a passage's context, the correct answer will typically not match the most commondictionary definition. Choice B is also incorrect. Although the passage does hint that some individualsconsider wolves to be harsh, that definition has no relevance to lines 45-46. Choice D is also not relevant inthe context of lines 45-46. In fact, choice D borders on the ridiculous. Wolves are never referred to as"judgmental" in the passage, and certainly, clouds are not judgmental. Thus, choice C is the correct answerchoice.

10. Difficulty = Moderate; Question Type = Logic;The correct answer is choice D. This question asks for the answer choice that would most weaken the

author's contention that "the wolf is less known that created by us." This means that most people's beliefsabout wolves are created by "Western" (white) man, not based in fact or on Indian/Eskimo legend.Remember, the author implies throughout the passage that Indians and Eskimos have a much betterunderstanding of the wolf; the author also implies that most "Western" truisms about the wolf are actuallyfiction. Choice D proposes that we have an abundance of scientific knowledge about the wolf, which

Page 7: Explanations to AAMC Practice Test IIIdocshare02.docshare.tips/files/14524/145247660.pdfAAMC MCAT PRACTICE TEST III VERBAL REASONING EXPLANATIONS Passage I (Questions 1-6) This passage

Kaplan Explanations to AAMC Practice Test III Verbal Reasoning

Copyright © 2001, Kaplan, Inc. 5None of this material may be reproduced by any means or incorporated into any information retrieval system, without written permission.

corresponds to Eskimo and Indian beliefs. This would greatly hurt the author's point, as it proves that Westernman did have accurate, detailed knowledge about wolves. Thus, choice D is correct.

Choice A is an incidental anecdote from the passage. The story of the wolf throwing a piece ofcaribou like a Frisbee is not directly related to the author's contention. It is only a small part of the passage,and, in fact, this anecdote was used to demonstrate wolves' mysterious behavior, not demonstrate Westernman's familiarity with wolf behavior. Thus, choice A is not applicable and incorrect. Choice B is a tricky one.The question asks for the fact that would most weaken the contention that the wolf is less known than createdby us. Certainly, if all Eskimo knowledge of wolves (which is believed to be accurate) was actually folklore,this would strengthen the author's theory, not weaken it – this would prove that most knowledge about wolvesis created, not known. Therefore, choice B is wrong. Similarly, choice C would also strengthen the author'shypothesis, not weaken it. Choice C suggests that all wolf behavior presently believed to be "intelligent" isactually instinct. This would prove that we know very little about wolves, and would support the author'sopinion that most of our knowledge about wolves is fictitious. So, again, choice C is wrong, and choice D iscorrect.

11. Difficulty = Moderate; Question Type = Application;The correct answer is choice B. Question 11 asks how the author would advise a party of historians to

approach a pack of wolves. Choice B indicates that the party should approach with "caution, as wolves arehunters, and some have been known to kill humans." This is correct. In paragraph five, the author states thatwolves have killed Indians and Eskimos, warranting caution. Also, the author regards wolves as cunninghunters, as indicated by the hunting skill he mentions in paragraph two. Thus, choice B best fits the author'sopinions, and is the correct answer.

Choice A claims that the author would advise the party to approach with love, as "a healthy wolf willseldom harm a human being." This is untrue, and the author directly contradicts this at the end of paragraphfive by disproving the statement "no healthy wolf ever killed anyone in North America." Thus, choice A isincorrect. Choice C states that the author would advise contempt toward wolves, as wolves are usually quitedangerous. Certainly, the tone of this passage makes it clear that the author does not regard the wolf withcontempt, but more accurately with respect. Although the author maintains a healthy respect for the wolf'shunting and killing abilities, he certainly does not consider them dangerous. Thus, choice C is wrong. Inchoice D, the author would advise the party to approach with confidence, as we know much more about thewolf now than in the past. This is obviously untrue – the entire passage addresses the concept that modernman knows far too little about the wolf. Choice D contradicts one of the main concepts of the passage, and isnot correct. Again, choice B is correct.

12. Difficulty = Easy; Question Type = Logic;The correct answer is choice C. This question asks how the author might include a 1928 description

of a wolf in this passage. The 1928 description defines the wolf as "a rapacious, flesh-eating animal belongingto the dog family." In the author's mind, this description would represent an antiquated and inaccurate accountof the wolf. The author would most likely use this account to bolster his argument that our knowledge of thewolf is primarily based in fiction. This is reflected in answer choice C, which states, "emphasize that what wethink we know about wolves is often the product of our imagination." Therefore, choice C is correct.

Choice A says that the 1928 description would be used to indicate that wolves are excellent hunters.Although this quote might support that point, the hunting ability of wolves is not a primary concern of theauthor, so it is unlikely that the author would use the 1928 quote to bolster this point. Thus, choice A isincorrect. Choice B implies that the 1928 quote would be used by the author to substantiate that people viewwolves as being like themselves. The author never emphasizes this point, and this answer is incorrect andnonsensical. Choice D implies that the 1928 account would help the author explain his own opinions aboutwolves. The entire passage addresses the point that wolves are complex creatures with a myriad ofpersonalities and characteristics. Therefore, to refer to the wolf as a savage, "rapacious" hunter wouldcontradict the entire point of the passage. Therefore, choice D is incorrect, and, again, choice C is thecredited response.

Page 8: Explanations to AAMC Practice Test IIIdocshare02.docshare.tips/files/14524/145247660.pdfAAMC MCAT PRACTICE TEST III VERBAL REASONING EXPLANATIONS Passage I (Questions 1-6) This passage

Kaplan Explanations to AAMC Practice Test III Verbal Reasoning

6 Copyright © 2001, Kaplan, Inc.None of this material may be reproduced by any means or incorporated into any information retrieval system, without written permission.

Passage III (Questions 13-18)

The language of this art history passage is much less straightforward than that of the precedingpassage on wolves. When working through a passage like this one, don't waste time pondering over obscurewords and concepts like "chiaroscuro" or "introduction of the Unconscious." If you find yourself getting hungup on anything, simply move on. If necessary, you can always come back to the passage when you'reanswering questions.

The first sentence of this passage deceptively leads you to believe that the author will beconcentrating on the relationship between the professional artist and the ruling upper class. In fact, the firstsentence is only a springboard for the overall argument of the passage, which is that the work of artists hasbeen limited by the artistic conventions they were trained to use. We'll come back to this misleading firstsentence when we discuss question 13.

Paragraph 1 argues that the conventions artists were trained to use apparently before the 19th centurywere closely tied to the social experience of the ruling class they served. In paragraph 2, the author uses FordMadox Brown's Work as an example of the difficulty 19th-century artists had when they tried to paint theexperience of other classes. They just couldn't get it right because they were still using the old conventions,which were better suited to depicting mythology than everyday life. Even when the old painting tradition wasdismantled in the 20th century, the artists' scope remained limited because they were being trained the sameway that they had always been.

What's ironic about all this limitation, the author points out in the final sentence, is that art claims tobe universal.

13. Difficulty = Moderate; Question Type = Main Idea;The correct answer is choice C. Remember how we said that the first sentence of the passage

misleads you into thinking that the main idea has to do with the relationship between the artist and the rulingclass? Take a look at choice A--there it is, a paraphrase of the first sentence. If you didn't go through the restof the passage and realize that the author is mainly concerned with the fact that artists have been limited bytheir training in artistic conventions C, choice A would be very tempting.

Choice B may be true, but it focuses solely on the first paragraph. The main idea has to be muchbroader in scope. Choice D twists information from the second paragraph to the effect that artists whoattempted to extend the tradition were met with opposition. Those artists were not abandoning conventionalmethods (which was their pitfall), and we don't know who opposed them.

14. Difficulty = Moderate; Question Type = Detail;The correct answer is choice B. The professional tradition of painting is first mentioned in the

beginning of the second paragraph. "Tradition" refers to the set of artistic conventions discussed in the firstparagraph, which makes B correct. You could also arrive at B by noting that in paragraph 4 "dismantling ofthe tradition" and "dismantling of conventions" mean the same thing.

A can be ruled out because the author would certainly not say that the tradition was the "best way torecord and preserve artistic truths" (although artists prior to the nineteenth century may have thought so). C isa misreading of the second paragraph: Brown did not intend to make his subjects look mythological orsymbolic, so there is no reason to think that the tradition involved applying such elements to realistic subjects.Choice D occurred after the tradition was dismantled, so it is wrong as well.

15. Difficulty = Moderate; Question Type = Implied Detail;The correct answer is choice A. Since the author argues throughout the passage that conventions have

hindered artists by limiting the area of experience they could draw from, the implication is that artists wouldbe much better off without conventions A.

This question asks for the author's opinion of what constitutes "art at its best," not the opinion of pre-nineteenth-century artists concerning the best art, so choice B is out. In the first paragraph, the author outlinesthe limitations of the art that reflected the social manners of the upper class, which eliminates choice C. Asfor choice D, art that "treats religious themes" is never discussed in the passage (Brown's Work just looksreligious).

16. Difficulty = Moderate; Question Type = Inference;The correct answer is choice D. The answer to this inference question must be in one of the last three

short paragraphs since it deals with the crisis of modern painters. Paragraph 3 explains that the professional

Page 9: Explanations to AAMC Practice Test IIIdocshare02.docshare.tips/files/14524/145247660.pdfAAMC MCAT PRACTICE TEST III VERBAL REASONING EXPLANATIONS Passage I (Questions 1-6) This passage

Kaplan Explanations to AAMC Practice Test III Verbal Reasoning

Copyright © 2001, Kaplan, Inc. 7None of this material may be reproduced by any means or incorporated into any information retrieval system, without written permission.

tradition was dismantled in the twentieth century but that the area of experience drawn upon by Europeanartists was still narrow. Paragraph 4 gives one of the reasons for this: the training of painters had not changedbecause there was no other body of knowledge to tap, aside from the old conventions. This makes D thecorrect answer.

You can rule out choice A and choice B right away because the author never says anything about lackof art appreciation or about viewers of art being disturbed by depictions of the unconscious. Choice C isincorrect because the relationship between modern painters and the ruling classes is not discussed in the lastthree paragraphs.

17. Difficulty = Hard; Question Type = Tone;The correct answer is choice D. This is one of the most difficult questions of the first four or five

passages because it requires you to key in on a minor detail and draw an inference from it. In the firstparagraph, the clue to why artistic conventions corresponded to the social manners of the ruling classes existsin lines 8-11: "...these conventions corresponded so closely to the social experience...of the class [the artist]was serving..." The fact that artists served the ruling classes implies that artists derived some sort of benefit--probably financial support--from the members of the ruling classes in return for their paintings D.

It may be easier to get the right answer to this question by eliminating the wrong ones. Choice A is adistortion of the information that twentieth-century paintings often dealt with isolation and narrowness. ChoiceB is contradicted by the entire second paragraph. Finally, there is no reason--based on the passage--to thinkthat artists desired the wealth and power of the ruling classes, making choice C correct.

18. Difficulty = Moderate; Question Type = Implied Detail;The correct answer is choice A. This logic question tests your understanding of the reason the artist

included a discussion of Ford Madox Brown's painting in the passage. In lines 24-26, the author announcesthat he will give an example of how difficult it was for certain nineteenth-century artists to extend the traditionof painting; Brown's painting is the example. A paraphrases the author's stated purpose for examining thisparticular work, so it is correct.

Abstract art is not discussed until much later in the passage, which rules out choice B. Choice C isincorrect because Brown's painting was done in the nineteenth century and thus tells us nothing about modernart. Choice D is a restatement of the first sentence of the passage, whereas Work is discussed in the secondparagraph, which contains no reference to the relationship between the professional artist and the rulingclasses. Therefore, D is wrong as well.

Passage IV (Questions 19-25)

The passage discusses the formation and subsequent cooling of the earth, focusing on the implications of andevidence for heat loss.

The first paragraph describes how the interior of the planet heated up and tells us that it is still cooling down.The planet formed as a result of gravity, the energy of which melted most or all of the earth's constituents.Energy release also came from the separation of materials by density, with heavier materials collecting in thecenter and the lighter ones - the ones which formed the mantle and crust - collecting closer to the planet'ssurface. The rate of cooling is given as "tens of degrees every billion years."

The second paragraph discusses the heat that comes from radioactive decay in the earth's interior. It isminimal compared to that given off by the sun. However, this heat of radioactive decay, which originates fromthe earth's mantle, accounts for processes such as hot springs and volcanoes.

The third paragraph describes the earth's core: a sphere about 4,000 miles across made mostly of iron.Currently, about a third of it is solid. This inner core is much hotter than the liquid-state outer core, but higherpressure keeps it solid.

The next paragraph links the earth's cooling to its magnetic field. The field is caused by electric currents, andwithout a constant energy supply, the earth's currents would have deteriorated, leaving the planet without asustainable magnetic field. The dynamo theory, which is described in more detail in the next paragraph,attributes the earth's magnetic field (and its periodic reversals) to heat loss in the core. The heat loss drivesfluid motions which create electric fields, which in turn sustain the earth's magnetic field.

Page 10: Explanations to AAMC Practice Test IIIdocshare02.docshare.tips/files/14524/145247660.pdfAAMC MCAT PRACTICE TEST III VERBAL REASONING EXPLANATIONS Passage I (Questions 1-6) This passage

Kaplan Explanations to AAMC Practice Test III Verbal Reasoning

8 Copyright © 2001, Kaplan, Inc.None of this material may be reproduced by any means or incorporated into any information retrieval system, without written permission.

The last paragraph takes a more detailed look at the physics of a dynamo and concludes that the earth's outercore must be a smoothly functioning dynamo. Otherwise, the earth wouldn't have the electric and magneticcharacteristics that it does.

19. Difficulty = Moderate; Question Type = Implied Detail;The correct answer is choice A. This implied detail question asks which answer choice does NOT

contribute to the earth's level of internal heat. The passage mentions all the choices except A, fluid motionsin the outer core. Choices B and D, primordial gravitational processes and separation of materials by density,are described in the first paragraph. Choice C, the decay of trapped radioactive elements, is mentioned in thesecond paragraph.

20. Difficulty = Moderate; Question Type = Application;The correct answer is choice C. The information relevant to this question is found in the beginning of

the second paragraph. The passage says that 42 terawatts of energy is released as heat from the earth'ssurface. The next sentence in the passage says that the heat received from the sun is about a thousand timesthe amount released from earth's surface. Therefore, the amount received from the sun, in terawatts, would be(42 X 1000) or 42,000 terawatts, answer choice C.

21. Difficulty = Hard; Question Type = detail;The correct answer is choice C. This detail question is drawn from the third paragraph in which the

passage describes the earth's core as comprised of mostly iron and perhaps nickel. We learn in lines 28-30that at some point the core was probably completely molten but "by now its inner third is solid," (implying thatthe other two-thirds is not solid). Hence, the correct answer choice is "fluid and metallic", choice C.

Choice A is incorrect, as it confuses information from the second paragraph and falsely states that thecore is primarily solid. Choice B would be correct if the core were gaseous; however, the passage states thatthe core is molten. Choice D would be correct if the question asked about just the inner core rather than theentire core.

22. Difficulty = Moderate; Question Type = Detail;The correct answer is choice B. In the fourth paragraph, you find the following: "Because

magnetized rocks show that the earth has had a magnetic field for at least three billion years..." This impliesthat it is data from rocks which serve as evidence for earth's continuous magnetic field, making choice B thebest answer.

Choice A refers to information discussed elsewhere in the passage that is not directly related to earth'smagnetic field. Thus, choice A is wrong. Choice C is never mentioned in the passage, and is thereforeincorrect. Choice D might seem to be plausible, since it refers to concepts related to a dynamo. However,rocks themselves are never described as converting energy. Furthermore, dynamo activity itself is not cited asevidence for a continuous magnetic field. So, choice D is not correct. Again, choice B is the correct answer.

23. Difficulty = Moderate; Question Type = Detail;The correct answer is choice A. This detail question refers to information from the fourth paragraph of

the passage. In the second-to-last sentence, it says, "The only theory that can explain the persistence of thefield and its propensity for reversing itself is the dynamo theory..." Answer choice A is most consistent withthe passage, and is correct.

Choice B is never mentioned in the passage and is incorrect. Choice C is not specifically related tothe dynamo theory specifically, so it is wrong. Finally, choice D is not related to the dynamo theory, althoughelectrical conductivity in the outer core is necessary for the function of the earth's dynamo.

24. Difficulty = Moderate; Question Type = Detail;The correct answer is choice D. This question is drawn from the last sentence, which describes the

workings of a dynamo. The second sentence says, "...the induced electric current generates its own magneticfield..." Thus, electric currents produce magnetic fields; choice D is correct. Choice A, fluid motion, is animportant theme in the passage, and the fourth paragraph does say that the magnetic field results from fluidmotions in the outer core. However, the question stem specifically asks what magnetic fields are primarily by-products of? The last paragraph explains the process in greater detail, describing the connection betweenelectric and magnetic fields. Choice B is irrelevant. The rotation of the earth and magnetism are not directly

Page 11: Explanations to AAMC Practice Test IIIdocshare02.docshare.tips/files/14524/145247660.pdfAAMC MCAT PRACTICE TEST III VERBAL REASONING EXPLANATIONS Passage I (Questions 1-6) This passage

Kaplan Explanations to AAMC Practice Test III Verbal Reasoning

Copyright © 2001, Kaplan, Inc. 9None of this material may be reproduced by any means or incorporated into any information retrieval system, without written permission.

linked in the passage, so choice B is incorrect. Choice C is vaguely related to magnetic fields, butmechanical energy and magnetic energy are only indirectly related (as evidenced in a dynamo).

25. Difficulty = Moderate; Question Type = Tone;The correct answer is choice B. This question stem asks which answer choice is most similar to the

convergence of disciplines necessary for the analysis and theories described in the passage. The theory in thepassage required the combination of geology, physics, mathematics, and chemistry. Choice B is the answerthat best matches this association from the passage. As stated in the question stem, the passage requires thecombination of disciplines to understand a commonly-linked situation. Choice B similarly links the threedisciplines of art, history, and technology to understand a common element – ancient civilizations.Choice A describes a process of inspiration, not scientific inquiry. Choice C does not involve a synthesis ofdifferent disciplines but rather the study of subjects of different ages. Choice D describes a single direction ofinquiry rather than a varied, broad approach across disciplines.

Passage V (Questions 26-35)

The authors of Passage V make their central claim in the first couple of paragraphs. They assert thatmetaphor is not just a poetic device or a feature of language; it is central to our conceptual system. Since ourconcepts govern how we perceive the world, think, and act, and our conceptual system is largelymetaphorical, this means that how we think and what we do depends on metaphor.

The authors turn in the fifth paragraph to an examination of ordinary language in order to provideevidence for their thesis. The example they focus on is ARGUMENT IS WAR, a metaphor they say isreflected not only in the phrases in lines 40-45 but also in the way we look at arguing in general in our culture.In the final paragraph, the authors propose that if another culture viewed argument in terms of dance, argumentwould take on an entirely different meaning. Someone from our culture would not regard this culture's"argument" as an argument at all.

26. Difficulty = Easy; Question Type = Main Idea;The correct answer is choice C. If you followed the thread of the authors' argument, the correct answer

to this main idea question should jump out at you. Let's take the choices one-by-one. Choice A is too narrowin scope: it's the main idea of the third paragraph, not the passage as a whole. Choice B is wrong because thediscussion of different cultural views of argument comes only in the final paragraph. In fact, this discussion isonly one of the ways the authors back up their central thesis that "metaphors control our perceptions, thoughtsand actions." Choice D is drawn from the first sentence and is wrong because the authors do not discussmetaphor as a poetic or rhetorical device in the passage.

27. Difficulty = Easy; Question Type = Tone;The correct answer is choice C. The authors point out in paragraph 6 that we plan and use strategies

in arguments just as we would if we were engaged in a battle--arguing "is partially structured by the concept ofwar" (line 52). In other words, arguments are battles to us and we use planning and strategy in order to win.Again, C is correct.

Choice A stretches too much in making the inference that planning means thinking before you speak.Even more important, this choice ignores the overall idea behind paragraph 6. The authors never say or implyanything about schools, so Choice B is out. Choice D might seem to make sense but, like A, it ignores thecontext in which the authors mention the use of planning and strategy in arguments.

28. Difficulty = Moderate; Question Type = Application;The correct answer is choice C. This question asks you to apply information from the passage to a

new set of expressions. Well, the authors say that the metaphors we use in expressions reveal the way weperceive and think about things. The common element in the expressions given in the question stem is thatenergy is some sort of quantifiable substance that we can have none of, a lot of, or even too much of. Theauthors would say, then, that people think of energy as a substance C.

Choice A and choice B might be reasonable inferences to make from the expressions if you hadn'tread the passage, but the question stem tells you to keep the claims of the passage in mind, so these choicesare wrong. There is no connection in the passage between energy levels and arguing, so choice D is wrong.

Page 12: Explanations to AAMC Practice Test IIIdocshare02.docshare.tips/files/14524/145247660.pdfAAMC MCAT PRACTICE TEST III VERBAL REASONING EXPLANATIONS Passage I (Questions 1-6) This passage

Kaplan Explanations to AAMC Practice Test III Verbal Reasoning

10 Copyright © 2001, Kaplan, Inc.None of this material may be reproduced by any means or incorporated into any information retrieval system, without written permission.

29. Difficulty = Easy; Question Type = Application;The correct answer is choice B. This question is very similar to the previous one. Paragraph 6 makes

clear that the metaphors we use in language reflect how we think about things. If a person talks about anargument in terms of winning and losing, the authors argue, this shows that she thinks of an argument as a war,or a "contest" B.

This does not mean, however, that she thinks arguments are violent choice A; this is a choice you canrule out using common sense. Choice C is what the authors think, not what the speaker of the question stemwould think. Choice D can be eliminated because nothing in the passage refers to competition as unpleasantD.

30. Difficulty = Moderate; Question Type = Tone;The correct answer is choice A. To answer this question, take a look at each of the choices to see if

one jumps out at you. Be careful, though. Choice B immediately looks interesting because half the passagetalks about arguments and B mentions debate, but stop to consider: would it help a senator in his work if youtold him that metaphor governs thought, that people think about arguing in terms of war and we know this bylooking at language etc.? Probably not. Would it help a general getting ready to fight choice D or a financialanalyst, choice C? No. The person who might benefit from reading the passage would be the ambassador to adifferent culture. It would be very useful for the ambassador to know that you can learn a lot about the waypeople from other cultures think by paying attention to the metaphors they use in language. Therefore, choiceA is the best response.

31. Difficulty = Easy; Question Type = Implied Detail;The correct answer is choice D. The authors essentially present a theory of metaphor in the passage,

so all you have to do to answer this question is pick out the Roman numeral options that the authors wouldagree with. They say in the second paragraph that how we think and what we experience is largely a matter ofmetaphor. Options I and II paraphrase this idea; to say we understand and experience one thing in terms ofanother is the same thing as to say that how we think and what we experience is a matter of metaphor. Thisnarrows the possible correct choices down to two, choice C and choice D. Since the authors would also agree(as would anybody who understands metaphor) that metaphor involves "communicating about one thing interms of another," option III is true and choice D is correct.

32. Difficulty = Moderate; Question Type = Application;The correct answer is choice A. Forget about the passage for a moment: you have to figure out what

madness is being compared to in the expression "this is driving me around the bend." Resist your inclinationto go for the obvious, "a road," choice C, or "a vehicle," choice B. "Around the bend" in this expressionmeans "mad," so madness is being compared to a location A. Think about it this way: if you say something is"driving you around the bend," that means that where you are now is sanity, but when you go mad you willhave moved to a different place, "around the bend." Choice D is wrong because if something is driving youaround the bend, it's that thing that is the force driving you to a state of madness. Madness itself is not theforce.

33. Difficulty = Moderate; Question Type = Implied Detail;The correct answer is choice B. Go directly to the last paragraph, in which the idea of argument being

viewed as dance in another culture is addressed. The goal of this foreign type of arguing, according to lines64-65, would be to "perform in a balanced and aesthetically pleasing way," not to fight a battle and win. Bcomes closest to meeting the goal of performing in a balanced way, and is therefore correct.

Choice A would be a strategy of the argument-as-war approach. Explaining to someone from theforeign culture that their whole concept of argument is wrong probably wouldn't go over very well, so choice Cis out. As for choice D, nothing in the passage suggests that irrationality has anything to do with the argument-is-dance type of discourse.

34. Difficulty = Easy; Question Type = Detail;The correct answer is choice D. As you know by this point, having been through most of the questions

in the set, arguments in our culture are supposedly based on the metaphorical concept that "argument is war."The evidence for this is presented in paragraph 6. Choices A, B and C are all either direct quotes orparaphrases from the first half of the paragraph. D, on the other hand, is not mentioned by the authors at all, soit has to be the correct answer.

Page 13: Explanations to AAMC Practice Test IIIdocshare02.docshare.tips/files/14524/145247660.pdfAAMC MCAT PRACTICE TEST III VERBAL REASONING EXPLANATIONS Passage I (Questions 1-6) This passage

Kaplan Explanations to AAMC Practice Test III Verbal Reasoning

Copyright © 2001, Kaplan, Inc. 11None of this material may be reproduced by any means or incorporated into any information retrieval system, without written permission.

35. Difficulty = Moderate; Question Type = Implied Detail;The correct answer is choice C. The last question in this set is a straightforward detail question. The

reasons people think they can get along without metaphor are put forward in the first two sentences of thepassage. First, most people think metaphor is a poetic and rhetorical device not used in ordinary language.Option I paraphrases this. Second, most people see metaphor as merely a characteristic of language, notthought or action (Option II). This leaves us with choices C and D as possible answers. Nothing in the firstparagraph indicates that people think metaphor "alters one's perception of reality," so option III and choice Dare incorrect, leaving choice C as the correct answer.

Passage VI (Questions 36-45)

This passage is about the Confucian idea of li, that acting in accord with "ceremony" and "ritual" reveals thosecharacteristics which are most uniquely human.

The first paragraph introduces the Confucian idea of li. Distinctively human characteristics, as "obvious" asthey are, have a magical quality about them. This quality is revealed through li.

In the second paragraph, we get a general idea that li is somehow related to ritual and ceremony. Ability andwillingness to act by li are, according to Confucius, necessary for reaching individual power and virtue.

In the next paragraph, the idea of li is described in the context of leadership. Coercion and force, which aretangible and manifest, are contrasted with the more vast, sacred, graceful force of li , which worksspontaneously and is rooted in reverence.

Paragraph 4 describes an example of using li to accomplish a physical task - retrieving a book "magically" byway of "ceremony" (social politeness) and "ritual" (appropriateness).

The next paragraph develops the idea of word power described in the previous example. It introduces the ideaof ritualistic words, phrases which by their very utterance execute their intentions.

In paragraph 6, we get an example of such a statement: a bequest. In saying it, the act is completed. Thus, bywords and ceremony, one may be bound to action. In keeping with li, one does not need to employ strategy orforce.

The last paragraph clarifies the point about ceremonial language. In many areas of human life, the process, orexistence, is itself ceremonial. Examples of this are promises, pacts, and commitments.

36. Difficulty = Hard; Question Type = Main Idea;The correct answer is choice D. This question stem asks about the passage's main idea. As stated

above, the passage is primarily concerned with the Confucian notion of li, a way to see true "human powers"as expressed through activities of ceremony or social ritual. This is most consistent with answer choice D.

Choice A refers to promises, commitments, excuses, etc. These are indeed described in the lastparagraph but they are not the main focus of the author. So, choice A is incorrect. Choice B misconstrues anexample from the passage as being indicative of manipulation. The passage describes the "magic," not themanipulation, of li, so choice B is wrong. Choice C is not on target with the general scope of the passage, butonly highlights a point discussed in paragraphs 5 and 6. Thus, choice C is also incorrect. Again, choice D iscorrect.

37. Difficulty = Hard; Question Type = Logic;The correct answer is choice A. Based on the author's discussion of li, it is clear that one of the basic

tenets of li is respect for others. Confucius contrasted leaders who use li with those who use force andpunishment. Since li operates spontaneously and is "rooted in reverent dignity", it is most consistent withanswer choice A, the credited response.

Choice B is a trite, simplistic interpretation of the meaning of li and is not what Confucius wouldconsider to be a reason for acting in accord with li. So, choice B is incorrect. Choice C refers to the fact thatli acts spontaneously; however, this is not cited as a reason for learning li, so choice C is wrong. Choice D

Page 14: Explanations to AAMC Practice Test IIIdocshare02.docshare.tips/files/14524/145247660.pdfAAMC MCAT PRACTICE TEST III VERBAL REASONING EXPLANATIONS Passage I (Questions 1-6) This passage

Kaplan Explanations to AAMC Practice Test III Verbal Reasoning

12 Copyright © 2001, Kaplan, Inc.None of this material may be reproduced by any means or incorporated into any information retrieval system, without written permission.

adds extraneous information, never addressed in the passage. The author does discuss the use of force asantithetical to li, but never explicitly describes the risks of using force. So D is also incorrect.

38. Difficulty = Hard; Question Type = Implied Detail;The correct answer is choice C. This Roman numeral question asks which ideas are supported by an

example. It is best to consider each statement separately, and eliminate answer choices in the process. Theauthor provides an example in the fourth paragraph for option I's assertion that "some human powers have amagical quality": the example is the magical quality of getting a book by politely asking a student to go getit. The bequeathal of the watch at the beginning of the sixth paragraph is the example the author uses tosupport option II, "words can function as actions." Option III, however, has to be ruled out because the authornever asserts that "much of human existence consists of ceremony" nor does he provide a specific example ofa specific human ceremony to support this. He does say that we have come to see that the area of humanexistence that involves ceremony is vast; however, saying this and saying, "much of human existence consistsof ceremony" are two different things. The area of human existence that involves ceremony could be giganticand still be only a small portion of overall human existence. In any case, III has to be wrong: we already knowthat options I and II are correct, and there is no answer choice that lists options I, II, and III as all correct. C isthe correct answer.

39. Difficulty = Moderate; Question Type = Logic;The correct answer is choice A. The magic of li is explained in the fourth paragraph through an

example. You already know that li is social ceremony and ritual rooted in reverent dignity. The magic in liis that through proper ritual expression of a wish you can have it fulfilled without any effort on your part. Forexample, you can ask your student politely to go get a book you want, and he'll bring it to you--you don't haveto move. It's the power of ordinary social conventions like this that Confucius sees as "magical" (choice A).

Choice B gives a standard interpretation of the word "magical" that is never used in the passage, so Bis incorrect. Choice C also sticks too closely to the meaning of magical as supernatural. Also, the passagenever speaks about learning li only from Confucian masters. Thus, choice C is incorrect. Don't be fooled bychoice D: it only distorts the point of the example in the fourth paragraph. This example demonstrated the"magic" of social convention, in keeping with the idea of the passage as a whole; it was not meant to showthat some people have a truly magical power to move physical objects with words, nor is the main point of lithe movement of physical objects. So, choice D is not correct, and, again, choice A is correct.

40. Difficulty = Moderate; Question Type = Application;The correct answer is choice B. This is another Roman numeral format question. Question 40

addresses "perfomative utterances." The fifth and sixth paragraphs describe performative utterances –statements for which the utterance is the act itself. Option I does not represent an act itself – it is a request.Similarly, option III is also a request not representative of an act. Of the options presented in the question,only option II conforms to the style of phrase described in the passage. Therefore, answer choice B, option IIonly, is correct.

41. Difficulty = Moderate; Question Type = Inference;The correct answer is choice A. You know from paragraph two that in li, social ritual and ceremony

are holy and sacred. What makes them so holy? Focus on the end of the third paragraph: "li works throughspontaneous coordination rooted in reverent dignity. The perfection in Holy Rite is esthetic as well asspiritual..." Even through the abstract language, you can see that what makes these social ceremonies specialis that they involve "reverent dignity." And the emphasis on ceremonial politeness in the fourth paragraphconfirms it for us: Confucius saw social ceremonies as "expressions of respect" (choice A).

Choice B is incorrect, as "divinely sanctioned laws" is not a topic of the passage. "Refined society"is never mentioned in the passage, so choice C is wrong. Choice D is inconsistent with the passage.According to the passage, all persons may perform rituals, so the concept of limiting rituals to holy personsmakes choice D incorrect.

42. Difficulty = Moderate; Question Type = Inference;The correct answer is choice C. In the fifth paragraph, Austin's idea of performative utterances is

described. According to the passage, these are phrases which, by their very execution in language, completethe action described within the statement. This is best covered in choice C, language and actions aresometimes the same thing.

Page 15: Explanations to AAMC Practice Test IIIdocshare02.docshare.tips/files/14524/145247660.pdfAAMC MCAT PRACTICE TEST III VERBAL REASONING EXPLANATIONS Passage I (Questions 1-6) This passage

Kaplan Explanations to AAMC Practice Test III Verbal Reasoning

Copyright © 2001, Kaplan, Inc. 13None of this material may be reproduced by any means or incorporated into any information retrieval system, without written permission.

Choice A is the reverse of what Austin implies and is therefore incorrect. The concealment of facts isnever covered in the passage, so choice B is not correct either. Choice D is too general and does not addressperformative utterances specifically.

43. Difficulty = Hard; Question Type = Logic;The correct answer is choice A. The correct answer is the one which most weakens the idea in the

question stem: that words are more binding than strategies or force. Choice A indeed weakens the argument,because if people don't take obligation seriously, the power of performative utterances is diminished. This isthe correct choice.

Choice B is irrelevant to the argument and incorrect. The question stem is concerned with whathappens when people act through words or ceremony, not with how often people do or not. Choice C isincorrect: the question is about people binding themselves with words or force, not about how frequently theyrespond to one or the other. Choice D, that people don't make promises, seems tempting, but it doesn'tweaken the author's argument because people can avoid making promises and still be more bound by themthan by force. D is therefore off-target and incorrect. Choice A, in comparison, suggests that people don'tkeep promises, not that they don't make them. This is a more direct challenge to the idea that words andceremony are binding.

44. Difficulty = Hard; Question Type = Application;The correct answer is choice B. Since the passage deals with the hidden power of ceremony, choice

B is the best answer. A Confucian would be likely to value someone who was sensitive to the effects ofceremony.

Choice A is not in keeping with the main idea of the passage – magic is not the focus here. Thus,choice A is wrong. Choice C overstates an example from the passage - using words and social politeness toaccomplish tasks through others – and is also incorrect. Choice D is not mentioned or implied in the passage.

45. Difficulty = Hard; Question Type = Application;The correct answer is choice C. The question stem gives additional information to consider: people

can be polite towards those they do not respect. Now, in the fourth paragraph the author says that he can turn"politely; i.e. ceremonially" to a student and express in an "appropriate and polite (ritual) formula" what hewants. The problem here is that if ritual and ceremony mean about the same thing as politeness, as thesentence seems to suggest, then as far as we know he could be using ritual and ceremony without feelingrespect towards the student. This, however, contradicts the author's point as well as the whole idea of li. Itjust doesn't seem that words like "polite," "ritual," and "ceremony" can really convey the reverence of li(choice C).

Choice A is not pertinent to the question's stem; invisibility and respect are not connected in thepassage. A is therefore wrong. For choice B to be correct, li would have to be mere word power and socialmanipulation; this cannot be the case since li is by definition rooted in reverent dignity. The author wouldrespond to the accusation of choice B by pointing out that when people are polite without feeling respect orwhen they're manipulative, this is simply not an instance of li. B is incorrect. Choice D is incorrect because,based on the definitions of li from the passage, it is clear that contempt is NOT a tenet of li. Also, whenpeople are being polite toward those whom they do not respect, they are not expressing contempt, so D doesnot follow from the question stem.

Passage VII (Questions 46-51)

The author of Passage VII takes three paragraphs to develop a simple idea: due to the current way ofteaching literature, students are missing the joy of reading.

The first paragraph begins to explore the reasons that teachers look down on popular, entertainingliterature. It's not just snobbery, the author says; the teachers have been put off by the "boring sameness" andthe "merely commercial" stuff. But he thinks that some popular artists and works have more to offer than theirmore sophisticated counterparts.

The author goes on in the second paragraph to suggest that there may be another explanation forteachers' dismissal of popular literature, The nature of education and its administrators is such that bookshave come to be seen as sources of useful information, not as life-enhancing experiences.

Page 16: Explanations to AAMC Practice Test IIIdocshare02.docshare.tips/files/14524/145247660.pdfAAMC MCAT PRACTICE TEST III VERBAL REASONING EXPLANATIONS Passage I (Questions 1-6) This passage

Kaplan Explanations to AAMC Practice Test III Verbal Reasoning

14 Copyright © 2001, Kaplan, Inc.None of this material may be reproduced by any means or incorporated into any information retrieval system, without written permission.

He continues to complain about this in the third paragraph, and then concludes that althoughunderstanding the mechanics of good writing is important, appreciating the importance of the delight a bookcan give is essential.

46. Difficulty = Moderate; Question Type = Implied Detail;The correct answer is choice C. Note: according to the answer key of this test, B is the correct

answer; however, upon careful review of the question, we feel that a stronger argument can be made for choiceC. In addition, note that the line reference in the question stem is incorrect; it should be line 13.

The author argues in the first paragraph that since we are put off by the commercial nature and theshoddiness of much of popular entertainment, we miss the fact that some of it is good. For example, he says,some popular songwriters are really "dedicated, energetic poets"--especially in comparison with the "wearysophisticates...and randy academics." It is this comparison with more academic poets that makes C the bestanswer.

Choices A, B and D are all wrong because you will not find testimony by literature teachers,examples of popular song lyrics, or an analysis of Coleridge's poetry in the first paragraph or anywhere else inpassage.

47. Difficulty = Moderate; Question Type = Inference;The correct answer is choice D. The author criticizes the use of books as instructional tools rather

than as "incomparably rich experiences" (line 39), so it can be inferred that he thinks books should be taughtas "engaging experiences" D.

The author also denigrates the practices of making life-enhancement courses "objective" (line 36) andof using books to "illustrate major themes in American literature" (line 41), so both choice A and choice B arewrong. He acknowledges that analysis choice C is important, but D is the best answer because this is what heargues for throughout the passage.

48. Difficulty = Easy; Question Type = Application;The correct answer is choice D. Look for the answer that fits in with the main idea of the passage.

The author's complaint is that books are taught not because they give joy but because they serve todemonstrate good composition (lines 54-55) and because they are "good for us." D is correct.

You can eliminate choice A and choice C; it seems there is already too much of an emphasis onteaching students to read critically and to recognize good literature. Choice B is wrong because the popularworks, not the literary works, lack originality.

49. Difficulty = Moderate; Question Type = Application;The correct answer is choice C. Whatever its faults, drugstore fiction "may have more to offer than

fiction thought to be of a higher class" (line 17) since it has entertainment value. Students would probably"enjoy the classes more," choice C, just because drugstore fiction has an "immediate appeal" that literaryworks do not.

However, since drugstore fiction isn't really literature, it's unlikely that teachers would learn muchabout their students' reading skills if they taught drugstore fiction, choice A. For the same reason, it's alsounlikely that students would learn more about literary terms, choice D. Based on the passage, there's noreason to think that committees would change curriculum objectives just because some drugstore fiction wasintroduced, so choice B is incorrect as well.

50. Difficulty = Moderate; Question Type = Inference;The correct answer is choice A. This question is just another variation on the same theme, so you can

predict the answer before looking at the answer choices. You know that the author doesn't like the fact thatliterature is taught as something "good for you" and instructional. He thinks the emphasis on the criticalanalysis of themes and composition has led to a lack of regard for the "immediate interest" of literature. Whenyou look for this idea among the answer choices, A is the one that fits.

Choice B and choice D both contradict paragraph 2, in which the author complains about the fact thatliterature courses are treated not as life-enhancing courses but as courses in useful information. Choice C iswrong because the author suggests that there is something to be said for the pleasures of commercialentertainment.

Page 17: Explanations to AAMC Practice Test IIIdocshare02.docshare.tips/files/14524/145247660.pdfAAMC MCAT PRACTICE TEST III VERBAL REASONING EXPLANATIONS Passage I (Questions 1-6) This passage

Kaplan Explanations to AAMC Practice Test III Verbal Reasoning

Copyright © 2001, Kaplan, Inc. 15None of this material may be reproduced by any means or incorporated into any information retrieval system, without written permission.

51. Difficulty = Moderate; Question Type = Logic;The correct answer is choice D. If most students are reading Asimov in literature class, then teachers

are not purposely ignoring popular literature. In other words, they are not "prejudiced against popular genres,"choice D.

Just because students are reading Asimov, this doesn't mean that Asimov is either simple-minded orlasting, so choice A is wrong. Choice B is out because it is easily possible that literature courses that includeAsimov are still meeting prescribed curriculum objectives. Choice C makes no sense: we can't judge howmuch the people determining school curricula know about literature from the fact that students read Asimov.

Passage VIII (Questions 52-59)

This is a social sciences passage concerning the ancient Greeks' religious worship. Specifically, this passagefocuses on the evolution of religious beliefs throughout the Hellenistic age.

The passage is about average in length, but it presents several unusual names and places, which canoccasionally trip up students. One tip is to underline or mentally note each name and what it refers to whenyou encounter it. Many of the questions require you to refer back to specific people and their beliefs.Normally, students should skim over large names or terms in Verbal Reasoning passages and return to themonly if required by a question. But this passage centers around these names and terms, and skimming overthem would mean losing a good amount of the passage's meaning.

In the first paragraph, the author discusses the earlier Greek's occasional irreverence toward the gods. Theauthor cites occurrences of disregard and "familiar flippancy" such as those found in the Iliad and HomericHymns, as well as Euripides' portrayal of the gods as "shady seducers," "figures of fun," and "psychologicalforces." The author also cites the fact that Socrates was accurately accused of "not believing in the gods inwhom the city believes."

The author continues his discussion into the second paragraph by addressing changes that evolved in the earlyHellenistic age. The author notes that many individuals began to regard the gods as merely symbolic, andeven the Stoics, who maintained a strong belief in Providence, often regarded many gods as simply"allegorical explanations of natural phenomenon." The author adds that two poets of this age, Callimachusand Theocritus, proved throughout their writings that the gods in this age were no longer a matter of faith orconsequence.

In the third paragraph, the author continues to present evidence of increasing disregard for the gods. Theauthor cites Euhemerus who, in an effort to flatter existing monarchs, claimed that several gods, includingUranus and Zeus, may have once been human monarchs. Euhemerus' declaration may have really been aneffort to rationalize atheism; after all, Strato of Lampsacus was also declaring that the gods were unnecessaryin order to construct an understandable world. Further, the author cites that several scholars of this time beganto declare that the gods either did not exist or were apathetic towards man. The author supports the claim ofincreasing paganism by quoting St. Paul who called this era a "world without hope – and without God."

In the final paragraph, the author cautions that St. Paul's description is misleading, as Paganism was notovershadowed by atheism but had just departed from traditional mainstream Greek beliefs. In fact, the authornotes that Paganism was alive and well when Christianity eventually overtook it. The author notes that manyceremonies to Pagan gods continued, but that individuals no longer placed their faith or destiny in the gods'hands but rather in "Divine Saviours." People asked two gifts of the Divine Saviours: strength and holiness onEarth and immortality and happiness after death. The author concludes by restating that religion was far fromdefunct in the Hellenistic age.

52. Difficulty = Moderate; Question Type = Detail;The correct answer is choice A. This is a simple detail question that asks Euripides' most favorable

portrayal of the Olympian gods. Euripides is mentioned in paragraph 1. Here, the author states that Euripidesreferred to the gods as "profound psychological forces," "shady seducers," and "figures of fun." The mostcomplimentary of these three is "psychological forces," which is choice A. Thus, choice A is correct.

Choice B says "Divine Saviours." Euripides never referred to the gods in this manner; in fact, DivineSaviours is mentioned in paragraph four in a totally different context. So, choice B is incorrect. Similarly,

Page 18: Explanations to AAMC Practice Test IIIdocshare02.docshare.tips/files/14524/145247660.pdfAAMC MCAT PRACTICE TEST III VERBAL REASONING EXPLANATIONS Passage I (Questions 1-6) This passage

Kaplan Explanations to AAMC Practice Test III Verbal Reasoning

16 Copyright © 2001, Kaplan, Inc.None of this material may be reproduced by any means or incorporated into any information retrieval system, without written permission.

choice C, "inquisitive spirits" is never mentioned by Euripides, so choice C is wrong. Finally, choice D,"figures of fun" is a term associated with Euripides' description of the gods. But, this is certainly not a term ofreverence and is not Euripides' "most favorable portrayal" of the gods. Certainly "psychological forces"(choice A) is a more favorable description. Therefore, choice D is incorrect, and, again, choice A is correct.

53. Difficulty = Moderate; Question Type = Application;The correct answer is choice D. This is an application question that requires you to interpret how a

hypothetical quote from Socrates would support/refute his prosecutors' claim that Socrates did not believe inthe city gods. The hypothetical quote is, "I will obey my god rather than you." The first thing to realize is thatthere is some ambiguity in this quote, as the "god" that Socrates refers to could be any entity or object. If this"god" was an Olympian (city) god, then this quote would refute the accusers, but if the "god" was anythingelse, it would support the prosecutors' claim. Therefore, one can immediately eliminate choice A and choiceB as incorrect because these choices are too restrictive. Choice C is also incorrect. Upon examination, onerealizes that if Socrates referred to an Olympian god in his quote, that would refute the prosecutor's claim, notsubstantiate it, as choice C suggests. Finally, choice D is the best answer. It says this quote would supportthe prosecutor's claim if the "god" was not an Olympian figure. This is true. Therefore, choice D is correct forquestion 53.

54. Difficulty = Moderate; Question Type = Application;The correct answer is choice A. This application question asks how the claim that a Stoic

philosopher originated the portrayal of the Zeus as the god of thunder would effect the author's assertions aboutthe Stoics. The Stoics are mentioned in the second paragraph (lines 18-20). Here the author notes that theStoics reinterpreted many deities as "merely allegorical explanations of natural phenomenon." Therefore, aclaim about a stoic philosopher originating Zeus as the god of thunder would support the author's information.So, one can eliminate choice C and choice D as incorrect, as they both state that Stoic stories about Zeus asgod of thunder would weaken the author's position. Although choice B may seem plausible, the Stoic legendof Zeus as god of thunder does not mean that the Stoics regarded the gods as merely symbolic. Zeus is not asymbol of thunder in this context but rather the god whose activity explains the phenomenon of thunder. Thuschoice B is incorrect. Choice A is a better answer choice and is correct. Choice A states that the legend ofZeus as god of thunder would support the author's assertions because, as already mentioned, the Stoicsexplained natural phenomena by assigning them to gods. Thus, choice A is the credited answer choice.

55. Difficulty = Moderate; Question Type = Implied Detail;The correct answer is choice B. This question requires you to select the answer choice that best

substantiates the claim that religion was a vital element of the Hellenistic world. Choice B claims that theexistence of cults devoted to Divine Saviours supports this claim. The Divine Saviours are mentioned in thefourth paragraph. Here, the author states that people no longer placed their faith in pagan gods, but in theDivine Saviours. Further, the author says that the Divine Saviours were worshipped "passionately" formiraculous gifts of strength on Earth and immortality/happiness after death. In essence, the Divine Savioursmaintained religious faith during the Hellenistic era once pagan beliefs decayed. Thus, choice B is correct, asthe Divine Saviours maintained religion as a vital element of the Hellenistic world.

Choice A is incorrect because the comments St. Paul made about pagan Hellenism (lines 42-45) wereto the effect that God (and therefore religion) was absent from that society. Choice C is equally incorrect, asthe writings of Menander and Epicurus (lines 35-42) claimed that the pagan gods were either non-existent orindifferent to the people. As with choice A, this choice neither satisfies the question stem nor advocatesreligion as vital to this era. Thus, choice C is incorrect. Regarding choice D, the idea that Olympian godswere one human kings was developed by Euhemerus (lines 27-32) as a rationalization of atheism. Obviously,a concept that rationalizes atheism cannot support the idea of religion as a vital part of society. Therefore,choice D is incorrect and choice B is correct for question 55.

56. Difficulty = Easy; Question Type = Detail;The correct answer is choice C. This is a detail question that asks why Menander and Epicurus

agreed that the Olympian gods were indifferent. Menander and Epicurus are mentioned in the third paragraphin lines 38-43. Here it is explicitly stated that Menander and Epicurus claimed that gods were indifferentbecause "the traditional gods seemed able to do nothing to ease the [people's] daily encounters with thevicissitudes of Hellenistic life." This directly corresponds to answer choice C, so, choice C is correct.

Choice A is incorrect because the reference to the gods once being human is used in lines 27-32 as arationalism for atheism, not to justify the gods' indifference. Also, had the gods once been human, it seems

Page 19: Explanations to AAMC Practice Test IIIdocshare02.docshare.tips/files/14524/145247660.pdfAAMC MCAT PRACTICE TEST III VERBAL REASONING EXPLANATIONS Passage I (Questions 1-6) This passage

Kaplan Explanations to AAMC Practice Test III Verbal Reasoning

Copyright © 2001, Kaplan, Inc. 17None of this material may be reproduced by any means or incorporated into any information retrieval system, without written permission.

likely that they would have expressed more interest in human life, not less. Choice B is misguided. TheOlympian gods did not deviate from the "classical cults," as choice B states; rather, people deviated from theclassical (Olympian) cults. Thus, choice B is incorrect. Similarly, choice D is misguided. The gods were notindifferent because they "could not construct an understandable world." This is actually irrelevant. The onlymention of construction of an understandable world comes in lines 33-35 where Strato of Lampsacus claimedthat he could construct an understandable world without the gods. Thus, choice D is incorrect, and choice B iscorrect.

57. Difficulty = Moderate; Question Type = Logic;The correct answer is choice C. This question asks why the Iliad and Homeric Hymns are cited in this

passage. These works are cited in the first paragraph (line 4), where the author discusses how the ancientGreeks occasionally would refer to their gods with a slight irreverence or a "familiar flippancy." Thiscorresponds directly to choice C, making this choice C correct.

The rationalist movements of the fifth century, as mentioned in choice A, are mentioned in the firstparagraph of the passage directly after the reference to the Iliad and Homeric Hymns. Since these movementscame much later in history than the poems did, the poems really can't be cited as evidence for any claimabout the movements. Choice B is tricky, but not correct. This choice implies that the Iliad and HomericHymns were mentioned because the poets believed that gods "were no longer a matter of serious concern."This is not true. The Iliad and Homeric Hymns are mentioned for their flippancy and subtle irreverence. Thisdoes not mean that the gods were unimportant. The key issue with choice B is that irreverence does not meanunimportance. Thus, choice B is incorrect as well. Choice D implies that the Iliad and Homeric Hymns arementioned because people believed the gods to be indifferent to the people. This indifference is mentioned inthe third paragraph, far removed from the reference to the Iliad and Homeric Hymns. Also, the reference toirreverence here was made in a completely different context, so choice D is incorrect, and, again, choice C iscorrect.

58. Difficulty = Moderate; Question Type = Inference;The correct answer is choice D. The question stem asks how poets and authors presented portraits of

Greek gods before the time of the poets Callimachus and Theocritus. These poets are mentioned in the end ofthe second paragraph (lines 20-26). The author says that these poets, like Hellenistic sculptors, began torepresent these gods in a less idealistic way. Specifically, the author says that Callimachus and Theocritusshowed that this was an age where "the old gods were no longer a matter of belief or serious concern. "Therefore, we can deduce that before these individuals were working, the gods were idealized, as stated inchoice D. Thus, choice D is correct.

Choice A says that poets before Callimachus and Theocritus presented portraits of gods "similar tosculpture." Whether sculptures of the gods were similar or dissimilar to poetic portraits prior to Callimachusand Theocritus is never mentioned in the passage. So this conclusion cannot be drawn, and choice A isincorrect. Choice B is incorrect. It says that poets prior to Callimachus and Theocritus depicted the gods as"flippant and unflattering." The passage states that, prior to the early Hellenistic age of Callimachus andTheocritus, poets were flippant toward the gods, but they were not necessarily unflattering. Also, the godswere held in higher esteem prior to the early Hellenistic period, as evidenced by the passage's account ofSocrates death. Thus, choice B is close to the correct answer but not really a good fit. Choice C suggests thatthe gods were described in an atheistic tone by poets prior to Callimachus and Theocritus. This is untrue, andgoes against the grain of the passage, which describes a move toward atheism through and after the earlyHellenistic period. Prior to that period was a time of less atheism, not more, once again as evidenced by theprosecution of Socrates for lack of faith in the Olympian gods. Thus, choice C is incorrect and choice D iscorrect for question 58.

59. Difficulty = Moderate; Question Type = Main Idea;The correct answer is choice B. This question requires one to infer what the passage suggests about

religion as a whole. Choice B is the best answer choice because it represents a theme seen throughout thepassage – religion must "respond in some way to people's needs." For example, pagan Hellenism became a"world without God" (line 45) after the traditional gods seemed able to do nothing to ease people's lives. Inthe fourth paragraph, we learn that people began worshipping the Divine Saviours instead of the Olympiangods when their cults held out the promise of strength and holiness, immortality and happiness--certainlyanswers to people's needs and desires. Choice A is incorrect because the beliefs that eventually gained popular support did not abandon thesearch for "miraculous gifts," but rather, encouraged the search for such gifts through the Divine Saviours.

Page 20: Explanations to AAMC Practice Test IIIdocshare02.docshare.tips/files/14524/145247660.pdfAAMC MCAT PRACTICE TEST III VERBAL REASONING EXPLANATIONS Passage I (Questions 1-6) This passage

Kaplan Explanations to AAMC Practice Test III Verbal Reasoning

18 Copyright © 2001, Kaplan, Inc.None of this material may be reproduced by any means or incorporated into any information retrieval system, without written permission.

Choice C and choice D are too narrow in their perspectives. C proposes that religion must respond to thechallenges set forth by the rationalists in order to remain vital. The rationalists are only mentioned in the firstparagraph as one group who battered the gods; this is too specific to be the entire passage's message aboutreligious vitality. Similarly, choice D addresses only one aspect of the overall argument of the passage.Choice D says that in order to survive, a religion must be attractive to poets and artists. Although the authordoes mention the opinions of poets and artists in the passage, those references are primarily used to illustratesocietal opinions about religion and gods, not to show that the favor of poets and writers are essential for areligion. So choice D is incorrect, and choice B is correct for question 59.

Passage IX (Questions 60-65)

This passage is about television and its unrealistic portrayal of life.

The passage begins by saying that, unlike Hollywood, television is not a dream factory. It is truer to life thaneven life itself, using repetition and immediacy to impact its audience. Originality has little place on thesmall screen; we are numbed by an overdose of news, the ultimate point of which is to keep us glued to ourseats.

The next paragraph charges that television has no language of its own. It recycles ideas and reuses characters.

The third paragraph describes how television has become more "real life" than life, with show times markingdelineations in our days and television becoming a voice of authority and credibility.

The last paragraph characterizes television as a "least common denominator" medium which encourages ourbehavior in life to mimic its language.

60. Difficulty = Moderate; Question Type = Main Idea;The correct answer is choice D. The question asks for the passage's main theme. The premise of the

passage is that television presents a distilled vision of reality, which is consistent with choice D.Choice A, that television relies on series, is a point in the passage, but it is used as evidence for the

main idea, not as the main idea itself. Choice B makes a reference to Hollywood, which is mentioned in thefirst sentence of the passage. However, a comparison between Hollywood films and television is not the maintheme of the passage, and B is incorrect. Choice C makes reference to news reports, which are mentioned inparagraph 1, but - again - this is not key to the author's main point. Thus, choice C is also incorrect.

61. Difficulty = Moderate; Question Type = Inference;The correct answer is choice A. The question asks for an inference based on the passage. The first

paragraph describes television news. Toward the end of the paragraph, the author says: "...the reward of beinga viewer depends on staying passive..." Viewer passivity is necessary for television to maintain its audience.This is consistent with answer choice A.

Choice B is incorrect, as it distorts a point made in the passage. The author doesn't suggest that newsshows aim for emotional release. What the passage says is that audiences remain frustrated even when theyfeel occasional emotional release. Choice C is in conflict with the passage, as the author contends thattelevision aspires to keep viewers in their chairs, not make them leave. Choice D might seem a valid answer -since the passage does say that discontinuous segmentation of news confuses the viewer- but this is not whatnews programs attempt to achieve. Instead, their attempt is to insure themselves ongoing viewership. So,choice is also incorrect.

62. Difficulty = Hard; Question Type = Application;The correct answer is choice D. The passage describes television dialogue as lacking "amateurish or

embarrassing passions", implying that such passions would be unsuitable for television. This is consistent withanswer choice D.

Choice A, the realism of television actors, is not discussed in the passage and is incorrect. Choice Bis incorrect because it is in conflict with the last paragraph, which says television dialogue "lacks precisearticulation". Choice C is incorrect because it makes reference to a typical television drama series – aconcept not addressed in the passage.

Page 21: Explanations to AAMC Practice Test IIIdocshare02.docshare.tips/files/14524/145247660.pdfAAMC MCAT PRACTICE TEST III VERBAL REASONING EXPLANATIONS Passage I (Questions 1-6) This passage

Kaplan Explanations to AAMC Practice Test III Verbal Reasoning

Copyright © 2001, Kaplan, Inc. 19None of this material may be reproduced by any means or incorporated into any information retrieval system, without written permission.

63. Difficulty = Hard; Question Type = Detail;The correct answer is choice A. Early in the passage, the author says, "What has been said before -

'characters' we have seen before, advertisements we 'love' - may well be the evidence that originality.. hasscant future on the box..." Thus, answer choice A is most consistent with the passage – familiarity ofcharacters means little originality on television.

Choice B concerns television news, but this choice is not relevant specifically to the author'sreference to characters and advertisements. So, choice B is wrong. Choice C is incorrect because it is tooindirect; the author never uses overfamiliarity of characters to prove that television is truer to life than life is.Choice D, because it refers to "repetition", might seem to be correct. However, the author does not use thetopics described in the question stem as evidence that television convinces through immediacy and repetition.Instead, the topics are used as evidence for lack of originality. Thus, choice D is also incorrect.

64. Difficulty = Moderate; Question Type = Logic;The correct answer is choice C. Because the question asks which choice would most strongly

challenge the passage, each answer choice must be considered carefully. Choice A - a critic enjoyingtelevision drama - would not seem to have a profound impact on the author's point because the passage doesnot focus significantly on how television is perceived by critics. Thus, A is not correct. Choice B does notchallenge the passage but rather is consistent with its portrayal of television news. So, choice B is alsoincorrect. Choice C - passionate, articulate characters - would be in contrast with the passage. So, C may becorrect, but one must examine choice D still. Choice D, recycling of ideas, is in keeping with the author'sdepiction of television. D does not satisfy the question stem. Therefore, choice C is correct, the only answerchoice which contradicts the passage.

65. Difficulty = Moderate; Question Type = Application;The correct answer is choice B. The portion of the passage quoted in the question stem leads into the

last line of the passage, "...television gives us back our language...docked of...passions or obsessions whichmight cause our audience to switch off." This implies that "good television" is television that viewers keepwatching. This is consistent with answer choice B.

Choice A is in conflict with the passage's description of television: viewers do not feel the full impactof their emotions, as mentioned in the bottom half of paragraph one. Answer choice C also is inconsistent withthe ideas in the passage, as the author cynically views television characters as dispassionate and inarticulate.Choice D is incorrect because it contradicts the passage, which says television programming lacks passion andvariety – certainly not reflective of the unusual and interesting aspects of everyday life.

Page 22: Explanations to AAMC Practice Test IIIdocshare02.docshare.tips/files/14524/145247660.pdfAAMC MCAT PRACTICE TEST III VERBAL REASONING EXPLANATIONS Passage I (Questions 1-6) This passage
Page 23: Explanations to AAMC Practice Test IIIdocshare02.docshare.tips/files/14524/145247660.pdfAAMC MCAT PRACTICE TEST III VERBAL REASONING EXPLANATIONS Passage I (Questions 1-6) This passage

Copyright © 2001, Kaplan, Inc. 21None of this material may be reproduced by any means or incorporated into any information retrieval system, without written permission.

AAMC MCAT PRACTICE TEST III PHYSICAL SCIENCES EXPLANATIONS

Passage I (Questions 66-70)

Passage I describes some reactions of lead. Lead is one of the Group IV elements; you should befamiliar with its chemistry and the types of reaction it will undergo. The passage describes four differentreactions. The quantities of reactant are given for some reactions; therefore, you will probably be given aquestion that tests your understanding of stoichiometry. A good approach to this passage would be to workthrough the reactions, and try to write balanced equations for Reactions 1 through 4. This will help youestablish what is happening in the reactions, and it may also save you time when answering the questions.

66. Difficulty = Easy; Topic = Stoichiometry;The correct answer is choice A. Reaction 4 tells us that PbCO3 —removed by filtration—reacts with

dilute HCl to form a gas. All the answer choices indicate the formation of a gaseous product; CO2 in choicesA and B, Cl2 in choice C, and HI in choice D. You should be aware that when carbonates orhydrogencarbonates are treated with acid, carbon dioxide is evolved. Therefore, the answer choices can benarrowed down to choices A and B. Reaction 4 clearly states that solid PbCO3 and HCl react. Therefore,choice A is the correct answer. Notice that this equation has the correct stoichiometry—one lead carbonatereacts with two hydrochloric acids to form one lead chloride, one carbon dioxide, and one water.

Reaction 3 states that all the lead iodide, PbI2 , is converted to lead carbonate, PbCO3 , and thatPbCO3 is removed by filtration. There is obviously no Na2 CO3 free to react with HCl since Na2 CO3

is used upin Reaction 3, and any excess Na2 CO3 would be removed by filtration; choice B is incorrect. In addition,there is no PbI2 free to react with HCl, so choice D is incorrect. Choice C is incorrect because this reactionwould not occur under the conditions given. For this reaction to occur, Cl– would need to be oxidized by C4+,which is highly unlikely. In addition, we have established that carbon dioxide, not chlorine, is formed in thereaction. Again, the correct answer is choice A.

67. Difficulty = Easy; Topic = Solution Chemistry;The correct answer is choice D. The reaction of Pb2+ with SO4

2– results in the formation of a whiteprecipitate—lead sulfate (you may have encountered this reaction when studying lead-acid batteries). Sincethere is an equal amount of lead nitrate and sodium sulfate in Reaction 1, all the lead nitrate will be convertedto lead sulfate:

Pb(NO3)2 (aq) + Na2 SO4(aq) → PbSO4 (s)+ 2NaNO3 (aq)

Choice A is incorrect since the passage states that all the lead nitrate reacted to form Compound A;therefore, no lead nitrate will remain. Choice B is wrong because, in Reaction 1, there are no iodide ionspresent to react with lead. In addition, you are told that Compound A is a white precipitate, and that PbI2 is ayellow precipitate. Although sodium sulfate is a product of Reaction 1, it is soluble (as are all sodium salts).Since Compound A is insoluble, choice C can be eliminated. Again, the correct answer is choice D.

68. Difficulty = Moderate; Topic = Solution Chemistry;The correct answer is choice A. This question requires you to know how lead hydroxide dissociates in

solution, and how Le Chatelier's principle applies to this equation. Since Pb(OH)2 is only slightly soluble inwater, the following equilibrium will exist:

Pb(OH)2(s) Pb2+(aq) + 2OH–(aq)

Le Chatelier's principle states that an equilibrium subjected to stress will move to alleviate that stress.The pH of water is 7, and the question asks how a pH of 9 would affect the amount of Pb(OH)2 that dissolves.At pH 9, the concentration of hydroxide ions is much greater than at pH 7. How would an increased hydroxideion concentration affect the equilibrium above? According to Le Chatelier's principle, if the hydroxide ionconcentration increased, the system would move to counteract the increase. This can only be done by shiftingthe equilibrium to the left (the excess hydroxide ions are used up). Therefore, an increase in pH results in adecrease in the amount of lead hydroxide that dissolves; choice A is correct.

Choice B is incorrect because a change in pH is associated with a change in hydroxide ionconcentration. Since the concentration of hydroxide ions affects the lead hydroxide equilibrium, there must bea shift in the equilibrium and therefore, a change in the solubility of lead hydroxide . Choice C would only becorrect if the OH– concentration were depleted (i.e., lowering the pH of the solution to less than 7) since thiswould shift the equilibrium to the right, resulting in an increase in the solubility of lead hydroxide. Choice Dis incorrect because a change in pH is associated with a change in OH– ion concentration, which does affectthe equilibrium. Therefore, there is enough information to predict what will happen to the equilibrium, andchoice D is incorrect. Again, the correct answer is choice A.

Page 24: Explanations to AAMC Practice Test IIIdocshare02.docshare.tips/files/14524/145247660.pdfAAMC MCAT PRACTICE TEST III VERBAL REASONING EXPLANATIONS Passage I (Questions 1-6) This passage

Kaplan Explanations to AAMC MCAT Practice Test III Physical Sciences

22 Copyright © 2001, Kaplan, Inc.None of this material may be reproduced by any means or incorporated into any information retrieval system, without written permission.

69. Difficulty = Difficult; Topic = Solution Chemistry;The correct answer is choice B. Since you are not given any information about the solubility products

of lead compounds, you have to obtain the answer by looking at the reactions. When lead iodide reacts withsodium carbonate (Reaction 3), all the lead iodide is converted to lead carbonate. Therefore, lead wouldrather form a precipitate with carbonate anions than with iodide anions. Hence, if Pb2 + were added to asolution containing equal amounts of CO3

2– and I– , PbCO3 would precipitate first; PbI2 would precipitatesecond (only after most of the CO3

2– ions were used up). Choice D can be eliminated.Choice A is incorrect since Reaction 2 states that when Compound A (lead sulfate) is mixed with a

solution of KI(aq), lead iodide is formed. Therefore, Pb2+ would form a precipitate with I– before it would forma precipitate with SO4

2–. So far, we have established that in Reaction 2, lead would rather form a precipitatewith I– than with SO4

2–, and that in Reaction 3, lead would rather form a precipitate with CO32– than with I– . It

follows, therefore, that the order of precipitation of the anions with lead is CO32– > I– > SO4

2–. Choice C isincorrect since, from the order of precipitation, lead would rather form a precipitate with CO3

2– than with SO42–

. Again, the correct answer is choice B.

70. Difficulty = Difficult; Topic = Stoichiometry;The correct answer is choice B. We are told in Reaction 1 that 15 mL of 0.3 M Na2 SO4 is used;

therefore, there are 0.3 × 0.015, or 4.5 × 10–3 moles of Na2 SO4 in solution. However, Na2 SO4 dissociates as

follows:

Na2SO4(aq) → 2Na+(aq) + SO42–(aq)

For every mole of sodium sulfate that dissolves, two moles of sodium ions form, so the actual numberof moles of Na2+ is 2 × (4.5 × 10–3), or 0.009—choice B.

Passage II (Questions 71–75)

Passage II is very short, but still contains a lot of important information. It defines terms such ascritical point and supercritical fluid, and states that some of the properties of supercritical fluids are similar tothe properties of liquids. It then presents two graphs: a pressure versus temperature phase diagram for CO2 ,(which you are probably familiar with), and a more complex pressure versus volume phase diagram for CO2.

To understand what is happening in this passage, you need to understand the implication of criticalpoints. Above the critical temperature, a substance cannot be compressed into a liquid at any pressure, and atthe critical temperature, the minimum pressure required to liquefy the substance is called the critical pressure.The critical point of a substance, as the passage states, occurs under specific conditions of temperature andpressure.

71. Difficulty = Moderate; Topic = Thermodynamics and Thermochemistry;The correct answer is choice D. To answer this question, you need to be able to read the pressure

versus volume diagram provided in Figure 2. The passage states that at the critical point, the densities ofliquid CO2 and gaseous CO2 become equal. Since the mass is constant, it follows that the density of the gasand liquid will only be the same at an equal volume. D is the only point where the liquid and vapor curvesintersect, and the volume of liquid and gas is equivalent, so choice D is the correct answer.

Choices A, B, and C do not meet the requirements stipulated for critical point. Although choice Alies on an isothermal curve, it is a random point above the critical pressure. (Remember, the passage statesthat the critical point occurs at "specific conditions of temperature and pressure.") Points B and C lie on theboundary of the region where the liquid and gas are in equilibrium. At the critical point, however, the gas andliquid exist in a single phase, not in an equilibrium, so choices B and C can be eliminated.

72. Difficulty = Moderate; Topic = Solution Chemistry;The correct answer is choice D. The passage states that supercritical fluids are just as effective as

liquids under normal conditions in their ability to dissolve other substances. Therefore, we can use eithersupercritical CO2 or a nonpolar liquid to extract an organic oil. In liquid-liquid extractions, the solvent musteventually be separated from the solute. The efficiency of this separation drastically improves if the solventreadily forms a gas, and evaporates from the solvent-solute mixture. Supercritical CO2 will evaporate muchmore easily (thus the extraction will be more efficient) than a liquid under normal conditions, so choice D isthe correct response.

Choice A is incorrect since CO2 is a relatively unreactive molecule. Even if this were true, theefficiency of the extraction would decrease since we are trying to separate, not react, the solute and thesolvent. Choice B is incorrect since critical point conditions—shown in Figure 1—occur slightly above roomtemperature and at very high pressure; under these conditions, CO2 would not be considered 'easily handled.'Choice C is false because CO2 does not crystallize easily. Even if it CO2 did crystallize easily, this would be

Page 25: Explanations to AAMC Practice Test IIIdocshare02.docshare.tips/files/14524/145247660.pdfAAMC MCAT PRACTICE TEST III VERBAL REASONING EXPLANATIONS Passage I (Questions 1-6) This passage

Kaplan Explanations to AAMC MCAT Practice Test III Physical Sciences

Copyright © 2001, Kaplan, Inc. 23None of this material may be reproduced by any means or incorporated into any information retrieval system, without written permission.

disadvantageous in the extraction. As Figure 1 demonstrates, supercritical CO2 exists at temperatures andpressures above the critical point; there is no solid phase possible under these conditions. Again, the correctanswer is choice D.

73. Difficulty = Moderate; Topic = Electronic Structure and the Periodic Table;The correct response is choice D. Water exists as a liquid at room temperature and atmospheric

pressure since there are strong intermolecular attractions—namely, hydrogen bonding and dipole-dipoleinteractions—between the water molecules. Due to its linear geometry and the pull of electron density inopposite directions by the oxygens, carbon dioxide has a net dipole moment of zero and, therefore, showsnonpolar character; as a result, carbon dioxide molecules will not undergo dipole-dipole interactions. Inaddition, carbon dioxide does not possess any hydrogens, so it cannot undergo hydrogen bonding. The onlyintermolecular forces that exist in CO2 molecules are weak dispersion forces or London forces. Therefore,choice D best accounts for the observation that CO2 cannot exist as a liquid at room temperature unlesspressurized.

Choices A and C are wrong because CO2 is nonpolar. Choice B is incorrect (as is choice A) becausewe have just established that CO2 only has weak intermolecular forces. Again, the correct answer is choice D.

74. Difficulty = Moderate; Topic = Phases and Phase Equilibria;The correct answer is choice C. Remember, the critical point is the point at which carbon dioxide

coexists as a gas and liquid; that is, the point at which liquid CO2 and gaseous CO2 cease to exist as distinctphases. In Figure 1, we can see that this point corresponds to a temperature of just below 40°C, and around 80atmospheres pressure. Since choice C is the only set of coordinates near these values, it is the correct answer.

Choice A is incorrect since these values correspond to the triple point of CO2 ; that is the point atwhich solid CO2 , liquid CO2 , and gaseous CO2 all coexist in equilibrium. Choice B is incorrect since thesecoordinates correspond to the solid-liquid phase boundary. Finally, choice D is incorrect since theseconditions correspond to room temperature and pressure, not critical temperature and pressure.

75. Difficulty = Moderate; Topic = Solution Chemistry;The correct answer is choice B. Applying the principle "like dissolves like" we should expect CO2 —a

nonpolar, covalent molecule—to be a good solvent for nonpolar, covalent solutes. Diethyl ether—choice B—is the only molecule that is nonpolar and covalent, so choice B is the correct response. Choices A, B and Care all ionic compounds and would only be slightly soluble, if at all, in supercritical CO2 . All of thesecompounds would be most soluble in polar solvents such as water.

Passage III (Questions 76–81)

This Research Study passage covers the production of X-rays. The first three paragraphs provide aconcise but straightforward explanation of both continuous spectra and line spectra. These definitions will bevery important when answering the questions. The fourth paragraph introduces a schematic diagram (Figure 1)of an experimental apparatus designed to produce X-rays. The fifth paragraph explains briefly how thecomponents of the apparatus operate. Essentially, electrons are accelerated towards an anode made of someheavy element. When they collide with the heavy atoms, X-rays are produced. The last paragraph explainsthe results of the experiment which are presented in the form of a graph (Figure 2) of X-ray intensity versuswavelength. Notice the two sharp emission peaks P1 and P2 which are examples of line spectra.

76. Difficulty = Easy; Topic = Atomic and Nuclear Structure;The correct answer is choice C, emission of X-ray photons as a result of electronic transitions in

atoms. First, you must realize that the emission peaks represent line spectra. This can be inferred from theinformation in paragraph three. According to paragraph three, a line spectrum results when an electron makesa transition from a higher energy level to a lower one and a photon is emitted with energy equal to thedifference in electron energies. Remember that the wavelength of a photon λ is related to its energy E by E =hc / λ , where h is Planck’s constant and c is the speed of light in a vacuum. It follows that a specific photonenergy corresponds directly to a specific photon wavelength, and the wavelength of an emitted photoncorresponds to the energy difference between electron energy levels. From this you can deduce that linespectra occur at specific wavelengths and that peaks P1 and P2 represent line spectra. Since the emissionpeaks represent line spectra and paragraph three states that line spectra result from electron transitions, choiceC is correct. Choice A can’t be right because bremsstrahlung produces continuous spectra (see the secondparagraph). Choice B is wrong because the question stem states that the peaks are emission peaks, notabsorption peaks. Finally, choice D is wrong because the acceleration of electrons in a magnetic field hasnothing to do with line spectra. Once again, choice C is correct.

Page 26: Explanations to AAMC Practice Test IIIdocshare02.docshare.tips/files/14524/145247660.pdfAAMC MCAT PRACTICE TEST III VERBAL REASONING EXPLANATIONS Passage I (Questions 1-6) This passage

Kaplan Explanations to AAMC MCAT Practice Test III Physical Sciences

24 Copyright © 2001, Kaplan, Inc.None of this material may be reproduced by any means or incorporated into any information retrieval system, without written permission.

77. Difficulty = Easy; Topic = Electric Circuits;The correct answer is choice A, 5 × 102 W. This question tests your knowledge of power dissipation

in an electric circuit. Power is dissipated as a current passes through a resistor. In the circuit shown in Figure1, there are no obvious resistors. However, as electrons travel through the gas in the tube, they experienceresistance due to collisions with the gas molecules. These collisions cause the electrons to lose energy. Thisenergy loss is indicated by existence of a potential drop between the anode and the cathode. Now that thetube has been identified as a resistor, a simple equation will allow you to compute the power consumed by thetube. Power equals voltage times current. Therefore, a voltage drop of 105 V and a current of 0.005 A resultsin a power of (105 )(0.005) W. The multiplication is most easily done using scientific notation, and the resultis (105 )(5 × 10–3) W = 5 × 102 W. Therefore, choice A is correct.

78. Difficulty = Moderate; Topic = Atomic and Nuclear Structure;The correct answer is choice C, 1.16 × 10–14 J. To answer this question, you must remember that an

X-ray is emitted when an electron in an atom makes a transition between two energy levels and the energy ofthe emitted X-ray is equal to the difference in the electron energy levels. This is explained in the thirdparagraph of the passage. Therefore, when an electron in the n = 2 level makes a transition to the n = 1 level,the energy of the emitted X-ray photon equals the energy of the n = 2 level minus the energy of the n = 1level. Don’t be confused by the fact that the electron energies are given as ionization potentials. Theionization potential is the energy required to ionize the electron. Therefore, an electron’s energy equals thenegative of its ionization potential. So the energy of the n = 2 state equals –240 × 10–17 J, and the energy ofthe n = 1 state equals –1,400 × 10–17 J. The energy difference, which equals the energy of the emitted X-rayphoton, therefore equals –240 × 10–17 – (–1,400 × 10–17) = 1,160 × 10–17 J. To get the answer in the form ofthe answer choices, you must use scientific notation to rewrite it as 1.16 × 10–14 J, which is choice C.

79. Difficulty = Moderate; Topic = Electrostatics and Electromagnetism;The correct answer is choice A, He. The only outside knowledge you need to answer this question is

the definition of an ion. The second paragraph of the passage explains that bremsstrahlung occurs when freeelectrons collide with ions. Well, He is a neutral atom and therefore not an ion, so choice A is correct. All ofthe other choices are ions because they have a net charge.

80. Difficulty = Moderate; Topic = Work and Energy;The correct answer is choice A, the voltage of HV was increased. This question requires reading

comprehension and a basic understanding of Newtonian dynamics. Since the answer choices all contain twoparts, begin with the first part. The function of the LV and HV voltage sources is described in the fifthparagraph of the passage. The LV regulates the filament temperature, the electrical current in the tube, andthe number of X-rays produced at the anode. The HV regulates electron acceleration. Well, intuition dictatesthat the maximum kinetic energy of electrons colliding with the anode is related to their acceleration.Therefore, you can eliminate choice C and choice D, since adjusting the LV power supply does not affect theacceleration of electrons colliding with the anode. To decide between choice A and choice B, consider anelectron that starts from rest at the cathode and is accelerated by a potential difference toward the anode. Atthe cathode, the electron has a potential energy equal to its charge times the potential difference between thecathode and anode. All of this potential energy is then converted to kinetic energy as the electron acceleratestowards the anode. So, a larger potential energy at the cathode results in a larger kinetic energy at the anode.Since the electron charge is constant, the only way to increase an electron’s potential energy at the cathode isto increase the potential difference between the cathode and the anode. This potential difference equals thevoltage of HV. Therefore, increasing the voltage of HV increases the maximum kinetic energy of the electron,and choice A is correct.

81. Difficulty = Moderate; Topic = Light and Geometrical Optics;The correct answer is choice D, P2 , because the peak has the higher intensity. This question looks

difficult at first, but it really only requires a little reasoning. First of all, you must understand what eventscaused the peaks P1 and P2 . As mentioned in the explanation to question 76, these peaks represent the linespectra of the emitted X-rays. In the last paragraph of the passage, it says that the intensity of the emitted X-rays is proportional to the number of photons emitted at wavelength λ . Therefore, the events in question arethe emission of X-ray photons. Since an event with a higher probability will occur more often, the higherprobability event will be the one corresponding to the emission of a greater number of photons. Therefore, thepeak with the higher intensity is the result of the more probable event. Looking at Figure 2, you should findthat P2 has the higher intensity, so choice D is correct.

Page 27: Explanations to AAMC Practice Test IIIdocshare02.docshare.tips/files/14524/145247660.pdfAAMC MCAT PRACTICE TEST III VERBAL REASONING EXPLANATIONS Passage I (Questions 1-6) This passage

Kaplan Explanations to AAMC MCAT Practice Test III Physical Sciences

Copyright © 2001, Kaplan, Inc. 25None of this material may be reproduced by any means or incorporated into any information retrieval system, without written permission.

Passage IV (Questions 82–87)

This is a fairly straightforward passage about dipole moments in chemical bonds, which discusses how dipolemoments are calculated and how the geometry of molecules can be determined by the presence of a dipole moment.The first paragraph reviews the idea of polar covalent bonds. The second paragraph then introduces the idea that thecharge separation induced by a polar covalent bond will result in a measurable dipole moment. We are also given theequation to calculate the dipole moment and the unit used to measure dipole moments. As with all equations on theMCAT, look at the equation carefully and make sure that you understand all the terms involved, but don’t take anytime to memorize it. Chances are that you will be asked to manipulate this equation.

The last paragraph discusses the idea that the geometry of a molecule can be predicted by knowing its netdipole moment. The net dipole moment is the sum of all the individual bond dipole moments in the molecule, andtherefore, will be affected by the direction of the bonds in three-dimensional space, as well as the magnitude of theirparticular dipole moments (determined by the polarity of the bond). The table gives the net dipole moments for variousmolecules. Since you are given the dipole moments for various compounds, be prepared to predict the geometry ofsome of these compounds.

82. Difficulty = Easy; Topic = Electronic Structure and the Periodic Table;The correct answer is choice D. This question asks about the observed dipole moment of SnBr4 , which,

according to Table 1, is zero. Looking at the periodic table, you can see that there will be an electronegativitydifference between tin and bromine since tin is located to the left of bromine. Therefore, each individual covalentbond between tin and bromine will be polar since electron density is pulled towards bromine. If the molecule possessesa net dipole moment of zero, the SnBr4 molecule must be symmetrical, so that the individual bond moments canceleach other out: choice D. Choice A is incorrect since Br has a greater effective nuclear charge than Sn; that is, thepull of electrons toward the nucleus is greater in Br, which accounts for its greater electronegativity in the chemicalbond. Choice B is wrong since tin and bromine are not the same size—the tin atom will be larger since it appearslower down in the table. Also, the net dipole moment of the molecule is not affected by the size of the individualatoms, just by their arrangement in space. Choice C is incorrect since the electronegativity of tin predicts that theelectrons will be weakly attracted to it, not more strongly attracted. Again, the correct answer is choice D.

83. Difficulty = Moderate; Topic = Electronic Structure and the Periodic Table;The correct answer is choice D. The electrons in the sigma bond are shared between both atoms, so the

electron cloud extends all the way from one atom to another to form a bonding orbital. Choice A—which depicts anantibonding orbital—is, therefore, incorrect. There is a difference in electronegativity between nitrogen and oxygen:oxygen is the more electronegative atom since it appears farther to the right in the periodic table. Hence, the electronswill be more attracted to the oxygen atom, and the electron cloud around the oxygen atom will be larger. This isdepicted in choice D.

Since the atoms have different relative attractions for electrons, the electrons in the sigma bond will not beshared equally, and choice B is incorrect. Choice C is incorrect because it depicts a larger electron cloud aroundnitrogen. This suggests that nitrogen has a higher electronegativity than oxygen, which is incorrect. Again, the correctresponse is choice D.

84. Difficulty = Moderate; Topic = Acids and Bases;The correct answer is choice B. This question asks us to predict the observed dipole moment for HCl. Since

we are not given the separation distance, we do not have to calculate the value of the dipole moment exactly—we justhave to estimate, based on the value of the observed dipole moment for HF (given in Table 1). The answer choicesreinforce the lack of calculation since the values are relatively far apart.

The observed dipole moment for HF is 1.82 debyes. Looking at the periodic table, we can predict that the HClbond will be less polar than the HF bond since Cl– is less electronegative than F– . Therefore, the dipole moment ofHCl will be slightly lower than for HF: choice B.

Although the dipole moment in choice A is lower than that of HF, this dipole moment would be characteristicof a bond that is not particularly polarized. Since there is a large electronegativity difference between hydrogen andchlorine, the dipole moment will be much larger than 0.02. Choices C and D are incorrect since these values are largerthan the dipole moment of HF, and we have already established that HCl will have a lower dipole moment since thebond is less polarized.

85. Difficulty = Easy; Topic = Electronic Structure and the Periodic Table;The correct response is choice D. Understanding why this choice is correct requires a familiarity with the

concept of effective nuclear charge. Remember that electrons are more strongly attracted to atoms with greaterelectronegativities (chlorine in this particular example). A higher electronegativity means that outer bonding electronsfeel a stronger pull towards the nucleus, so chlorine is considered to have a greater effective nuclear charge thanhydrogen, and choice D is correct.

Since we have established that chlorine is more electronegative than hydrogen, choices B and C can beeliminated. Choice A is a true statement in that hydrogen atoms are smaller than chlorine atoms, but is incorrect since

Page 28: Explanations to AAMC Practice Test IIIdocshare02.docshare.tips/files/14524/145247660.pdfAAMC MCAT PRACTICE TEST III VERBAL REASONING EXPLANATIONS Passage I (Questions 1-6) This passage

Kaplan Explanations to AAMC MCAT Practice Test III Physical Sciences

26 Copyright © 2001, Kaplan, Inc.None of this material may be reproduced by any means or incorporated into any information retrieval system, without written permission.

it has nothing to do with why the HCl bond is a polar covalent. (This is a common type of wrong answer choice seenon the MCAT: it is a true statement, but does not answer the question you are being asked.)

86. Difficulty = Moderate; Topic = Electronic Structure and the Periodic Table;The correct answer is choice D. We are asked here what will happen to the dipole moment if we remove an

oxygen from carbon dioxide to form carbon monoxide. Let's start by determining the dipole moment of carbon dioxide.The individual bonds in carbon dioxide are polar: there is enough of an electronegativity difference between carbonand oxygen so that, in the C=O bond, the carbon will acquire a slight positive charge and the oxygen will acquire aslight negative charge. Recall, however, that the geometry of a molecule also influences the net dipole moment in themolecule. Since the CO2 molecule contains a carbon with two regions of electron density, the geometry of CO2 islinear:

C OO180°

δ–δ+δ–

This molecule is symmetrical, and the two individual bond dipoles are equal in magnitude but opposite indirection; therefore, the observed dipole moment for carbon dioxide is zero.

In carbon monoxide, there is still the same electronegativity difference between carbon and oxygen, but thereis only one carbon-oxygen bond. Therefore, one end of this molecule will assume a partial positive charge (thecarbon), and the other end will assume a partial negative charge (the oxygen). Thus, there is a dipole moment in thismolecule. The only choice in which the dipole moment increases when oxygen is removed from CO2 is choice D, thecorrect response.

87. Difficulty = Moderate; Topic = Electronic Structure and the Periodic Table;The correct answer is choice B. Assessing the dipole moment of the two molecules shown involves making

decisions similar to those we made when we answered question 86. The first decision is whether the individualbonds—P-Cl bonds in this case—will be polar. By checking the periodic table, we can see that chlorine is moreelectronegative than phosphorus, so the P-Cl bonds will be polar. Next, we must consider the overall geometry of themolecule to determine the sum of the individual bond dipole moments. Looking at the figures which have been drawnfor us, we see that PCl5 (trigonal bipyramidal geometry) will have a net dipole moment of zero, since the individualdipole moments in the vertically oriented bonds cancel out, and the individual dipole moments in the horizontallyoriented bonds cancel out.

Unlike PCl5 , PCl3 has a total of four "substituents" attached to the central phosphorus atom—the three chlorineatoms and a lone pair of electrons. Since one of the substituents is a lone electron pair, it will repel the bondingelectrons, hence PCl3 has a pyramidal geometry as opposed to a trigonal planar geometry (choice A is incorrect). ThePCl3 molecule will, therefore, possess a net dipole moment, and choice B is the correct response:

PCl

ClCl

δ+δ–

δ–δ–Dipole

moment

Choices C and D are incorrect because we have established that PCl5 , because of the pull of electrons by Clin equal but opposite directions, will not possess a net dipole moment. In addition, the geometry of PCl5 is trigonalbipyramidal, not octahedral as stated in choice C. Again, the correct answer is choice B.

Discretes

88. Difficulty = Easy; Topic = Work and Energy;The correct answer is choice C, 200 J. To answer this question, you have to remember that work

equals the component of force acting along the direction of motion times the displacement in that direction. Inthis case the entire force of 20 N acts along the direction of motion, and the displacement is 10 m. Therefore,you don’t have to worry about resolving the force into components, and the work equals (20 N)(10 m) = 200 J.So choice C is correct.

89. Difficulty = Easy; Topic = Thermodynamics and Thermochemistry;The correct answer is choice B—kinetic energy. In the liquid state, molecules do not possess a lot of

translational kinetic energy because there are strong intermolecular forces between the molecules. To vaporize, a

Page 29: Explanations to AAMC Practice Test IIIdocshare02.docshare.tips/files/14524/145247660.pdfAAMC MCAT PRACTICE TEST III VERBAL REASONING EXPLANATIONS Passage I (Questions 1-6) This passage

Kaplan Explanations to AAMC MCAT Practice Test III Physical Sciences

Copyright © 2001, Kaplan, Inc. 27None of this material may be reproduced by any means or incorporated into any information retrieval system, without written permission.

liquid molecule must attain a certain "escape velocity" sufficient to overcome these intermolecular attractions. Sinceenergy due to motion, or velocity, is kinetic energy, choice B is the correct response.

Although some molecules can exist as resonance hybrids, and the contributing resonance structures may affectthe boiling point, resonance is not the reason evaporation occurs; therefore, choice A is incorrect. Surface tension is ameasure of the attraction between molecules on the surface of a liquid and molecules in the interior of the liquid. Thelarger the surface tension, the more difficult it is for a molecule to leave the liquid phase. Therefore, attaining asufficient amount of surface tension would actually prevent the liquid from evaporating, thereby eliminating choice C.In a liquid, it is the position of a molecule with respect to the other molecules, and the attractive forces thesemolecules experience, that determines potential energy. However, it is the kinetic energy attained by the molecule,not the potential energy, that ultimately determines whether it will escape from a liquid, so choice D is incorrect.

90. Difficulty = Moderate; Topic = Translational Motion;The correct answer is choice C, 2.0 sec. To answer this question, you must remember (or be able to

derive) the equation for objects that are accelerated uniformly in one dimension. It is x = v0 t + (1/2)at2 , wherex is the displacement at time t, v0 is the initial velocity, and a is the acceleration. In this case, the runnerstarts from rest, so you can set v0 = 0 and the equation reduces to x = (1/2)at2 . Solving for t gives t = 2x / a .Plugging in the numbers, you find t = (2)(3) / (1.5) = 2.0 sec , so choice C is correct.

91. Difficulty = Moderate; Topic = Equilibrium and Momentum;The correct answer is choice C, F = 5 N. This question has a somewhat confusing setup, but don’t

complicate the situation. Since it only asks about translational equilibrium and not rotational equilibrium, youcan treat all of the forces as if they are acting at the center of mass of the rectangle. Therefore, consider thefollowing force diagram.

θ

F

4 N

3 N

In order for the rectangle to be in translational equilibrium, the vector sum of all the forces must be zero.Therefore, the horizontal component of F must equal 4 N, and the vertical component of F must equal 3 N.All you need to do is find the magnitude F, which is the hypotenuse of the right triangle containing θ. Sincethe legs must be 4 and 3, respectively, use the Pythagorean theorem to find that

F = 32 + 42 = 5 N,

and choice C is correct.

Passage V (Questions 92–97)

Passage V is a problem-solving passage. Two experiments are carried out to investigate the melting andfreezing behaviors of the organic compound acetamide. Read the experimental procedure carefully—you will probablybe asked questions based on you understanding of the procedures. Also, try to look at Figure 1 whenever applicable;this will help you relate the experiment to the results and, hopefully, draw conclusions from the results. Remember,the MCAT is not just a science test—it tests your ability to interpret information (textual and graphical in thisparticular passage), and draw conclusions based on that information.

92. Difficulty = Easy; Topic = Thermodynamics and Thermochemistry;The correct answer is choice B. To answer this question, you need to identify the key difference in Trial 1 and

Trial 2; in trial 1, the test tube was left to cool in air at 20°C, whereas in trial 2, the test tube was inserted into abeaker of water at 20°C. Therefore, the difference in the procedures is the medium in which acetamide is cooled, andchoice B is the correct answer.

The passage states that the same test tube was used for Trial 1 and Trial 2. No acetamide—other than theinitial 10 g—was added to, or removed from the test tube, so choice A is incorrect. Choice C is incorrect because both

Page 30: Explanations to AAMC Practice Test IIIdocshare02.docshare.tips/files/14524/145247660.pdfAAMC MCAT PRACTICE TEST III VERBAL REASONING EXPLANATIONS Passage I (Questions 1-6) This passage

Kaplan Explanations to AAMC MCAT Practice Test III Physical Sciences

28 Copyright © 2001, Kaplan, Inc.None of this material may be reproduced by any means or incorporated into any information retrieval system, without written permission.

trials were initiated above the melting point of acetamide. In Experiment 1, the test tube was lowered into a beaker ofboiling water while the temperature of acetamide in the test tube was monitored. After Experiment 1 was performed,the test tube was immediately removed from the beaker of boiling water, and Trial 1 was carried out; in Trial 2, thetest tube was initially placed in boiling water. Since we are studying the freezing behavior and not the meltingbehavior of acetamide in Trials 1 and 2, the length of time allowed for acetamide to melt is not an issue: choice D isincorrect. Again, the correct answer is choice B.

93. Difficulty = Easy; Topic = Phases and Phase Equilibria;The correct answer is choice D. Question 93 is very similar to question 92 in that it tests your understanding

of the experimental procedure. Choice D is correct because water molecules at a temperature of 90°C have lesskinetic energy than water molecules at a temperature of 100°C. As a result, it will take longer for the water moleculesto transfer the same amount of kinetic energy to the test tube, and it will take longer for acetamide to melt.

Choices A and B are incorrect because, as long as the test tube is completely immersed in the beakerthroughout the experiment, the addition or loss of water at the same temperature will not affect how much acetamide isin contact with the water, and, therefore, the melting behavior of acetamide. Choice C is incorrect because,eventually, the temperature of acetamide in the test tube will equilibrate to the same temperature as the surroundingwater—90°C. The passage clearly states that acetamide melts at 80°C, so choice C can be eliminated. Again thecorrect response is choice D.

94. Difficulty = Easy; Topic = Kinetics and Equilibrium;The correct answer is choice A. Experiments 1 and 2 are being carried out under controlled conditions and are

well monitored: the chemist uses the same sample for all the experiments; where applicable, the initial and finaltemperatures of acetamide are noted; and the temperature of acetamide is recorded at regular intervals, which givesvalid data. Therefore, choice A is correct and choice B is incorrect.

Choices C and D are incorrect since the amount of acetamide is never varied throughout Experiments 1 and 2.Therefore, the experiment is not at all useful in investigating how the amount of acetamide controls the melting andfreezing temperature (choice C), or the time needed for melting and freezing (choice D).

95. Difficulty = Easy; Topic = Phases and Phase Equilibria;The correct answer is choice B. In Trials 1 and 2, the chemist takes temperature readings at thirty second

intervals. The passage states that acetamide was completely frozen after 23 minutes, or when the 46th reading wastaken by the chemist. If temperature readings were taken at 1 minute intervals, the solid would still freeze after 23minutes; the only factor that would differ is that only 23 readings would have been taken by the chemist. Since thetime taken for acetamide to freeze does not change, choice B is the correct response.

Choices A and C are wrong because we have just established that the time taken for acetamide to freeze doesnot change, only the intervals at which the readings are taken change. Choice D is incorrect because the intervals atwhich measurements are taken have no influence on the freezing temperature. Again, the correct answer is choice B.

96. Difficulty = Moderate; Topic = Phases and Phase Equilibria;The correct answer is choice D. The purpose of Trial 1 and Trial 2 is to investigate the freezing behavior of

acetamide; that is, when acetamide forms a solid. Therefore, acetamide must be in its liquid form so that the point atwhich it solidifies, i.e., freezes, can be identified. In Trial 1, the test tube containing acetamide is removed from thehot water, so it is already in its liquid form. However, in Trial 2, the test tube containing acetamide is used after Trial1—when acetamide was frozen. In order to investigate the freezing properties, the test tube was placed in hot water,so that acetamide could melt. Choice D is the correct answer.

Choice B is incorrect because it describes a different stage of the experiment. The fact that acetamide wascooled in air in Trial 1, and cooled in water in Trial 2 has nothing to do with the initial immersion of the test tube intoboiling water. Choice A is incorrect since it states that the water for Trial 2 needed to be cold. Again, this is also adifferent stage in the experiment where the acetamide is cooled, not melted. Choice C can be eliminated since this isa consequence of cooling the test tube in cold water, not a result of placing the test tube in hot water as the questionstem states. Again, the correct answer is answer D.

97. Difficulty = Moderate; Topic = Thermodynamics and Thermochemistry;The correct response is choice A. Since acetamide was cooled in air in Trial 1, the cooling rate would be

slower as compared to cooling in water in Trial 2. Therefore, if the rate of cooling is slower in air, the temperature willdrop over a greater time, i.e., the downward slope will be less steep. In Trial 2, the sample takes 23 minutes to freeze,and the graph in Figure 1 only presents data collected over a period of 4.5 minutes. Therefore there is no way that allthe data for Trial 1 could be shown, and choice A is the correct response.

Page 31: Explanations to AAMC Practice Test IIIdocshare02.docshare.tips/files/14524/145247660.pdfAAMC MCAT PRACTICE TEST III VERBAL REASONING EXPLANATIONS Passage I (Questions 1-6) This passage

Kaplan Explanations to AAMC MCAT Practice Test III Physical Sciences

Copyright © 2001, Kaplan, Inc. 29None of this material may be reproduced by any means or incorporated into any information retrieval system, without written permission.

Choice B is incorrect since we have established that the graph will slope less steeply downward, and not allthe data will be shown. Choices C and D are incorrect because they state that the graph will slope upward. An upwardslope indicates an increase in temperature of acetamide, but Trials 1 and 2 deal with the cooling of acetamide. Again,the correct answer is question A.

Passage VI (Questions 98–103)

This Information Presentation passage is concerned with the physics of musical tones. You’re toldthat the timbre, or quality, of a musical tone depends on the way in which the various harmonics, including thefundamental, are mixed together. Since most of the information in this passage is presented in the form ofgraphs, you can expect the majority of the questions to require graphical interpretation and graphical analysis.From a quick glance at the graphs given, it appears that the concepts that might apply here are displacement,amplitude, frequency, period, phase, and the concept of different harmonics. You usually learn the subject ofharmonics by studying the standing waves that can be produced in cylindrical pipes that are either open atboth ends or open at one end and closed at the other end. However, a string or anything else capable ofsupporting standing waves can also produce harmonics.

98. Difficulty = Moderate; Topic = Wave Characteristics and Periodic Motion;The correct answer is choice C, third harmonic. To answer this question, refer to Figs. 1a, 1b, and 1c.

The passage states that the three waveforms in Figure 1a are the first three harmonics of a tone. You shouldremember that for a pipe or a string or anything capable of supporting a standing wave, the standing wave withthe lowest frequency and therefore the longest wavelength is called the fundamental or the first harmonic. Astanding wave with twice the frequency of the first harmonic is called the second harmonic, a standing wavewith three times the frequency of the first harmonic is called the third harmonic, and so on for higherharmonics. This relationship is explicit in the resonance conditions for a pipe open at one or both ends. Forexample, for a pipe open at both ends the frequency of the nth harmonic is given by f = nv/2L, where v is thespeed of sound and L is the length of the pipe. Remember n = 1 corresponds to the fundamental or firstharmonic. Of the three waves in Figure 1a, clearly the one represented by the short dashes has the highestfrequency, because it repeats itself in the shortest amount of time. The waveform represented by short dashesmust be the third harmonic because it has the highest frequency. Since the period of a wave is equal to oneover its frequency, the third harmonic will have a shorter period than the first and second harmonics, makingchoices A and B incorrect. Comparing Figures 1a and 1c, you can see that the period of the wave representedin Figure 1c appears to be equal to the period of the first harmonic. Since the period of the first harmonic islonger than the period of the third harmonic, choice C is the correct answer.

99. Difficulty = Moderate; Topic = Wave Characteristics and Periodic Motion;The correct answer is choice C, at zero displacement. Looking at Figure 1a, notice that the three

curves intersect at the origin and then at a second position which is approximately halfway along the timeaxis. It’s this point that you’re being asked about. Since this is an open-ended question, you have to lookthrough the answer choices to find the best answer. Choice A states that at this point, all three curves are inphase. Notice that at the point in question, the fundamental has gone through half a period, the secondharmonic has gone through a full period, and the third harmonic has gone through one and a half periods.Since the first and third harmonics differ in phase by one full period, they are in phase. The second harmonic,however, is out of phase by half of a period from the other two, and thus we can eliminate choice A. Choice Bis incorrect because it states that all three waves are out of phase and we just determined that the fundamentaland the third harmonic are in phase. Choice C states that all three waves have zero displacement at the pointin question. By looking at the graph, you can determine this is true, so choice C is the correct answer. Notethat choice D is wrong because it directly contradicts choice C.

100. Difficulty = Difficult; Topic = Wave Characteristics and Periodic Motion;The correct answer is choice A, 0.005 sec. Although the question refers to Figure 2a, you don’t need

any information from that figure to answer the question. To answer this question, you have to find thefrequency of the second harmonic and then use the fact that period equals one over frequency to determine theperiod of the second harmonic. As discussed in the explanation to question 98, the frequency of the secondharmonic is twice the frequency of the fundamental or first harmonic. Thus, the frequency of the secondharmonic is 200 Hz. The period is, therefore, equal to 1/200 = 5/1000 = 0.005 sec. Thus, the correct answer ischoice A.

101. Difficulty = Moderate; Topic = Sound;The correct answer is choice A. To answer this question, you have to refer to Figure 1a. As discussed

in the explanation to question 98, the solid line is the first harmonic, the long dashed line is the secondharmonic, and the short dashed line is the third harmonic. Using the fact that the amplitude of a wave is equalto its maximum displacement, you can see that the second and third harmonics have amplitudes that are equal

Page 32: Explanations to AAMC Practice Test IIIdocshare02.docshare.tips/files/14524/145247660.pdfAAMC MCAT PRACTICE TEST III VERBAL REASONING EXPLANATIONS Passage I (Questions 1-6) This passage

Kaplan Explanations to AAMC MCAT Practice Test III Physical Sciences

30 Copyright © 2001, Kaplan, Inc.None of this material may be reproduced by any means or incorporated into any information retrieval system, without written permission.

to each other and less than the amplitude of the first harmonic. Now look at the answer choices to find the onethat represents this relationship. The graphs show amplitude versus harmonic number and consist of a series ofvertical bars with the harmonic number listed beneath each bar. The height of the bar is proportional to theamplitude, so you need to look for a graph that shows the highest bar for the first harmonic and shorter bars ofequal height for the second and third harmonics. Clearly graph A is the only one that represents thisrelationship and choice A is the correct answer.

102. Difficulty = Moderate; Topic = Sound;The correct answer is choice D, higher frequency. This question asks about the fourth harmonic for

the tone which is represented in Figure 1. As discussed in the explanation to question 98, for a given tone, thefundamental harmonic has the lowest frequency and each successive harmonic has a higher frequency than theprevious one. Therefore, the frequency of the fourth harmonic will be higher than the frequency of the thirdharmonic, and choice D is the correct answer. Choices A and B can be ruled out because there is nothing inthe passage about what determines the amplitude for a given harmonic. In fact, the information in the passageseems to indicate that the relationship between the amplitudes of the harmonics is different for different tonesand that there is no simple way to predict the amplitude of the fourth harmonic. For the tone in Figure 1, theamplitudes of the second and third harmonics are equal to each other and less than the amplitude of the firstharmonic. For the tone in Figure 2a, the third harmonic has the highest amplitude and the fifth harmonic hasthe lowest amplitude of the harmonics included on the graph. For the tone in Figure 2b, the third harmonicagain has the highest amplitude, but the harmonic with the lowest amplitude is the fourth harmonic. So youhave no way of ascertaining the amplitude of the fourth harmonic relative to that of the third.

103. Difficulty = Difficult; Topic = Wave Characteristics and Periodic Motion;The correct answer is choice A, same as the period of the first harmonic. The best way to answer this

question is to read through the answer choices and try to eliminate the wrong ones. Choice A says that theperiod of the waveform shown in Figure 1c is the same as the period of the first harmonic. Remember that thewaveform for the first harmonic is described by the solid line in Figure 1a. Looking at Figure 1a we can seethat the period of the first harmonic is the time from the origin of the time axis to the third point where allthree curves intersect which is almost at the end of the time axis. The waveform shown in Figure 1c seems tohave this same period. Although you don’t have a scale on the time axis to be absolutely precise about this,you can certainly use your eye to follow down the page from Figure 1a to Figure 1c and be reasonably surethat you’re correct. You should go through the other answer choices to check that choice A is the best answer.Choice B says that the period of the waveform in Figure 1c is the same as the period of the second harmonic.Remember that the second harmonic is represented by the long dashed curve in Figure 1a. Looking at Figure1a, you can see that the period of the second harmonic is the time from the origin to the second point wherethe three curves meet. Clearly that time is shorter than the period of the curve in Figure 1c, so you caneliminate choice B. Choice C says that the period of the waveform in Figure 1c is the same as the period ofthe third harmonic. The third harmonic is represented by the short dashed curve in Figure 1a and has a periodeven shorter than the second harmonic, so choice C can be eliminated. Choice D says that the period of thewaveform in Figure 1c is the same as the sum of the periods of the first, second, and third harmonics. Theperiod of such a curve would certainly be longer than the part of the time axis shown and since the period ofthe curve in Figure 1c is slightly less than the part of the time axis shown, choice D can also be eliminated.Therefore, choice A is the correct answer.

Passage VII (Questions 104–108)

This Research Study passage discusses photoelectric materials. These materials are useful becausethey convert light energy into electrical energy. The equation presented in the first paragraph represents theenergy conversion that takes place when light is incident on a material. Remember, energy is conserved.Therefore, it is necessary to account for all the energy that is input into the system. A new variable is definedin the first paragraph — conversion efficiency. You might want to underline this term so that it is easy to findif you need it to answer the questions. The second paragraph describes the device used to determine theconversion efficiency of two different materials as a function of the wavelength of the incident light. Don’tworry about the details of the design of the device on your first read through the passage. If you get to aquestion about the device, you can always refer back to the passage. Two important things to note about thetable are that it presents data for two different materials and that the units of the wavelength are 10–6 m. Theefficiency is a unitless quantity because it is a ratio of two energies: the electrical energy output and theincident light energy input.

104. Difficulty = Moderate; Topic = Wave Characteristics and Periodic Motion;The correct answer is choice C, 2.8 × 1014 Hz. To answer this question, you need to know that the

frequency of light f = c/λ , where c is the speed of light in a vacuum, and λ is the wavelength of light. Youalso need to know that the speed of light in a vacuum is 3 × 108 m/s. Your first step should be to refer to the

Page 33: Explanations to AAMC Practice Test IIIdocshare02.docshare.tips/files/14524/145247660.pdfAAMC MCAT PRACTICE TEST III VERBAL REASONING EXPLANATIONS Passage I (Questions 1-6) This passage

Kaplan Explanations to AAMC MCAT Practice Test III Physical Sciences

Copyright © 2001, Kaplan, Inc. 31None of this material may be reproduced by any means or incorporated into any information retrieval system, without written permission.

table to find the wavelength of light at which Material B has a conversion efficiency of 0.42. It is 1.06 × 10–6

m. Plugging the numbers into the equation for the frequency gives f = (3 × 108 )/(1.06 × 10–6) = (3/1.06) × 1014

Hz. At this point you can determine that choice C is the correct answer because it is the only answer with thecorrect order of magnitude.

105. Difficulty = Easy; Topic = Light and Geometrical Optics;The correct answer is choice C, maximize the conversion efficiency. This question requires

information from paragraphs two and three and an elementary understanding of light and its property ofreflection. Paragraph three states that the coating maximizes the absorption of light at some frequencies.When light is incident on a surface, like the surface of photoelectric materials, some of it will be absorbed andsome of it will be reflected. The number of photons incident on the material must equal the number of photonsabsorbed plus the number of photons reflected. Therefore, in order to maximize absorption, the coating mustminimize reflection.

Paragraph two states that the conversion efficiency is the fraction of light energy incident on thesurface of the material that is converted into electrical energy. It is important to realize that only the fractionof incident light that is absorbed can potentially be converted into electrical energy. The fraction of incidentlight that is reflected can not be converted into electrical energy. Therefore, if the fraction of incident lightthat is absorbed is maximized, the conversion efficiency is maximized, and choice C is correct. Choices Aand B both refer to the temperature of the device. You can rule them both out because temperature is notmentioned in the description of the coating or anywhere else in the passage. As for choice D, we justdetermined that the coating maximizes the conversion efficiency. This has the effect of maximizing theelectrical energy produced. Therefore, the coating increases the number of electrons generated by the devicein a given time which increases the magnitude of the current. So choice D is wrong.

106. Difficulty = Moderate; Topic = Electric Circuits;The correct answer is choice D, the same. This question tests your knowledge of parallel electric

circuits. You can think of the two photoelectric devices as two emf’s. The function of an emf in a circuit is tomove charge from lower to higher potential, thereby increasing the energy of the charge and supplying voltageto the circuit. In effect, when a charge passes through an emf, its electric potential energy increases by anamount equal to the charge times the voltage supplied by the emf. In this question the two identical emf’s areconnected in parallel to each other in an electric circuit. The following is a diagram of this type ofconnection.

+ –

+ –

When a charge reaches the junction between the emf’s, it will go to either the top or the bottom emf.Therefore, each charge flowing around the circuit only passes through one emf, and the potential energy ofeach charge only increases by an amount qV, where q is the magnitude of the charge and V is the voltagesupplied by the emf. Thus, the effect is the same as if there were only one emf which generated a voltage V,and choice D is the correct answer. If the two emf’s were connected in series, then there would be nojunction, and each charge flowing around the circuit would pass through both emf’s. In that case, the potentialenergy of each charge would increase by amount equal to 2qV.

107. Difficulty = Moderate; Topic = Work and Energy;The correct answer is choice A, greater than φ. This question pertains to the conservation of energy.

In the passage you are given a formula which describes the energy conversion that takes place when a photonis absorbed by a photoelectric material. The kinetic energy of the electron that is ejected is equal to theenergy of the photon that was absorbed minus the work function of the material. Therefore, part of the photon’senergy is accounted for by the work function of the material, which is the energy required to free an electronfrom the surface of a material. The remainder of the photon’s energy is converted into the kinetic energy ofthe electron that is emitted. If the photon’s energy were less than φ, the equation in paragraph one wouldrequire that the kinetic energy be negative. This is an impossibility because the kinetic energy is equal to 1/2mv2 , where m is the mass and v is the velocity and neither m nor v2 can be negative. Therefore, if the photon’senergy is less than φ, the electron would not be ejected, and choice B can be ruled out. If the photon’s energyequaled the work function of the material, the kinetic energy of the electron would be zero, and thus, thevelocity of the electron would be zero. An electron with zero velocity would not leave the surface of thematerial. Therefore, the photon’s energy must be greater than the work function of the material, which makeschoice A correct. Choices C and D are wrong because, like choice B, they violate the law of conservation ofenergy, the energy input must equal the energy output. The energy input is the energy of the absorbed photon,and the energy output is the kinetic energy of the electron plus the energy required to free the electron. If the

Page 34: Explanations to AAMC Practice Test IIIdocshare02.docshare.tips/files/14524/145247660.pdfAAMC MCAT PRACTICE TEST III VERBAL REASONING EXPLANATIONS Passage I (Questions 1-6) This passage

Kaplan Explanations to AAMC MCAT Practice Test III Physical Sciences

32 Copyright © 2001, Kaplan, Inc.None of this material may be reproduced by any means or incorporated into any information retrieval system, without written permission.

photon’s energy were less than or equal to the kinetic energy of the electron, the energy input would be lessthan the energy output because φ is always positive.

108. Difficulty = Moderate; Topic = Light and Geometrical Optics;The correct answer is choice C. This question requires straightforward graphical analysis. You are

told that Material A is coated so that the conversion efficiency ε is independent of the wavelength λ . Thisimplies that the conversion efficiency will remain constant as the wavelength of the light incident on MaterialA is varied. The only graph that represents ε as a constant is the graph in choice C. The y-coordinate, whichis ε, does not change as the x-coordinate, λ , is varied. Therefore, C is the correct answer. Choice A is wrongbecause it indicates that ε increases and then decreases as λ increases. Choice B is wrong because itindicates that ε increases linearly with λ . Choice D is wrong because it indicates that ε decrease as λincreases.

Passage VIII (Questions 109–113)

Passage VIII is all about acids and bases—a chemistry topic frequently found on the MCAT. This is aproblem-solving passage since you have to identify an unknown acid based on the information given. You are told thatthe acid is either propionic acid, crotonic acid, butyric acid, or oxalic acid. The characteristics of these acids aregiven in Table 1. An experiment is then described in which the unknown acid is titrated with a strong base (NaOH).The titration curve is shown in Figure 1.

By looking at Table 1, you can start to identify the nature of the acid before you start answering the questions.For example, at room temperature, the unknown acid exists as a liquid. Since the melting points of oxalic acid andcrotonic acid are 100°C and 71.6°C, respectively, they will not exist as liquids at room temperature and can beeliminated as candidates for the unknown. Establishing an important point such as this early on in the passage isextremely helpful.

109. Difficulty = Difficult; Topic = Phases and Phase Equilibria;The correct answer is choice C. This question asks you to identify which two acids, when compared, best

demonstrate how melting point varies with molecular weight. In other words, only the molecular weight must besignificantly different between the compounds that are compared. The only major difference between butyric acid andpropionic acid is that butyric acid possesses one more –CH2 group than propionic acid, and, therefore, has a greatermolecular weight by about 14 atomic mass units. Thus, the difference in the melting points of these two compoundscan be attributed to molecular weight.

Crotonic acid differs from all the other acids since it contains a double bond. If we were to compare propionicacid and crotonic acid (choice A), or, butyric acid and crotonic acid (choice D), the difference in their melting pointscould be attributed to structural differences. Therefore, choices A and D are incorrect. Oxalic acid is a carboxylic acidcontaining two carboxyl groups. The difference in melting points between these two compounds could be attributed tothe extra carboxyl group in oxalic acid. Therefore, choice B is incorrect and, again, choice C is the correct response.

110. Difficulty = Difficult; Topic = Acids and Bases;The correct answer is choice A. This question is testing your ability to read and interpret graphical data.

Looking at Figure 1, you can see that before any sodium hydroxide is added to the solution (that is, x = 0) the pH ofthe solution is 3. Using the equation pH = –log [H+], we can work out the hydrogen ion concentration: –log [H+] = 3,so log[H+] = –3, and [H+] = 10–3 M or 0.001 M—choice A.

You would have ended up with incorrect choice B if you accidentally calculated the hydrogen ionconcentration to be 10–2 M, while choices C and D result from the incorrect log calculations.

111. Difficulty = Difficult; Topic = Phases and Phase Equilibria;The correct response is choice C. From the information in the question stem, it appears that a calculation is

required; however, the answer choices do not contain numbers, so you can take a qualitative approach to this question.From your knowledge of colligative properties, you should know that a change in freezing point is directly proportionalto the molality of a solution; that is, an increase in the molality of a solution leads to a lowering of the freezing point.Therefore, if oxalic acid has a greater molar concentration of solute particles, it will have a lower freezing point thancrotonic acid: choice C is the correct response.

A lower concentration of solute particle causes a smaller depression of freezing point. Therefore, if crotonicacid contains a lower concentration of solute particles, it will have a higher freezing point than oxalic acid, and choiceA can be eliminated. Mass percentage of solute—stated in choices B and D—is not the relevant concentration unit forcolligative properties such as freezing point depression. In addition, oxalic acid has a greater molar mass than crotonicacid, so a solution of oxalic acid will have a higher mass percentage of solute, and choices B and D can be eliminated.

112. Difficulty = Difficult; Topic = Acids and Bases;The correct answer is choice A. The question stem tells us to identify the point where the R-COO–

concentration equals the R-COOH concentration. This point of the titration corresponds to the buffer region, where the

Page 35: Explanations to AAMC Practice Test IIIdocshare02.docshare.tips/files/14524/145247660.pdfAAMC MCAT PRACTICE TEST III VERBAL REASONING EXPLANATIONS Passage I (Questions 1-6) This passage

Kaplan Explanations to AAMC MCAT Practice Test III Physical Sciences

Copyright © 2001, Kaplan, Inc. 33None of this material may be reproduced by any means or incorporated into any information retrieval system, without written permission.

pH does not significantly change: the addition of acid results in the 'mopping up' of protons by R-COO– , so there is nota significant pH decrease; the addition of base results in the neutralization of OH– ions by R-COOH, so there is not asignificant increase in the pH. Therefore, the buffer region in Figure 1 corresponds to that part of the titration curvewhere the slope is the most horizontal, i.e., the region where there is not a significant change in the pH. Thiscorresponds to the region between 10 and 20 mL on the x-axis. The mid-point in this region, where [RCOO–] = [R-COOH], corresponds to about 15 mL NaOH, and a pH of approximately 5. Choice A is the only answer choice with avalue near to 5, so it is the correct response.

113. Difficulty = Moderate; Topic = Phases and Phase Equilibria;The correct answer is choice C. The question could be reworded as "which of the following allows us to

conclude that the unknown acid is NOT crotonic acid?" To answer this, we have to interpret information in thepassage. At room temperature (20°C), crotonic acid and oxalic acid are solids (since their melting points are higherthan 20°C). Since the unknown acid is a liquid at room temperature, crotonic acid is not a possible candidate, makingchoice C the correct answer.

Since crotonic acid is a low molecular weight carboxylic acid with a polar functional group, it will berelatively soluble in water: choice B can be eliminated. Choice A is incorrect because, according to the pKa valuesgiven in Table 1, all the acids are weak, not strong. Therefore, choice A can be eliminated. Finally, choice D isincorrect because crotonic acid has a molecular weight of 86.09 g/mole, and the student concluded that the unknownacid had a molecular weight between 85 and 92 g/mole. Therefore, this observation would support crotonic acid as theunknown.

Discretes

114. Difficulty = Easy; Topic = Kinetics and Equilibrium;The correct answer is choice C. Catalysts speed up reactions by lowering the activation energy. In the case

of a chemical equilibrium, a catalyst will speed up the rate of both the forward and reverse reaction; therefore, theequilibrium will be attained more rapidly. However, the position of the equilibrium, i.e., the relative quantities ofreactants and products present in the equilibrium mixture, will not be affected, so the equilibrium constant—Keq—willnot change: choice C is correct.

Choices A and B are incorrect since they are not general statements about all catalysts. Many catalystssimply increase the number of effective collisions between reactant molecules, not the total number of collisions.Choice B—the transfer of kinetic energy—is characteristic of an increased reaction temperature, rather than a catalyst.Finally, choice D is incorrect because, as we established in choice C, catalysts do not affect the position ofequilibrium nor the value of the equilibrium constant, Keq.

115. Difficulty = Difficult; Topic = Equilibrium and Momentum;The correct answer is choice B, 2.7 × 105 m/s. This question tests your understanding of nuclear

physics and momentum. The way to approach this problem is to use the principle of conservation ofmomentum. The initial momentum is just the momentum of the radium nucleus before it decays, zero. Thus,the final total momentum of the decay products must also equal zero. This means that the momentum of theradon nucleus must be equal in magnitude and opposite in direction to the momentum of the alpha particle.Remember, you’re told to neglect the momentum of the gamma ray. The magnitude of the momentum of thealpha particle is equal to its mass times its speed. Since an alpha particle consists of two protons and twoneutrons, it will have a mass which is approximately four times the mass of a proton. Thus, the momentum ofthe alpha particle is 4 × 1.5 × 107 using units of proton mass times m/s2 . To determine the momentum of theradon nucleus, you need to first find its mass. Since radon was formed from radium via the emission of analpha particle, the radon nucleus has 4 fewer nucleons than the original radium nucleus, or a mass number of220. Thus, the magnitude of the momentum of the radon nucleus will be 220 times its speed. Setting thismomentum equal to that of the alpha particle, you find that the speed of the radon nucleus is equal to (4/222)× 1.5 × 107 m/s. Since 222 is between 200 and 300, the speed will be greater than 2 × 105 m/s and less than 3× 105 m/s. The only answer choice that falls in this range is choice B, which is the correct answer.

116. Difficulty = Moderate; Topic = Kinetics and Equilibrium;The correct answer is choice C. To calculate the rate order of a reaction, you have to add the exponents of

each reactant in the rate law. In this law, NO2 is raised to the first power, and F2 is raised to the first power, so thetotal order of the reaction, is 1 + 1, or second order.

If you assumed that the rate of the reaction was second order with respect to NO2 , and first order with respectto F2 , you would have ended up with a third order reaction. You could have easily fallen into this trap since 2molecules of NO2 react with one molecule of F2 . However, the coefficients in the reaction equation are not theexponents in the rate law, so remember that the reaction stoichiometry does not necessarily determine the overallreaction order. Choice D can be eliminated. Choice B is incorrect because the reaction is only first order with respectto NO2 or F2 . For a zero order reaction to occur, the rate of the reaction would equal k, not k[NO2 ][F2], so choice Acan be eliminated.

Page 36: Explanations to AAMC Practice Test IIIdocshare02.docshare.tips/files/14524/145247660.pdfAAMC MCAT PRACTICE TEST III VERBAL REASONING EXPLANATIONS Passage I (Questions 1-6) This passage

Kaplan Explanations to AAMC MCAT Practice Test III Physical Sciences

34 Copyright © 2001, Kaplan, Inc.None of this material may be reproduced by any means or incorporated into any information retrieval system, without written permission.

117. Difficulty = Difficult; Topic = Thermodynamics and Thermochemistry;The correct answer is choice D. To answer this question, we need to apply the equation, overall change in

enthalpy of the reaction is equal to the enthalpy of formation of the products minus the enthalpy of formation of thereactants. For the forward reaction: ∆Hrxn = ∆Hf (products)–∆Hf (reactants). Therefore, ∆H rxn = ∆H f (H2 ) + ∆H f(Cl2 )–2∆H f (HCl). Since the enthalpy of formation of a species in its standard state is zero, it follows that for thisreaction: ∆Hrxn = –2∆Hf (HCl), and therefore, ∆Hrxn = –2(–92.5) = +185.0 kJ. A quicker route to the correct answer isby noticing that the reaction is simply the reverse reaction for the formation of 2 moles of HCl. By reversing the signof the enthalpy of the formation of HCl, and multiplying this value by 2, you would have ended up with +185.0 kJ,choice D.

118. Difficulty = Moderate; Topic = Light and Geometrical Optics;The correct answer is choice A, 1/3. This question tests your knowledge of light and geometrical

optics. To answer it, you have to know that the magnification factor is the ratio of the image height to theobject height. Remember that the magnification factor, apart from a minus sign, is just equal to the ratio ofthe image distance to the object distance. Since the image distance is 4/3 f and the object distance is 4 f, themagnification factor is (4/3)/4 = 1/3. Thus, the ratio of the image height to the object height is 1/3, andchoice A is the correct answer.

Passage IX (Questions 119–123)

This Problem Solving passage describes a device used to measure the speed of blood flowing throughan artery. Figure 1 is a diagram of a flowmeter. Note that the directions of the magnetic field and thevelocity of flow are indicated in the figure. The magnetic field is oriented horizontally from left to right andthe blood is flowing in the direction that is out of the page. The second paragraph discusses the magnetic andelectric forces on the ions in the blood and presents two potentially useful relations: V = Ed and v = V/Bd. It isuseful to underline these two relations so that they are easy to find when you are answering the questions.Note that the controlled variable is the magnetic field, which is applied across the artery, and the measuredvariables are the voltage across the artery and the diameter of the artery. The blood speed is then calculatedusing the equation v = V/Bd.

119. Difficulty = Moderate; Topic = Fluids and Solids;The correct answer is choice C, it is 2 times as fast. This question tests your knowledge of fluid

dynamics. To answer this question, you need to use the continuity equation A1 v1 = A2 v2 , where A1 and A2 arethe cross-sectional areas of the artery at two different regions of the artery, and v1 and v2 are the speeds of theblood through the two different regions. This equation implies that the speed of blood through an artery isinversely proportional to the cross-sectional area of the artery. Therefore, since the cross-sectional area of theartery at the constriction is decreased by 1/2, the speed of blood through the constricted region of the arterywill increase by a factor of 2. Therefore, the blood passing through the constricted region will travel two timesas fast as the blood passing through the normal region, and choice C is the correct answer.

120. Difficulty = Moderate; Topic = Electrostatics and Electromagnetism;The correct answer to this question is choice C, the voltage will increase. Refer back to the

description of the magnetic field set up by the flowmeter to answer this question. In the second paragraph ofthe passage, you are told that the speed of the blood at equilibrium is given by v = V/Bd, where v is the speed,V is the voltage across the artery, B is the magnitude of the magnetic field, and d is the diameter of the artery.Since the flowmeter is intended to measure the speed of blood flowing through an artery, it should not changethe speed of the blood. The reason it doesn’t change the speed is because the magnetic force it produces isperpendicular to the direction of blood flow. Therefore, v must be a constant with respect to the voltage acrossthe artery and the magnetic field, and choice D can be ruled out. The diameter of the artery should also beunaffected by the device, because according to the continuity equation if the diameter of the artery changed,the speed would change. Therefore, d is also constant with respect to V and B. One would expect that thespeed of blood through the flowmeter varies due to biological factors. Otherwise, there would be little point inmonitoring the speed of flow. However, for a specific v, V/B is equal to a constant, which means that V isdirectly proportional to B. So if B increases, V will increase, which makes choice C the correct answer.

Choice A is incorrect because the polarity of the electric field set up is independent of the strength ofthe magnetic field. The polarity of the electric field is determined by the relative positions of the positive andnegative ions in the artery. The positions of the positive and negative ions are determined by the direction ofthe magnetic forces on them due to the magnetic field. In this question, only the magnitude of the magneticfield changes not the direction of the magnetic force on the ions; therefore, the polarity of the electric fielddoesn’t change. In the passage, you are told that the voltage across the artery V = Ed , where E is themagnitude of the electric field within the artery. This implies that the magnitude of the electric field is

Page 37: Explanations to AAMC Practice Test IIIdocshare02.docshare.tips/files/14524/145247660.pdfAAMC MCAT PRACTICE TEST III VERBAL REASONING EXPLANATIONS Passage I (Questions 1-6) This passage

Kaplan Explanations to AAMC MCAT Practice Test III Physical Sciences

Copyright © 2001, Kaplan, Inc. 35None of this material may be reproduced by any means or incorporated into any information retrieval system, without written permission.

proportional to the voltage. Since the voltage increases as the magnitude of the magnetic field increases, themagnitude of the electric field must also increase, which makes choice B incorrect.

121. Difficulty = Difficult; Topic = Fluids and Solids;The correct answer is choice B, 5π × 10–6 m3 /s. This question tests your understanding of fluid

dynamics. The volume flow rate of blood in an artery is the rate at which a unit volume of blood flows acrossa cross-sectional area of artery. The volume flow rate equals the speed of flow times the cross-sectional areaof the artery. If you didn’t know the definition of the volume flow rate, you might have been able to deduce itusing dimensional analysis. The answer choices are all given in units of m3 /s, and in the question stem youare given the speed of the blood, which has units of m/s, and the diameter of the artery, which has units of m.The only way to get m3 /s from these quantities is to multiply m/s by m2 which is a speed times an area. If youremembered that the volume of a cylinder is its cross-sectional area times its length, it might have occurred toyou to multiply the cross-sectional area of the artery times the speed of flow. This would give you a quantitywith the correct units. Once you have the formula for the volume flow rate, set up the equation and plug inthe numbers. The volume flow rate equals the cross-sectional area πr2 times the speed of flow v. By pluggingin the numbers, you will see that the volume flow rate = π(0.5 × 10–2 m)2 (0.2 m/s) = 5π × 10–6 m3 /s, which ischoice B. Remember that the wrong answer choices for questions involving calculations are often the resultsof commonly made mistakes. For example, if you used the diameter instead of the radius in the aboveequation, you would have picked choice D.

122. Difficulty = Moderate; Topic = Fluids and Solids;The correct answer is choice B, 2. This question is testing the same concept as question 121, but in a

slightly different way. Here you have to know how the speed of blood in an artery is related to the volumeflow rate of blood through the artery. Since the volume flow rate equals the speed of blood times the cross-sectional area of the artery, the speed of blood is directly proportional to the volume flow rate, assuming thatthe cross-sectional area of the artery doesn’t change. Since you are not told otherwise, you can assume thatthe cross-sectional area of the artery doesn’t change. Therefore, if the speed of blood v increases by a factor of2, the volume flow rate will increase by a factor of 2, and choice B is the correct answer.

123. Difficulty = Difficult; Topic = Electrostatics and Electromagnetism;The correct answer is choice D, perpendicular to both the direction of v and the direction of B. This

question tests your understanding of electromagnetism. First note that although v is defined to be the speedand speed is a scalar, you can assume that what is meant by the direction of v is the direction of the velocityv. Also, the magnetic field is a vector quantity and should be represented as B. You should know, from theright-hand rule, that the direction of magnetic force on a charge is perpendicular to the direction in which thecharge moves and is also perpendicular to the direction of the magnetic field. If you remember this, then youcan immediately determine that choice D is the correct answer.

If you didn’t quite remember this relation, you could have eliminated two of the answer choices usingonly the information in the passage and increased your chances of guessing the correct answer. You are told inthe passage that the magnetic forces cause the positive and negative ions to move to opposite sides of theartery. The sides of the artery are the walls of the artery, so the magnetic forces on the positive and negativecharges must be directed toward the walls of the artery. This implies that the magnetic force is perpendicularto the direction of blood flow, and you can rule out choices A and B.

Passage X (Questions 124–130)

Passage X is all about sulfuric acid, its properties, how it is synthesized, and some of the reactions it willundergo. There appears to be very little extraneous information in this passage; however, do not try to memorize allthe information—you can always look back at the passage. It may help to write down (balanced or unbalanced) thereaction equations for the synthesis of sulfuric acid, and the reactions it will undergo to form HCl, HNO3 , and SO2 gas.This would have helped you to answer questions 124 and 126, for example. Let's start looking at the questions.

124. Difficulty = Moderate; Topic = Stoichiometry;The correct answer is choice B. In this question, you need to establish the oxidation number of sulfur when it

is present is sulfuric acid, the reactant, and sulfur dioxide, the product. Remember, sulfur is acting as an oxidizingagent; in other words, it is being reduced. Since reduction involves the gain of electrons, the oxidation number ofsulfur in H2SO4 must be higher than the oxidation number of sulfur in SO2 . From oxidation counting rules, the oxidationstate of sulfur in H2 SO4 can be obtained; the oxidation number of oxygen is –2, and there are 4 atoms, so the totalcharge that oxygen contributes to the molecule is –8. In addition, the H2 SO4 contains two hydrogens, each with anoxidation state of +1. Therefore, when combined, hydrogen and oxygen contribute a charge of –6 to H2 SO4. To attainneutrality, the oxidation state of sulfur must be +6. In sulfur dioxide, the total charge contributed by the oxygens is –4,so to attain neutrality in the molecule, the oxidation state of sulfur must be +4. Therefore, the oxidation state of sulfur

Page 38: Explanations to AAMC Practice Test IIIdocshare02.docshare.tips/files/14524/145247660.pdfAAMC MCAT PRACTICE TEST III VERBAL REASONING EXPLANATIONS Passage I (Questions 1-6) This passage

Kaplan Explanations to AAMC MCAT Practice Test III Physical Sciences

36 Copyright © 2001, Kaplan, Inc.None of this material may be reproduced by any means or incorporated into any information retrieval system, without written permission.

decreases from +6 to +4. (By the way, you could have taken a short cut in this question since the oxidation number ofsulfur in SO2 is +4, and the only answer choice that gives this value is choice B.)

Since sulfur acts as an oxidizing agent, the oxidation number cannot increase; therefore, choices A and C, areincorrect. In addition, choice A gives incorrect oxidation states of sulfur in H2 SO4 and SO2 (this is also a reason whychoice D is incorrect), and choice C gives the incorrect oxidation state of sulfur in SO2 .

125. Difficulty = Moderate; Topic = Phases and Phase Equilibria;The correct answer is choice B. This question tests your understanding of the reaction that is occurring to form

nitric acid, and the conditions that will favor this reaction. The passage states that HNO3 may be prepared by thereaction of sulfuric acid with the sodium salt of HNO3 's conjugate base. Therefore, in this question, NaNO3 is reactedwith H2 SO4. Sulfuric acid is added to sodium nitrate, and the resulting nitric acid that is formed vaporizes andcondenses in the vessel labeled "tube." Therefore, to maximize the yield of nitric acid, the flask must be kept abovethe boiling point of nitric acid (86°C), so that nitric acid will evaporate into the tube. In addition, the tube must bekept as cold as possible so that nitric acid will condense, thus driving the reaction toward HNO3 formation (since nitricacid is removed from the reaction flask). If the flask is maintained at a temperature of 100°C, nitric acid willevaporate; if the tube is maintained at a temperature of 0°C, nitric acid will condense. Therefore, choice B is thecorrect response.

Choice A is incorrect because a temperature of 0°C in the flask is not sufficient to remove nitric acid from thevessel, and a temperature of 100°C in the tube is not sufficient to condense nitric acid. The condensation of nitric acidwould also be prevented in choices C and D because the temperature of the tube would be 150°C and 100°C ,respectively. In addition, choices C and D would discourage a good yield of nitric acid since the boiling point ofsulfuric acid is 338°C, and the flask into which sulfuric acid drops is maintained at a temperature of 350°C. Again, thecorrect answer is choice B.

126. Difficulty = Moderate; Topic = Stoichiometry;The correct response is choice D. The question asks you to identify the correct balanced equation for the

combustion of elemental sulfur. The only answer choice that shows elemental sulfur as a reactant is choice D. Inaddition, the stoichiometry for this reaction is correct—a sulfur atom reacts with one molecule of oxygen to form onemolecule of sulfur dioxide. Choices A and B are incorrect since they show the reaction of hydrogen sulfide withoxygen, not elemental sulfur with oxygen. Choice C shows the decomposition of sulfur trioxide into elemental sulfurand oxygen, so this choice can be discarded.

127. Difficulty = Moderate; Topic = Kinetics and Equilibrium;The answer is choice C. The balanced equation for the catalytic oxidation of sulfur dioxide to form sulfur

trioxide is as follows:

2SO2(g) + O2(g) 2SO3(g)

Le Chatelier's principle states that when an equilibrium is subjected to stress, the system will move toalleviate that stress. Therefore, if SO3 —the reaction product—is removed (thus placing a stress on the system), theequilibrium will shift to the right; that is, the formation of products will be favored. Therefore, if we want to increasethe yield of sulfur trioxide, a good experimental technique would be to remove SO3 as it is produced, thus driving theforward reaction.

Choice B is incorrect since, according to Le Chatelier's principle, a reduction in pressure will result in theshifting of the equilibrium to increase the pressure. An increase in pressure can be achieved by increasing the numberof molecules in the reaction vessel, and we can see that there are 3 molecules of reactant as opposed to 2 molecules ofproduct. Therefore, decreasing the pressure will favor the formation of reactants, not products; choice B can beeliminated. If the removal of SO3 favors the formation of products, then the removal of oxygen favors the formation ofreactants. Therefore, choice D can be eliminated. We are not given any information about the enthalpy of thisreaction, so we cannot predict whether a decrease in temperature—choice A—will result in a greater yield of SO3 . Inaddition, the question asks which of the choices will most likely increase the yield of SO3 , and the best answer ischoice C.

128. Difficulty = Moderate; Topic = Stoichiometry;The correct answer is choice B. The first step in calculating the number of moles of water in 1 mL of

concentrated sulfuric acid is to find the mass of water. The mass of water in the sample is equal to the density ofconcentrated sulfuric acid—1.84 g/mL—multiplied by the volume—1 mL—multiplied by the mass percentage of waterin the sample—0.02 (remember, the passage states that concentrated sulfuric acid contains 2% water). Since themolar mass of water is 18 g/mol, the number of moles of water in the sample is (1.84 × 0.02)/18—choice B.

Page 39: Explanations to AAMC Practice Test IIIdocshare02.docshare.tips/files/14524/145247660.pdfAAMC MCAT PRACTICE TEST III VERBAL REASONING EXPLANATIONS Passage I (Questions 1-6) This passage

Kaplan Explanations to AAMC MCAT Practice Test III Physical Sciences

Copyright © 2001, Kaplan, Inc. 37None of this material may be reproduced by any means or incorporated into any information retrieval system, without written permission.

129. Difficulty = Moderate; Topic = Stoichiometry;The correct answer is choice A. One of the most common reactions of dilute mineral acids is their reaction

with metals to generate the metal salt and hydrogen gas. Therefore, the reaction of iron with dilute sulfuric acid is asfollows:

Fe(s) + H2 SO4(aq) → FeSO4 (aq) + H2 (g)

A reducing agent is defined as a species that donates electrons, and is, therefore, oxidized. Choice A iscorrect since the oxidation state of iron changes from 0 in its elemental state, to +2 when it is in solution. Choice C isincorrect since the sulfate ion acts as a spectator ion in the above reaction; it remains as SO4

2– throughout the reaction.Choice B is incorrect since FeSO4 is a product of the reaction, and a product can be neither the oxidizing nor thereducing agent. Choice D—H3 O+—is not even involved in the above reaction. In addition, H+ ions are oxidizingagents in this reaction, since the oxidation state of hydrogen is +1 in H2 SO4, and 0 in H2.

130. Difficulty = Difficult; Topic = Solution Chemistry;The correct answer is choice A. When sulfuric acid ionizes in water, the following reactions occur:

H2SO4 + H2 O → H3O+ + HSO4–

HSO4– + H2O → H3 O+ + SO4

2–

Although sulfuric acid is a strong acid and would prefer to completely ionize in water, there is not enoughwater present in concentrated sulfuric acid to ionize all the sulfuric acid. Therefore, the above reactions proceed as faras possible until there is no more water left. Since there is not enough water to completely ionize the sulfuric acid,most of the acid that does ionize forms H3 O+ and HSO4

–. Hence, SO42– has the smallest concentration, and choice A is

correct.Choice B is incorrect because, due to a lack of water, sulfuric acid will be more abundant than SO4

2– . Sincecomplete ionization does not occur in a limited amount of water, the concentration of H3 O+, according to the reactionsabove, will always be greater than the concentration of SO4

2–. Therefore, choice C is incorrect. Choice D is incorrectbecause the first ionization will occur to a greater extent than the second ionization and thus the HSO4

– concentrationwill be higher than that of SO4

2–.

Passage XI (Questions 131–136)

This Persuasive Argument passage discusses the precession of the Perihelion of Mercury’s orbit. Thefirst paragraph defines the perihelion point and describes the precession of this point about the Sun. The rateof precession is given in this paragraph as well. Don’t try to remember the exact definition of precession, justuse the definition given in the passage. Pay attention to the note that the figure is not drawn to scale. It isvery important that you remember this when you refer to the figure to answer the questions. The secondparagraph discusses the use of radar signals to determine the precession rate. The final paragraph mentionsthat, taking into account all the known planets in the solar system, Newton’s theory of gravitation does notpredict the measured precession rate of Mercury’s perihelion.

131. Difficulty = Moderate; Topic = Light and Geometrical Optics;The correct answer is choice A, Mercury was moving toward Earth at the time of the experiment. The

change in frequency can be explained in terms of the Doppler effect for light (electromagnetic waves). Youshould know that the Doppler effect for light is qualitatively similar to that for sound. Therefore, when a lightsource is moving toward an observer, or when the observer moves toward the source, the frequency seen by theobserver is greater than that emitted by the light source. This is because fewer wavefronts reach the observerper unit time when the observer and light source move toward each other, and therefore the wavelength isreduced and the frequency is increased. The exact relationship is

f o =c −v

c +v

f s ,

where fo is the frequency seen by the observer, c is the speed of light in a vacuum, v is the relative velocitybetween the source and observer, and fs is the frequency of light emitted by the source. Note that the relativevelocity between the source and observer is negative if they are moving toward each other and positive if theyare moving away from each other. Choice B is incorrect because, if the two planets were receding from eachother there would be a reduction in the frequency of the light. On the other hand, if there were no relativemotion between Mercury and Earth, the frequency of the light would not change. Since the frequency doeschange, choice C is incorrect. Whether or not Mercury is rotating (about an axis through its center) isirrelevant to the frequency shift. Only translational motion of the planets with respect to each other can resultin a frequency shift, so choice D is also wrong.

Page 40: Explanations to AAMC Practice Test IIIdocshare02.docshare.tips/files/14524/145247660.pdfAAMC MCAT PRACTICE TEST III VERBAL REASONING EXPLANATIONS Passage I (Questions 1-6) This passage

Kaplan Explanations to AAMC MCAT Practice Test III Physical Sciences

38 Copyright © 2001, Kaplan, Inc.None of this material may be reproduced by any means or incorporated into any information retrieval system, without written permission.

132. Difficulty = Difficult; Topic = Force and Motion, Gravitation;The correct answer to this question is choice A, Sun’s rotation. The best way to approach this

rotational motion question is to rule out as many wrong answer choices as possible. First, you can reason thatthe equatorial bulge must be caused by an asymmetrical effect. Choices B and C are wrong because theyindicate phenomena that would affect the Sun symmetrically. High pressures or temperatures in the interior ofthe Sun, would only lead to a spherically symmetric expansion, if anything. Similarly, the existence ofhydrogen in the atmosphere (choice D) would not cause an asymmetrical effect such as a bulge. Therefore, bythe process of elimination, choice A must be correct.

The fact that the Sun consists primarily of gaseous hydrogen and helium is an important factor in thecreation of a bulge at the equator, but only because the Sun is rotating. This can be understood by consideringa gas particle of mass m on the surface of the Sun. As depicted in the figure below, when the Sun is rotatingwith angular frequency ω, the particle will move in an orbit of radius Rcosθ, where R is the radius of theplanet and θ is the angle between the displacement of the particle from the center of the Sun and theequatorial axis. First consider the situation if the Sun were not rotating. The shape would be approximately asphere. The particles on the surface of the Sun would effectively experience a gravitational force given by F =GMm/R2 , where M is the mass of the Sun, m is the mass of the particle, and R is the distance of the particlefrom the center of the Sun.

Once the Sun begins to rotate, the net force experienced by a particle on the surface of the Sunchanges. It exhibits uniform circular motion at a radius of Rcosθ. The centripetal force can therefore bewritten as F = mv2 /Rcosθ , where v is the velocity of the particle. The velocity of the particle is equal to theangular frequency of the Sun ω multiplied by the distance between the particle and the axis of rotation Rcosθ.Substituting v = Rωcosθ, gives F = mω2 Rcosθ. Since m and ω are constants of the motion, the centripetalforce increases as Rcosθ increases and reaches a maximum at the equator, where θ = 0.

Remember, the centripetal force is the force required to keep a particle traveling in a circular orbit ofradius Rcosθ. Therefore, the force required to keep a particle at the surface of the Sun is a maximum at theequator and a minimum at the poles. Since the force of gravity is the same for all particles on the surface of asphere, the particles at the poles will feel a greater net attractive force toward the Sun than the particles at theequator. As the Sun rotates, an equilibrium will be reached such that the distance between the particles at thepoles and the center of the Sun will be less than the distance between the particles at the equator and thecenter of the Sun. This accounts for the bulge at the equator of the Sun.

ω

R

θ

mRcosθ

133. Difficulty = Difficult; Topic = Translational Motion;The correct answer is choice C, 360 × 60 × (60/500). You are told in the passage that the rate of

precession of Mercury’s perihelion is 500 arcsec/century. Therefore, the time it takes to precess 1 arcsecequals 1/500 centuries. Since there are 60 seconds of arc contained in an arcmin and 60 minutes of arc in 1°,the time it takes to precess 1° equals 60 × 60 × 1/500 centuries, and the time it takes to precess 360° equals360 × 60 × 60 × 1/500 centuries, which is choice C.

134. Difficulty = Moderate; Topic = Force and Motion, Gravitation;The correct answer is choice C, focus of the ellipse that is within the Sun’s core. This question tests

your knowledge of gravitation and orbits. Choices A and B can be dismissed immediately since theycontradict the question stem, which states that all points on the perimeter of the ellipse precess. This is alsoobvious from the figure. When referring to the figure, keep in mind the note in the passage which states that itis not drawn to scale. If you forget this, it is easy to be misled because both foci of the ellipse (as drawn inthe figure) appear to move. According to Kepler’s first law of planetary motion, all planets move in ellipticalorbits with the Sun at one focus; it is the gravitational attraction between the Sun and Mercury that is the

Page 41: Explanations to AAMC Practice Test IIIdocshare02.docshare.tips/files/14524/145247660.pdfAAMC MCAT PRACTICE TEST III VERBAL REASONING EXPLANATIONS Passage I (Questions 1-6) This passage

Kaplan Explanations to AAMC MCAT Practice Test III Physical Sciences

Copyright © 2001, Kaplan, Inc. 39None of this material may be reproduced by any means or incorporated into any information retrieval system, without written permission.

source of the motion. Therefore, the focus at the Sun is fixed and choice C is correct. Since the other focusis not within the Sun, it precesses with the orbit, and choice D is incorrect.

135. Difficulty = Difficult; Topic = Translational Motion;The correct answer is choice B, 300 sec. To answer this question, which tests your understanding of

translational motion, you have to realize that the radar signal is electromagnetic radiation or light. It thereforetravels at the speed of light in a vacuum, 3 × 108 m/s. The round-trip time t for this signal is equal to theround-trip distance, 2 × 5 × 1010 m, divided by its speed. Thus,

t = 2 × 5 × 1010 m

3 × 108 m / s =

10

3 × 102 sec ≈ 300 sec ,

which is choice B.

136. Difficulty = Difficult; Topic = Force and Motion, Gravitation;The correct answer is choice A, circle. You are told in the question stem that the major axis (or the

length) and the eccentricity (or the ratio of the length to the width) are constant over time. "Width" here isnot defined, but it is logical to assume that it refers to the length of the minor axis. The definition ofeccentricity given in the question stem is not the standard definition. Consider the following diagram of anellipse with a major axis equal to 2a and a minor axis equal to 2b and with the distance between the fociequal to 2c.

2a

2b2c

The eccentricity e is usually defined to be the ratio of the distance between the foci of an ellipse 2c to thelength of the major axis 2a . This is equivalent to 1 − b2 / a2 . Therefore, the fact that the ratio of the lengthto the width is fixed implies that the eccentricity is fixed. Since the eccentricity is constant and the length ofthe major axis is constant, the distance between the perihelion and the focus of the ellipse at the Sun’s core isconstant. This, together with the fact (discussed in the explanation to question 134) that the focus of theellipse located at the Sun’s core doesn’t precess, indicates that the figure traced by the perihelion must be acircle, and choice A is correct. Choices B and C are wrong because they both indicate that the distancebetween the perihelion changes with the precession. They also imply that the precession is not periodic, andthat the perihelion will become increasingly farther from the Sun, with Mercury eventually leaving the solarsystem. For the path of the perihelion to trace out a sphere, as choice D suggests, there would have to beforces on Mercury which were perpendicular to the line adjoining that planet with the Sun that were close inmagnitude to the Sun’s gravitational force on Mercury. We know, however, that there is nothing else in oursolar system that is as massive as the Sun, and therefore there are no sources for these large forces.

Discretes

137. Difficulty = Moderate; Topic = Electrochemistry;The correct answer is choice B. We are told that the standard potential for the reaction shown is –0.763 V.

You should know that a negative potential corresponds to a nonspontaneous reaction, and a positive potentialcorresponds to a spontaneous reaction. Since standard reduction potentials are defined in relation to the standardreduction potential of hydrogen, the reaction will spontaneously proceed in the reverse direction in the presence ofhydrogen ions; the zinc in the zinc strip will lose two electrons and become oxidized to Zn2+ . The Zn2+ ions will thencombine with 2Cl– -

ions to form ZnCl2 . Since zinc loses two electrons, 2H+ ions gain two electrons to form H2 gas.

(By the way, since the standard potential for the hydrogen half-reaction is zero, it does not affect the standard potentialof the overall process.) Therefore, the zinc strip will dissolve and ZnCl2 and H2 will form, making choice B correct.Choice A is incorrect because Zn2+ ions will combine with two Cl– ions, not one Cl– ion. Choices C and D areincorrect because they state that the zinc strip will not dissolve.

138. Difficulty = Easy; Topic = Work and Energy;The correct answer is choice D, work performed and elapsed time of work. This question tests your

understanding of work and power. To answer this question, you have to know that the power output of anautomobile engine is described in terms of the amount of work performed by the engine. The average power of

Page 42: Explanations to AAMC Practice Test IIIdocshare02.docshare.tips/files/14524/145247660.pdfAAMC MCAT PRACTICE TEST III VERBAL REASONING EXPLANATIONS Passage I (Questions 1-6) This passage

Kaplan Explanations to AAMC MCAT Practice Test III Physical Sciences

40 Copyright © 2001, Kaplan, Inc.None of this material may be reproduced by any means or incorporated into any information retrieval system, without written permission.

an engine is the work it performs divided by the amount of time required to perform that work. Lookingthrough the answer choices, you see that the only one that has both the work and the time required to performthat work is answer choice D. Thus, the correct answer is D. Choices A, B, and C can be ruled out on thebasis of dimensional analysis. There is no way to get a quantity with the units of power, watts orjoules/second, using the quantities listed in any of these wrong answer choices.

139. Difficulty = Moderate; Topic = Bonding;The correct answer is choice B. The bond length is defined as the distance between the two nuclei of the

atoms involved in a bond. Here we are asked to determine which bond is the longest. Choices B, C, and D are allbonds between nitrogen and oxygen. However, the bonds are not equivalent; the oxygen attached to nitrogen by bondB is also attached to a hydrogen, whereas the oxygens bound to nitrogen by bonds C and D are equivalent to each other(so we can immediately eliminate choices C and D).

If we were to draw the Lewis structure of this molecule, we would find that the NO3 portion of the moleculehas a negative charge. This negative charge is shared between the two oxygens joined by bonds C and D. Therefore,the molecule exists as a resonance structure. Since both of these oxygens share a negative charge, and bonds C and Dshow partial double bond character, bonds C and D will be shorter than bond B. Bond B will be the longest bond in themolecule, making choice B the correct answer.

Choice A is incorrect since the electron shell surrounding the hydrogen atom is small and therefore thehydrogen nucleus will be relatively close to the oxygen nucleus, making bond A the shortest bond in the molecule.

140. Difficulty = Moderate; Topic = Atomic and Nuclear Structure;The correct answer is choice A, 1.4 eV. This question tests your understanding of atomic physics.

You are asked to find the kinetic energy of the ejected electron when a hydrogen atom is ionized. Sinceenergy is conserved, some of the energy of the photon goes into ionizing the atom, and the remainder goes intothe kinetic energy of the ejected electron. You should realize that the ionization potential is the energyrequired to eject an electron from the ground state of an atom. Therefore, the kinetic energy of the ejectedelectron equals the energy of the photon minus the ionization potential. Plugging in the numbers, you get thatthe kinetic energy of the electron equals 15.0 eV – 13.6 eV = 1.4 eV. Thus, the correct answer is choice A.

141. Difficulty = Moderate; Topic = Atomic and Nuclear Structure;The correct answer is choice B, 3

7Li . This question tests your understanding of nuclear physics. Inthe question stem you’re told that the nucleus decays by electron capture, a form of β+ decay. You shouldknow that for β+ decay, in general, a proton in the nucleus is converted to a neutron. In the specific type of β+

decay known as electron capture, an orbital electron is captured by the nucleus and a proton in the nucleus isconverted into a neutron. Thus, the resulting nucleus will have one less proton and one more neutron than theBeryllium nucleus. Remember that the mass number of a nucleus is equal to the total number of nucleons(neutrons and protons) in the nucleus and the atomic number is equal to the number of protons in the nucleus.That means that the resulting nucleus has the same mass number as Beryllium and an atomic number that isone less than that of Beryllium. Thus, the resulting nucleus must have an atomic number of 4 –1 or 3 and amass number of 7, and choice B is the correct answer.

142. Difficulty = Difficult; Topic = Work and Energy;The correct answer is choice D, 200 J. You’re told that a particular force is used to raise a 4-kg mass

from the ground to a height of 5 m, and you are asked to determine the amount of work done by this force.Don’t be thrown off by the complex diagram or by the fact that you’re given detailed information about thesine and cosine of the angle shown in the figure. You don’t need any of this information to answer thequestion. Keep in mind that the discrete items on the MCAT often include extraneous information, so don’tworry if you can answer a question without using all the information given in the question stem. Remember,that the work done to lift a mass to a certain height above the ground is simply equal to the potential energythat the mass has at that height. Since potential energy is mass times the acceleration of gravity times theheight above the ground, we see that the potential energy equals approximately 4 × 10 × 5 = 200 J. Thus, thework done by the force F is 200 J, and choice D is the correct answer.

Page 43: Explanations to AAMC Practice Test IIIdocshare02.docshare.tips/files/14524/145247660.pdfAAMC MCAT PRACTICE TEST III VERBAL REASONING EXPLANATIONS Passage I (Questions 1-6) This passage

Copyright © 2001, Kaplan, Inc. 41None of this material may be reproduced by any means or incorporated into any information retrieval system, without written permission.

AAMC MCAT PRACTICE TEST III BIOLOGICAL SCIENCES EXPLANATIONS

Passage I (Questions 143-149)

Passage I is a persuasive-argument passage dealing with two hypotheses of cell differentiation duringembryonic development. The Mosaic Hypothesis proposes that cell differentiation is a function of the unequaldistribution of “determinants” during embryonic cell division, and is independent of external factors and cellposition in the embryo. The Regulative Hypothesis, on the other hand, proposes that cell differentiation is afunction of external factors and cell position in the embryo, and that “determinants” are equally distributedduring embryonic cell division.

Most of the questions require you to determine which hypothesis is most supported by the informationin the question stem, and why. If you were aware that the Regulative Hypothesis is the scientifically-acceptedview of cell differentiation, then you would have had a fairly easy time tackling these questions andeliminating wrong answer choices.

143. Difficulty = 85 (easy); Topic = Generalized Eukaryotic Cell;Choice C is correct. The Mosaic Hypothesis is based on an experiment in which a biologist killed

one cell of a two-celled frog embryo, leaving the dead cell attached. The resulting partially-developed embryoled the biologist to conclude that the cells of a developing embryo are independent of each other, and duringdevelopment, act like the individual components of a mosaic. Choice C most supports this hypothesis; in fact,it essentially paraphrases the results of Biologist 1’s experiment. Thus, choice C is correct.

Choices A and D both support the Regulative Hypothesis. The very fact that identical twins andtriplets even exist supports the Regulative Hypothesis. Identical twins and triplets are called identical becausethey share the same genomes; they have the same genomes because identical twins and triplets are derivedfrom a single fertilized egg. This proves that the earliest stages of cell division yield cells with a complete setof genes (as do all subsequent divisions). Each cell is independently capable of developing into a completeorganism. So, choices A and D can be eliminated. Choice B is wrong because under the tenets of the MosaicHypothesis, “determinants” (i.e., genes) coding for different body parts are parceled out to different cells. So,gut genes go to gut cells only and head genes would go to head cells only. Under this scenario, it would notbe possible for an organism to develop two heads since there would only be one set of cells that would havethe genes necessary for head development. (While you could argue that a “developmental accident” couldresult in the duplication of only the head cells and the organism would therefore develop two heads, it ishighly unlikely that this would occur.)

144. Difficulty = 80 (easy); Topic = Reproductive System and Development;Choice B is correct. The gist of the experiment in the question stem is that the nucleus of an

undifferentiated frog egg (zygote) was replaced with the nucleus of a differentiated tadpole gut cell and thatthe resulting egg developed into a complete adult frog. The fact that the activated egg with a gut-cell nucleuswas able to develop into a complete frog means that even though the tadpole gut cell was alreadydifferentiated, its nucleus had to have the same genetic material as the frog egg nucleus. Even though thenucleus formerly directed the synthesis of proteins unique to gut cells, it was able to take a step back, so tospeak, and direct the activities of an undifferentiated cell — the frog zygote. This result is more consistentwith the Regulative Hypothesis, so choice B is correct.

Choice A is wrong because radiation was used to destroy the frog-egg nucleus, not to activate thefertilized egg. Radiation disrupts normal cell activity and development. Choices C and D are wrong becausethey both contradict the experimental results given in the question stem. Furthermore, if the MosaicHypothesis were true, the frog egg would have developed into gut cells.

145. Difficulty = 75 (moderate); Topic = Reproductive System and Development;Choice C is correct. If an undifferentiated embryonic cell develops into one type of cell (e.g., a gut

cell) if left alone, but develops into another type of cell (e.g., a heart cell) when transplanted, then it isreasonable to conclude that cell development is position-dependent. Showing this would best demonstrate thevalidity of the Regulative Hypothesis, and so choice C is correct.

Choices A, B, and D are all consistent with the Mosaic Hypothesis, which claims that (1) the cells ofan embryo act as individual pieces of a developing whole; (2) during cell division, each cell is allotted onlythose genes it needs for its own development; and (3) a cell’s fate is dependent upon its allotment of genesand is independent of position and environment. Thus, choices A, B, and D can all be eliminated.

146. Difficulty = 85 (easy); Topic = Reproductive System and Development;Choice A is correct. The fact that the descendants of undifferentiated embryonic Cell B developed

into pharynx cells after Cells A and B were switched, indicates that cell differentiation during nematodedevelopment is position-dependent. Choice A best accounts for these results and is therefore correct.

Choices B, C and D are wrong because they would best account for a different set of results — the setof results that would have occurred if the Mosaic Hypothesis were actually true. If the Mosaic Hypothesis

Page 44: Explanations to AAMC Practice Test IIIdocshare02.docshare.tips/files/14524/145247660.pdfAAMC MCAT PRACTICE TEST III VERBAL REASONING EXPLANATIONS Passage I (Questions 1-6) This passage

Kaplan explanations to AAMC Practice Test III Biological Sciences

42 Copyright © 2001, Kaplan, Inc.None of this material may be reproduced by any means or incorporated into any information retrieval system, without written permission.

were true, then the descendants of Cell B would not have developed into pharynx cells; they would havedeveloped into whatever type of cell the genetic “determinants” in Cell B fated them to differentiate into.

147. Difficulty = 80 (easy); Topic = Molecular Biology;Choice D is correct. Answering this question is made easier by knowing a bit about human embryonic

development. Present-day understanding of human differentiation is based on the fact that cell division, ormitosis, yields daughter cells with a genetic composition identical to that of the parent cell. Furthermore,research has shown that differentiation is strongly influenced by both cell position and environmental factors.All somatic cells are mitotically derived from a single fertilized egg. Cell differentiation is the end result ofthree main factors: selective transcription of the genome, cell position within the embryo, and the actions ofchemical substances known as inducers. Thus, choice D is correct and choice C is wrong.

Choice A is wrong because the fact that meiosis reduces the chromosome number by half is unrelatedto our understanding of human differentiation. Meiosis is the process by which gametes are formed; mitosis isthe process by which cellular and organismal growth occurs. Both hypotheses deal with the net results ofmitotic, not meiotic, divisions. Choice B is wrong, because although it is true that genes are selectivelyactivated during cell differentiation, this is consistent with the Regulative Hypothesis, not the MosaicHypothesis. The Mosaic Hypothesis is based on the selective distribution of “determinants,” not theirselective activation. Selective activation accounts for why some cells of the pancreas secrete insulin whileothers secrete glucagon.

148. Difficulty = 75 (moderate); Topic = Generalized Eukaryotic Cell;Choice D is correct. Mitosis results in diploid daughter cells with the same number of chromosomes

(2n) as the parent cell (2n), while meiosis results in haploid daughter cells with half the number ofchromosomes (n) as the parent cell (2n). According to the figure, a sea urchin embryo cell contains 6chromosomes (2n = 6; n = 3). Since embryo cells are the result of mitosis, the nucleus of any sea urchinembryo cell will also have the same 3 pairs of chromosomes, and so choice D is correct.

Choices A and B contain the haploid number of sea urchin chromosomes (n = 3). Meiosis isresponsible for the production of haploid gametes and does not occur during the early stages of embryonicdevelopment. Choice C is wrong because it has twice the genome of the two-cell stage nucleus. Cells withmore than the diploid number of chromosomes are known as polyploid cells.

149. Difficulty = 75 (moderate); Topic = Reproductive System and Development;Choice D is correct. This question is the opposite of question 145, in which you’re asked to

demonstrate the validity of the Regulative Hypothesis. The Mosaic Hypothesis holds that cell lineage isindependent of the position of a cell in the embryo. Thus, showing that the fate of transplanted embryoniccells is independent of their new position in the embryo would be consistent with the Mosaic Hypothesis. Forexample, if an undifferentiated embryonic cell develops into one type of cell (e.g., a gut cell) whether it is leftalone or transplanted, then it is reasonable to conclude that cell development is not position-dependent.Showing this would best demonstrate the validity of the Mosaic Hypothesis, and so choice D is correct.

Choices A, B, and C are all wrong, because illustrating any of these points would actuallydemonstrate the validity of the Regulative Hypothesis, which is the scientifically-accepted view of celldifferentiation.

Passage II (Questions 150–154)

Passage II may seem like a biology passage when you first look at it; however, it is an organicchemistry passage. You will often encounter this on the MCAT—the passage gives you a biological topic, butthe questions test your understanding of organic chemistry.

There is much information in the passage, but do not attempt to memorize everything; read thepassage to understand, and only remember key points. For example, the passage tells you that the isoelectricpoint of casein is 4.6. You are more likely to be given a question about isoelectric points than, say, thecomponents of milk which are discussed in the first paragraph. Notice also that you are given theexperimental procedure to isolate some of the components of milk, and that two unidentified components areisolated—Solid 1 and Solid 2. The chances are that you will be asked to identify these components from theinformation given.

150. Difficulty = Easy; Topic = Biological Molecules;The correct answer is choice A. You are told in the passage that casein—an important milk protein—

is soluble at the normal pH of milk (6.6), but it is insoluble at a pH of 4.6. Therefore, a decrease in the pH ofmilk from 6.6 to 4.6 is associated with the precipitation of casein. In addition, you are told that when a dilutesolution of acetic acid is added to nonfat milk, a precipitate forms. This further supports the precipitation ofcasein when the pH decreases, and choice A is the correct response.

Page 45: Explanations to AAMC Practice Test IIIdocshare02.docshare.tips/files/14524/145247660.pdfAAMC MCAT PRACTICE TEST III VERBAL REASONING EXPLANATIONS Passage I (Questions 1-6) This passage

Kaplan explanations to AAMC Practice Test III Biological Sciences

Copyright © 2001, Kaplan, Inc. 43None of this material may be reproduced by any means or incorporated into any information retrieval system, without written permission.

There is no evidence in the passage that an increase in pH is associated with the precipitation ofcasein, so choice B is incorrect. We are told in the fourth paragraph that when milk is acidified, whichobviously results in a pH drop, lactose—the main carbohydrate in milk—is hydrolyzed to its constituentmonosaccharides, glucose and galactose. Therefore, when the pH decreases, no lactose will be found in milk;choice C is incorrect. As in choice B, we are not given any information that suggests lactose forms aprecipitate when the pH of milk increases, so choice D is incorrect.

By the way, you needed very little outside knowledge to answer this question. By extracting therelevant information from the passage, the answer is quite clear. Again, the correct response is choice A.

151. Difficulty = Easy; Topic = Oxygen-Containing Compounds;The correct answer is choice A. To answer this question, you need to apply your knowledge of

experimental techniques to understand what is happening in the experimental procedure in the passage.Carbonates are strong bases, hence they can accept protons. Therefore, if an excess of acetic acid is presentafter the first step, the addition of carbonate will result in "mopping up" of excess protons, and the pH of themilk will increase. In other words, any excess acetic acid is neutralized by calcium carbonate; choice A.

The passage states that Solid 1—which we established as casein in question 150—is removed afterthe acidification of milk. Ideally, all the casein should have been removed in this step, so the purpose ofcalcium carbonate addition is to deal with a component other than casein; therefore, choice B is incorrect.

Choice C is very similar to choice B in that it suggests that the purpose of calcium carbonate additionis to interfere with casein in some way. However, we have already found that casein is removed in theacidification step. Also, it does not state in the passage that calcium carbonate reacts with calcium caseinateto form casein, so choice C can be discarded. Notice that an understanding of the experiment helps you toreason why choice A is correct, and why choices B and C are incorrect.

Finally, choice D is incorrect because if calcium carbonate increased the solubility of Solid 2, wewould not be able to remove it from milk as a precipitate. Therefore, choice D contradicts why calciumcarbonate is added to the milk. Again, the correct answer is choice A.

152. Difficulty = Moderate; Topic = Oxygen-Containing Compounds;The correct answer is choice B. This question asks you about the conversion of lactose to glucose

and galactose. If you look at the part of the passage that discusses this conversion, it states that lactose ishydrolyzed by acid to glucose and galactose. The definition of hydrolysis is the alteration of a chemicalsubstance by water. If you look at the figure in the passage, you can see that lactose gains one molecule ofwater to form glucose and galactose, so you should immediately see that choice B is the correct response.You could have also arrived at the right answer by looking at the figure and noticing the key word 'hydrolyzed.'

By the way, lactose is simply a modified acetal, and like all acetals, it will readily undergo acidhydrolysis. The bond in lactose that holds together the two constituent monosaccharides is called a glycosidicbond, and cleavage of this bond is initiated by the protonation of the oxygen by an acid. Therefore, acidserves as a catalyst in the formation of glucose and galactose, not as a reactant. Remember, the questionstem asks you the net adduct, which is water, so choice D is incorrect.

Hydrogenation is the addition of H2 to a molecule. If you confused this definition with hydrolysis, youwould have ended up with incorrect choice A. Choice C is a 'red herring'; oxidation is not mentioned in theconversion of lactose to glucose and galactose. Choices A and C are confirmed as incorrect answers sinceglucose and galactose combined differ from lactose only by one molecule of H2 O, not one molecule of H2 orO2. Again, choice B is the correct response.

153. Difficulty = Moderate; Topic = Oxygen-Containing Compounds;The correct answer is choice C. The question asks why an excess of acetic acid must be avoided in

the first step of the isolation procedure. We have already established that, in the presence of acid, lactose ishydrolyzed into its constituent monosaccharides. Also, we are told in the last paragraph that the filtrateobtained at the end of the experiment is left overnight and that lactose crystallizes in the flask. If excessacetic acid was added in the first step, it is quite possible that calcium carbonate would not be able toconsume all the protons. Therefore, the remaining protons could catalyze the hydrolysis of lactose, whichcould result in its loss at the end of the procedure. Again, choice C is the correct response.

The effect of acidity upon the solubility of lactose is not an issue here. Rather, it is the hydrolysis oflactose by acid with which we are concerned; therefore, choice A is incorrect. Solid 2 forms when theneutralized solution is heated. Therefore, it is the effect of heat that determines whether solid 2 is obtained,not acidity. Hence, choice B is incorrect. Polysaccharide formation, choice D, is not addressed in thepassage. Also, since we have established that the presence of acid breaks down lactose by hydrolysis, itseems unlikely that an excess of acid will build up lactose units into a polysaccharide. This is indeed thecase—polysaccharides will only form when monosaccharides or disaccharides lose water, not gain water.

154. Difficulty = Moderate; Topic = Biological Molecules;The correct response is choice C. Let's recall what we have learned so far. After the addition of

acetic acid, Solid 1—casein—is formed, and then removed. Calcium carbonate is then added to ensure that

Page 46: Explanations to AAMC Practice Test IIIdocshare02.docshare.tips/files/14524/145247660.pdfAAMC MCAT PRACTICE TEST III VERBAL REASONING EXPLANATIONS Passage I (Questions 1-6) This passage

Kaplan explanations to AAMC Practice Test III Biological Sciences

44 Copyright © 2001, Kaplan, Inc.None of this material may be reproduced by any means or incorporated into any information retrieval system, without written permission.

excess protons do not catalyze the hydrolysis of lactose. Solid 2 is obtained when the solution is heated, andwe are told in the third paragraph that lactalbumins and lactoglobulins—two other types of protein found inmilk—are denatured and precipitated when milk is heated. Therefore, the sole components of Solid 2 must bethese two proteins, and choice C is correct.

We have already established that casein constitutes Solid 1, so choices A and B can be eliminated.When Solid 2 is filtered, lactose remains in the filtrate since, when the filtrate is left to stand overnight,lactose crystallizes. Therefore, choice D can be eliminated, and choice C is correct.

Passage III (Questions 155-159)

Passage III is a research-study passage that investigates two possible treatments for a runny nose.When a person catches a cold, the body responds by producing histamine and acetylcholine. Consequently,the histamine and acetylcholine bind to their respective receptors on the nasal mucosa cells, resulting inincreased mucous secretion (a.k.a, a runny nose).

Eighteen cold patients participated in a study comparing two “anti-runny-nose” medications. Thepatients were divided into three groups of six. The subjects in Group 1 were treated with Drug A, which blocksnasal histamine receptors and some acetylcholine receptors. The subjects in Group 2 were treated with DrugB, which blocks only nasal histamine receptors. The subjects in Group 3 were the controls. They were treatedwith placebos, so none of their nasal receptors were blocked. Four hours after treatment with the drugs, thepatients reported their symptoms on a 5-point scale ranging from dry nose (1) to excessively runny nose (5).The results were reported in Table 1.

As you read the passage and the table, you should have kept in mind that the point of the experimentwas to compare the effects of the different drugs on runny nose improvement. A quick scanning of the tableshould have quickly revealed that the patients in Group 1 had the lowest symptom scores, meaning that theyhad improved the most. Thus, Drug A was the most effective treatment. The patients in Group 2 experiencedmediocre symptom improvement, so Drug B was somewhat effective. The patients in the control group (Group3), with the exception of Patient A, did not improve at all. If you got the gist of the results, then you were ingood shape to tackle the questions.

Note: It might’ve helped to annotate the table, since it didn’t communicate much info. For example,above the Group 1 column, you could’ve written: “histamine + some acetylcholine blocking”; above theGroup 2 column, you could’ve written “histamine blocking only.” A quick note in the margin, that 1= goodand 5 = bad, could’ve saved you from losing easy points. Your pencil can be a powerful tool for dissectingMCAT passages.

155. Difficulty = 95 (easy); Topic = Respiratory and Skin Systems;Choice A is correct. Remember, all Group 3 patients were given non-drug placebos. Therefore, it

makes sense that most of Group 3 reported severe symptoms since none of the patients were treated withreceptor-blocking drugs. Although Patient A reported a dry nose, he received a placebo along with the rest ofGroup 3. Thus, since he did not receive any drugs, his improvement could not be associated with thetreatment, so choice A is correct.

Choice C is incorrect because none of the experimental drugs blocked acetylcholine receptors only.Choices B and D are also incorrect since all patients in Group 3, including Patient A, did not receive anyreceptor-blocking drugs. Thus, his response could not have been due to histamine blocking or to a combinationof histamine/acetylcholine blocking.

156. Difficulty = 60 (moderate); Topic = Molecular Biology;Choice D is correct. Pay close attention to the terminology in this question — there’s more to it than

meets the eye. The patients in Group 1, who were treated with Drug A, had the lowest symptom scores.Therefore, you can conclude that Drug A, which blocks histamine receptors and partially blocks acetylcholinereceptors, is the most effective treatment for reducing nasal secretions. So you are looking for a drugtreatment that inhibits the actions of both histamine and acetylcholine. Choice D is the only one that fits thebill; here’s why: As is obvious from its name, an antihistamine blocks the actions of histamine. And as itsname implies, acetylcholinesterase is the enzyme that breaks down acetylcholine. Administration ofacetylcholinesterase would decrease the amount of acetylcholine in circulation. In essence, antihistamine andacetylcholinesterase decrease the binding of both histamine and acetylcholine to their respective receptors.So, choice D is correct.

Choice A is wrong because an antihistamine alone is not as effective as an antihistamine plus an“anti-acetylcholine,” as was demonstrated by Drug B. Choices B and C are incorrect because administeringacetylcholine would actually make the symptoms worse, since it would stimulate the acetylcholine receptorsrather than blocking them.

Page 47: Explanations to AAMC Practice Test IIIdocshare02.docshare.tips/files/14524/145247660.pdfAAMC MCAT PRACTICE TEST III VERBAL REASONING EXPLANATIONS Passage I (Questions 1-6) This passage

Kaplan explanations to AAMC Practice Test III Biological Sciences

Copyright © 2001, Kaplan, Inc. 45None of this material may be reproduced by any means or incorporated into any information retrieval system, without written permission.

157. Difficulty = 75 (moderate); Topic = Respiratory and Skin Systems;Choice D is correct. This question is concerned with whether or not you understand how to read the

results in Table 1. You are first looking for the experimental group that received the drug that blockedhistamine blockers only. Next, you should select the individual with the lowest symptom score in that group.Drug B specifically blocked histamine receptors only, and Group 2 was the only group treated with Drug B.The individual in Group 2 who had the lowest symptom score was Patient C. Hence, choice D is correct.Choices A and B are both incorrect because they refer to individuals in Group 1. Choice C is also incorrect:Although Patient B is in Group 2, he had a symptom score of 4, which is higher than Patient C’s score of 2.Therefore, he did not benefit from the treatment as much as Patient C did.

158. Difficulty = 65 (moderate); Topic = Respiratory and Skin Systems;Choice A is correct. Epithelial tissue comprises the interior and exterior surface linings of an

organism. The epithelial tissue that lines the respiratory system contains specialized ciliated and mucous-producing cells. These respiratory linings include those of the tracheal, bronchial and nasal passages, sochoice A is correct.

Choice B is incorrect because connective tissue “holds the body together” and is composed of cellsthat produce a non-cellular connective tissue matrix. Examples of connective tissue are bone, blood andcartilage. Choice C is incorrect because contractile tissue is muscle tissue. The cells of muscle tissue arecharacterized by very specific features, such as actin and myosin fibers, which the cells of nasal passages donot have. Examples of muscle tissue are skeletal muscle, cardiac muscle, and smooth muscle. You mighthave been tempted by choice D since the cells in the nasal lining contain receptors for acetylcholine, which isa neurotransmitter. However, neurosecretory cells are neurons that secrete hormones. An example ofneurosecretory tissue is the hypothalamus, which secretes hormones such as thyrotropin-releasing hormone andluteinizing hormone-releasing hormone. Cells in the nasal lining do not secrete hormones, so choice D isincorrect.

159. Difficulty = 60 (moderate); Topic = Molecular Biology;Choice C is correct. In the nasal lining, parasympathetic nerve endings secrete acetylcholine into the

synapses separating the presynaptic nerve endings from the acetylcholine receptors found on the postsynapticnasal mucosal cells. When activated by the binding of acetylcholine, these cells are induced to increasemucus secretions. Since acetylcholinesterase is the enzyme that breaks down acetylcholine, anacetylcholinesterase inhibitor would actually increase the amount of acetylcholine in the synapse. This is thebiological equivalent of a “double negative.” Consequently, there would be increased parasympatheticactivity, resulting in an increase in nasal secretions. Thus, choice C is correct and choice D is wrong.

Choice A is wrong because an inhibitor of acetylcholinesterase increases the amount of acetylcholinein the synapse. Furthermore, if an acetylcholinesterase did block acetylcholine release (which it doesn’t), thiswould result in a decrease of nasal secretions. So choice A has two strikes against it. The same line ofreasoning can be applied to choice B, which is doubly wrong for the same reasons as choice A.

Passage IV (Questions 160–164)

Passage IV is an information-presentation passage that discusses three important facts about familialhypercholesterolemia (HC). First, the passage states that HC is associated with high cholesterol levels.Second, it implies that HC is a genetic disease. Finally, it states that HC is caused by the absence of the cellmembrane receptor that binds cholesterol-containing particles (LDL). As a result, cholesterol cannot entercells and remains in the blood.

Many of this passage’s questions test your comprehension of these three concepts, so if youunderstood the basics of HC, you would have been in good shape to answer the questions.

160. Difficulty = 90 (easy); Topic = Genetics and Evolution;Choice A is correct. The third paragraph of the passage states, “HC is relatively common in some

families and absent from others.” The paragraph goes on to say that there is no difference between the dietaryhabits of individuals of affected and unaffected families, so it is reasonable for you to conclude that HC iscaused by a genetic disorder. Choice A is essentially a paraphrase of paragraph 3. Thus, choice A is correct.

Choices B and C are incorrect because the passage specifically states that there is no correlationbetween diet and incidence of HC. Furthermore, if there were, that would be an argument against a geneticorigin for HC. So, there are two strikes against choices B and C. Choice D is wrong because the passageimplies that HC is caused by the absence of LDL receptors in all cells. This must result from a mutation inthe DNA coding for the LDL receptor. Furthermore, a DNA mutation expressed in all cells is most likelyinheritable, which supports the conclusion that HC is a genetic disorder.

Page 48: Explanations to AAMC Practice Test IIIdocshare02.docshare.tips/files/14524/145247660.pdfAAMC MCAT PRACTICE TEST III VERBAL REASONING EXPLANATIONS Passage I (Questions 1-6) This passage

Kaplan explanations to AAMC Practice Test III Biological Sciences

46 Copyright © 2001, Kaplan, Inc.None of this material may be reproduced by any means or incorporated into any information retrieval system, without written permission.

161. Difficulty = 85 (easy); Topic = Generalized Eukaryotic Cell;Choice B is correct. The defining characteristic of HC is a high cholesterol level. Thus, a drug that

causes an elevated cholesterol level in a healthy person would allow researchers to reproduce the symptoms ofHC. According to the passage, individuals with HC are missing the plasma membrane protein that binds tocholesterol, leading to high cholesterol levels in the bloodstream. Therefore, a drug that prevents the uptake ofcholesterol by cells would mimic HC symptoms. A drug that binds to the LDL receptors would preventcholesterol uptake, and so choice B is correct.

Choice A is wrong because a drug that enhances lipid absorption would most likely not appreciablyincrease the cholesterol level as long as the individual had functional LDL receptors. Choice B is wrongbecause bile is a key component of fat digestion, and inhibiting its production and secretion would result in adecrease in fat absorption. In addition, bile salts enhance the absorption of fatty acids, monoglycerides andcholesterol from the intestinal tract. So if anything, inhibiting bile would result in a decrease in cholesterollevels. (FYI: One drug treatment for high cholesterol, cholestyramine, works by binding bile acids in theintestine, thereby decreasing absorption of cholesterol into the enterohepatic circulation.) You might havebeen tempted by choice D since you know that HC is a consequence of a missing plasma membrane protein.However, if you gave a normal individual a drug that prevented the synthesis of all plasma membrane proteins,he would most likely experience the symptoms of death. Thus, choice D is incorrect.

162. Difficulty = 70 (moderate); Topic = Genetics and Evolution;Choice B is correct. In this question, you need to determine the pattern of inheritance of the HC

allele. According to the passage, there are three distinct phenotypes: no HC, moderate HC, and severe HC.Let’s assume that HC is determined by two alleles at a single locus. Let H = the normal allele, and h = theHC allele. The only possible genotypes these two alleles can produce are HH, Hh, and hh. The fact that thereare three phenotypes that correspond with the three genotypes means that neither allele can be dominant orrecessive. It is most likely that the H allele and h allele are codominant. Two alleles are said to becodominant if the two homozygote genotypes express distinct phenotypic traits and the heterozygote genotypeexpresses both traits simultaneously. The classic example is the ABO blood groups: the A and B alleles arecodominant.

Expression of the HC allele corresponds directly to the blood cholesterol level. The genotype HHwould therefore correspond to the normal phenotype, the genotype hh would correspond to the severe HCphenotype, and the genotype Hh would correspond to the moderate HC phenotype. Given that both parentshave moderate HC, this means that they are simultaneously expressing the H allele and the h allele, and thatthey are both heterozygotes with the genotype Hh. Similarly, the child expresses the severe form of HC,which means that he is expressing two copies of the allele, and that he is a homozygote with the genotype hh.Thus, choice B is correct.

Choices A, C, and D are wrong because both recessiveness and complete dominance do not permitthe expression of intermediate phenotypes. If the HC allele were completely dominant, then homozygousdominants and heterozygotes would express the same phenotype. This means that both parents and their childwould have the same form of HC. Likewise, if the HC allele was recessive, this would imply that HC wouldonly be expressed in homozygote recessives. However, these two scenarios contradict the facts as presentedin the passage.

163. Difficulty = 60 (moderate); Topic = Digestion and Excretory Systems;Choice A is correct. In a roundabout way, this question is really asking if you know how fat

absorption occurs. This topic is not even discussed in the passage, so you had to rely on your outsideknowledge. The small intestine is lined with finger-like projections known as villi. Each villus consists of alymphatic vessel, or lacteal, surrounded by a capillary bed, which in turn is surrounded by a layer ofendothelial cells. After food is digested, the carbohydrates and proteins are absorbed directly into thebloodstream via the capillaries, while fats are absorbed into the lacteals. Therefore, choice A is correct, sincevasoconstriction of lacteals would most effectively reduce fat absorption. Choices B, C, and D are wrongbecause fat absorption occurs only in the lacteals of the small intestine, and because the peritoneum is thelining surrounding the digestive organs, and is not involved in absorption.

164. Difficulty = 55 (moderate); Topic = Genetics and Evolution;Choice B is the correct answer. This question is very similar to question 160. You’re asked to

identify the statement that provides the STRONGEST support that HC is a result of genetics, not environment.Choice B fits the bill. The fact that dietary habits are unrelated to the incidence of HC suggests that HC has agenetic, rather than environmental origin.

You may have been tempted by choice A because it is a true statement. However, the sole fact thatHC is prevalent in certain families is not enough evidence to conclude that HC is a genetic disorder. ChoiceA seems to hint at a genetic origin, but doesn’t take into account the possibility that environment, e.g. diet,may play a role. Both choices C and D are incorrect because they simply describe characteristics of HC.Although they are both true statements, they neither support nor contradict the hypothesis that HC is a geneticrather than an environmentally-induced disease.

Page 49: Explanations to AAMC Practice Test IIIdocshare02.docshare.tips/files/14524/145247660.pdfAAMC MCAT PRACTICE TEST III VERBAL REASONING EXPLANATIONS Passage I (Questions 1-6) This passage

Kaplan explanations to AAMC Practice Test III Biological Sciences

Copyright © 2001, Kaplan, Inc. 47None of this material may be reproduced by any means or incorporated into any information retrieval system, without written permission.

Discretes

165. Difficulty = Moderate; Topic = Molecular Structure of Organic Compounds;The correct answer is choice A. Cis and trans isomers are types of geometric isomers; that is, isomers

that differ in their arrangement of substituents about a double bond. The terms cis and trans can only beapplied to simple disubstituted alkenes. If the alkene is tri- or tetra- substituted, the cis and trans designationsare ambiguous, and the E/Z method for naming alkenes has to be employed. Also, if one of the double-bondedcarbons bears two identical groups, the cis/trans designation is useless.

Since the molecule in choice A is trisubstituted with two fluorines and an ethyl group, it would bemore accurate to assign E /Z designations to this molecule. However, it is the best example of cis/transisomerism out of all the answer choices. If the fluorines lie on the same side of the double bond, the moleculeis a cis isomer; if the fluorines lie on opposite sides of the double bond, the molecule is a trans isomer.Therefore, choice A is the correct response.

Choices B, C, and D are incorrect since they all contain double-bonded carbons which possess twoidentical substituents, so the cis/trans designation is useless: In choice B, one of the double-bonded carbons isattached to two hydrogens; in choice C, the double-bonded carbon is part of a cyclobutane ring, and hence isattached to two equivalent –CH2 groups; and in choice D, the double-bonded carbon is attached to twohydrogens. Again, the correct answer is choice A.

166. Difficulty = 75 (moderate); Topic = Generalized Eukaryotic Cell;Choice D is correct. The breakdown of glucose begins with the process of glycolysis, which occurs in

the cytoplasm. Each 6-carbon glucose is converted into two 3-carbon pyruvate molecules, which are thentransported into the mitochondria. Within the mitochondria each pyruvate is decarboxylated to form acetylCoA, which then enters the citric acid cycle and is further broken down into two more carbon dioxidemolecules. Each carbon in the original glucose molecule ends up being part of a carbon dioxide molecule inthe mitochondria, whether through pyruvate decarboxylation or through the Krebs cycle. Therefore, thelabelled carbons would first be detected in the cytoplasm, then in the mitochondria, so choice D is correct.

Choice A is wrong since the processing of glucose begins in the cytoplasm, so the labeled carbonswill spend some time there before being transported to the mitochondria. The carbons will never spend anytime in the nucleus (choice B) or the ribosomes (choice D), so both of these choices are incorrect.

167. Difficulty = 75 (moderate); Topic = Molecular Biology;Choice C is correct. This question appears more difficult than it actually is because of the fairly

technical information about the inheritance of blood types included in the question stem. However, thisquestion is really just asking how genes work. Genes code for mRNA, which code for proteins. Often, theseproteins are enzymes. The only choice that is consistent with this general scheme is choice C. The gene forblood type codes for an enzyme that attaches a carbohydrate to a common molecular backbone. Blood typeis determined by the type of carbohydrate that is attached by the particular enzyme coded for by the bloodtype gene.

Choice A is incorrect because the RNA that is transcribed from a gene is not converted to acarbohydrate, but instead serves as a template for peptide synthesis at the ribosomes. Choice B is incorrectbecause genes do not code for carbohydrates. Choice D is incorrect because all genes contain the samecarbohydrate (deoxyribose) as part of their nucleic acid structure.

168. Difficulty = 70 (moderate); Topic = Digestive and Excretory Systems;Choice B is correct. Getting the right answer hinges on your knowledge of the alternate energy

sources used by the body during nutritional deprivation. In order of use, carbohydrates, then fats, and thenproteins, are broken down and converted to glucose, which is the ultimate energy source for essential cellfunctions. Under conditions of long-term starvation, the final energy source utilized by the body is thebreakdown of its own protein molecules. Amino acids undergo a transamination reaction and are converted tocarbohydrates. This process results in the production of ammonia, which accounts for the nitrogen seen in theurine of the famine area resident. Thus, choice B is correct. The nitrogen comes from the NH3 terminals ofthe amino acids making up the protein chains. Simply knowing that the proteins are the only alternativeenergy source containing nitrogen would also have led you to the right answer.

Glycogen is the primary storage carbohydrate. It can be broken down to glucose within the liver andmuscle tissue. However, it is used up fairly rapidly (within hours to days), and its breakdown does not yieldany nitrogen by-products, since carbohydrates are composed only of carbon, hydrogen, and oxygen. Choice Ais therefore incorrect. Adipose tissue would be utilized as an energy source only when glycogen stores haverun out. However, triglyceride metabolism also does not produce any nitrogenous waste products, so choice Cis wrong. Choice D is incorrect because there is nothing in the question stem to indicate that this person issuffering from kidney failure. Furthermore, even if there was kidney failure, this would result in nitrogenouswaste in the blood, not in the urine.

Page 50: Explanations to AAMC Practice Test IIIdocshare02.docshare.tips/files/14524/145247660.pdfAAMC MCAT PRACTICE TEST III VERBAL REASONING EXPLANATIONS Passage I (Questions 1-6) This passage

Kaplan explanations to AAMC Practice Test III Biological Sciences

48 Copyright © 2001, Kaplan, Inc.None of this material may be reproduced by any means or incorporated into any information retrieval system, without written permission.

Passage V (Questions 169-175)

Passage V is an information-presentation passage that deals with two concepts: a hypothesis aboutmitochondrial origins and details of the electron-transport system. The first part of the passage introduces whatis known as the endosymbiotic hypothesis (although the name itself is never mentioned) and providesevidence that supports it. According to this widely-accepted hypothesis, modern eukaryotic cells evolved fromthe relationship established between small, aerobic bacteria (symbionts) and larger, anaerobic eukaryotes(hosts). In time, a symbiotic relationship developed between the two organisms: The host cell provided foodand protection to the bacterium in exchange for the capacity to carry out aerobic respiration. Eventually, theybecame completely dependent upon one another, and the symbionts (now known to us as mitochondria) lostthe ability to exist outside of their eukaryotic hosts.

If present-day mitochondria and bacteria do indeed share a common ancestor, as the endosymbiotichypothesis proposes, you would expect them to share many characteristics. In fact, their many similaritiesprovide very strong evidence for the hypothesis. Bacteria and mitochondria are about the same size, reproducein the same way, contain the same type of DNA (circular, with no histones), have the same type of ribosomes,and possess the same types of enzymes. Bacteria and mitochondria also accomplish electron transport in asimilar fashion. As Figure 1 shows, electron transport in both the mitochondria and the bacteria isaccomplished with the aid of three protein complexes: NADH dehydrogenase, cytochrome b-C1, andcytochrome oxidase. These protein complexes are located on the plasma membrane of bacteria and on theinner membrane of mitochondria.

The final paragraph of the passage and the figure present an overview of the electron-transport chain.Electron transport is one of those biological concepts that drives most students crazy because it looks socomplicated, but it is actually based on simple physical concepts. The electron-transport system can bethought of as the “motor” that provides energy for ATP synthesis. Looking at the left side of the diagram, youwill see that NADH undergoes oxidation, releasing hydrogen ions (protons) and electrons. The electrons movethrough the system, moving from a higher to lower energy state, until they eventually reduce oxygen to formwater (on the right side of the diagram). The resultant energy release is used to drive the protons from theinner compartment (matrix) into the outer compartment (intermembrane space) of the mitochondrion. Theprotons are transported via an ATP-synthetase complex. This creates both a pH gradient and an electricpotential across the inner mitochondrial membrane, since the inner mitochondrial membrane is impermeableto protons. The protons pumped into the intermembrane space “want” to move back to a region of lowerconcentration, i.e., the matrix, but as previously mentioned, the membrane is not proton-permeable. When theconcentration of protons reaches a certain critical mass, however, they do flow back into the matrix along theirconcentration gradient, the same way they left it — though the ATP-synthetase complex. This movement ofthe protons down their gradient through the ATP synthetase, back into the mitochondrial matrix, creates aproton-motive force that drives ATP synthesis. (FYI: This is known as the chemiosmotic hypothesis.)

Note: When you encounter potentially confusing diagrams, such Figure 1, remember to orientyourself and take control of the situation. Pretend the diagram is a “map,” and jot down road signs to help younavigate your way through it. For example, observe that protons are moving from the bottom of the diagram tothe top. You’re told that the electron-transport system pumps protons across the inner mitochondrial membraneinto the outer compartment of the mitochondrion. Therefore, jot down that the outer mitochondrial membraneis at the top of the diagram and the matrix is at the bottom.

169. Difficulty = 75 (moderate); Topic = Molecular Biology;Choice B is correct. According to the hypothesis in the passage, mitochondria evolved from aerobic

bacteria that entered and established symbiotic relationships within anaerobic hosts (primitive eukaryotes).Because the bacteria from which mitochondria evolved were aerobic, they must have been able to carry outthe Krebs cycle and electron transport. The primitive eukaryotic hosts were anaerobic, so they could not carryout the Krebs cycle and electron transport. Therefore, choice B is correct.

Choice A is incorrect because glycolysis is required for ATP production in both aerobic and anaerobiccells. (Note: In present-day eukaryotic cells, glycolysis is the first stage of cellular respiration.) Choice C isincorrect because both types of organisms must have carried out cell division; otherwise, they would not havesurvived. Choice D is incorrect because both lineages must have carried out transcription and translation;otherwise, they could not manufacture proteins and they would not have survived.

170. Difficulty = 60 (moderate); Topic = Genetics and Evolution;Choice A is correct. Important: The answer key at the back of Practice Test III states that choice C

is the correct answer: This is a typo. (We have alerted the AAMC to the error and they have confirmed thatchoice A is the correct answer.)

This question was a lot easier if you knew that the hypothesis discussed (endosymbiotic hypothesis) iswidely accepted by the scientific community as correctly accounting for the origin of mitochondria. Knowingthis would have predisposed you to one of the “Yes;...” answer choices. According to the hypothesis, if the

Page 51: Explanations to AAMC Practice Test IIIdocshare02.docshare.tips/files/14524/145247660.pdfAAMC MCAT PRACTICE TEST III VERBAL REASONING EXPLANATIONS Passage I (Questions 1-6) This passage

Kaplan explanations to AAMC Practice Test III Biological Sciences

Copyright © 2001, Kaplan, Inc. 49None of this material may be reproduced by any means or incorporated into any information retrieval system, without written permission.

ancestors of mitochondria were bacteria, then modern bacteria and mitochondria should share manycharacteristics. Carrying the logic further, mitochondria should produce all of their own proteins. However,according to the question stem, most mitochondrial proteins are synthesized by cytoplasmic ribosomes frommRNA transcribed from nuclear, not mitochondrial, genes. You are asked if it is possible to reconcile theendosymbiotic hypothesis with this apparently contradictory observation. The only answer choice that fits thebill is choice A. Gene transfer from the symbionts (former aerobic bacteria) to the host nucleus is veryplausible and is believed to account for this discrepancy. In fact, gene transfer between bacteria is verycommon (e.g., conjugation and transduction). Further proof: Researchers have even discovered through DNAhybridization that short, non-coding mitochondrial DNA fragments have “migrated” to the eukaryotic genome.(Note: The language of the answer choice forces you to consider it as a possible correct answer. Nearlyanything “could have occurred during the last billion years of evolution.”)

Choice B, the other “Yes” answer, is incorrect because it doesn’t make any attempt to reconcile thecontradictory observation with the hypothesis. Choice B is intellectually “lazy.” It simply dismisses theobservation, as if to say, “I can’t account for this discrepancy, and I don’t even need to.” Choice C is wrongbecause the use of the phrase, “Most proteins...” in the question stem implies that the rest of the mitochondrialproteins are synthesized by the mitochondria themselves. Therefore, this is not convincing evidence thatmitochondria are not of bacterial origin. Choice D is wrong because it merely takes the information in thequestion stem (that mitochondria don’t produce all of their own proteins) and throws it back in your face, so tospeak, without ever giving you a reason why this is strong enough evidence against the hypothesis.Furthermore, choices C and D are so similar in reasoning as to effectively cancel each other out. Remember,if two answer choices are incredibly similar, then neither one of them can be right, since there is only onecorrect answer to these types of multiple-choice questions.

171. Difficulty = 55 (moderate); Topic = Molecular Biology;Choice C is correct. This question requires you to have a solid understanding of electron transport. If

you are having trouble visualizing what happens when gramicidin is inserted, go back to Figure 1 and map outwhat you think would happen. During normal electron transport, protons are actively pumped out of themitochondrial matrix via a transmembrane ATP synthetase. When the concentration of protons in theintermembrane space becomes very high, the protons move back into the matrix via the ATP synthetase,thereby providing the energy for the conversion of ADP into ATP. The insertion of gramicidin creates analternate pathway for protons to move back across the inner membrane into the matrix, along theirconcentration gradient, without releasing their energy to form ATP. Thus, when gramicidin is inserted, the rateof ATP synthesis will decrease and so choice C is correct and choice A is incorrect.

Choice B is incorrect because gramicidin, which creates artificial pathways for proton movement, hasno effect on the rate of hydrogen-atom donation by NADH, since the two are independent events. Choice D iswrong because sufficient protons will not remain between the membranes to generate ATP. Instead, theprotons will leave via the gramicidin-created pathways, thereby disrupting the reservoir of potential energy inthe intermembrane space. The decrease in potential energy would most definitely decrease the amount ofATP synthesized, since it is the potential gradient in combination with the proton gradient that drives ATPsynthesis.

172. Difficulty = 65 (moderate); Topic = Generalized Eukaryotic Cell;Choice C is correct. Important: The answer key at the back of Practice Test III states that choice D

is the correct answer: This is a typo. (We have alerted the AAMC to the error and they have confirmed thatchoice C is the correct answer.)

According to the hypothesis in the passage, mitochondria evolved from primitive aerobic bacteria thatentered larger anaerobic hosts (primitive eukaryotes). Yet, if mitochondria evolved from bacteria, how is itthat mitochondria are enveloped by two (count ‘em two) membranes while bacteria are surrounded by only onemembrane? The passage states that electron transport occurs on the plasma membrane of modern prokaryotesand on the inner membrane of mitochondria. This makes it very likely that the inner mitochondrial membranewas derived from the prokaryotic plasma membrane. Figuring out the origin of the outer mitochondrialmembrane is trickier. Remember that according to the hypothesis, the aerobic bacteria were ingested by theirprimitive eukaryotic hosts. This occurred via phagocytosis, whereby the eukaryotic plasma membraneengulfed the aerobic bacteria, forming a vesicle within the cytoplasm. Thus, the outer mitochondrialmembrane is most likely derived from the eukaryotic plasma membrane, while the inner mitochondrialmembrane is most likely derived from the prokaryotic plasma membrane. So, choice C is correct and choiceD is incorrect.

Choice A is incorrect because it implies that primitive aerobic bacteria were enveloped by twoplasma membranes. Choice B is wrong because it implies that primitive anaerobic eukaryotes were envelopedby two plasma membranes.

173. Difficulty = 55 (moderate); Topic = Microbiology;Choice D is correct. According to the hypothesis in the passage, mitochondria evolved from small

aerobic bacteria that were ingested by larger, anaerobic prokaryotes. A symbiotic relationship developed

Page 52: Explanations to AAMC Practice Test IIIdocshare02.docshare.tips/files/14524/145247660.pdfAAMC MCAT PRACTICE TEST III VERBAL REASONING EXPLANATIONS Passage I (Questions 1-6) This passage

Kaplan explanations to AAMC Practice Test III Biological Sciences

50 Copyright © 2001, Kaplan, Inc.None of this material may be reproduced by any means or incorporated into any information retrieval system, without written permission.

between the two lineages, and eventually, the modern eukaryotic line evolved. Choice D provides thestrongest evidence for this hypothesis because it proves that symbiotic relationships between prokaryotes andeukaryotes, in which the eukaryote serves as the host, can occur. The fact that this type of relationship isobserved today strongly suggests that an analogous relationship could have occurred millions of years ago.Thus, choice D is correct.

Choice A is incorrect because it provides evidence against the endosymbiotic hypothesis. Ifmitochondria and bacteria do share a common ancestor, one would expect them to have similar genomes.Choice B is incorrect because the fact that mitochondria contain hundreds of different enzymes neithersupports nor contradicts the endosymbiotic hypothesis. Choice C is incorrect because the fact thatmitochondria and bacteria are similar in diameter does not prove anything.

174. Difficulty = 55 (moderate); Topic = Microbiology;Choice C is correct. This is the type of question where you really can’t predict the right answer based

on the question stem alone. According the endosymbiotic hypothesis, mitochondria evolved from smallaerobic bacteria. If mitochondria are descended from prokaryotes, then the two should be similar in manyways. Thus, the right answer will support the hypothesis by describing a characteristic unique to all bacteria,since mitochondria should possess this trait as well. Choice C is correct because mitochondria should havecircular DNA if they are in fact descendants of bacteria, since circular DNA is one of the definingcharacteristics of all bacteria.

Choice A is incorrect because prokaryotes and mitochondria both possess 70S ribosomes, whilemodern eukaryotes possess 80S ribosomes. Choice B is incorrect because if mitochondria evolved fromprimitive prokaryotes, they must have been capable of binary fission, which is the means by which prokaryotesreproduce. You can eliminate choice D for two reasons. First, you know that mitochondria are the eukaryoticorganelles responsible for aerobic respiration. Second, the hypothesis specifically states that the primitivebacteria believed to be the ancestors of modern-day mitochondria were aerobic.

175. Difficulty = 50 (moderate); Topic = Molecular Biology;Choice B is correct. During normal electron transport, protons are actively pumped out of the

mitochondrial matrix via a transmembrane ATP synthetase. When the concentration of protons in theintermembrane space becomes very high, the protons move back into the matrix along their electrochemicalgradient, via the ATP synthetase, thereby providing the energy for the conversion of ADP into ATP. Theinsertion of valinomycin causes the movement of K+ into the mitochondrial matrix, thereby disrupting thevoltage gradient established by the proton gradient. In other words, the intermembrane space will become lesspositive with respect to the mitochondrial matrix due to the influx of positive charge (the K+) into the matrix.Since valinomycin disrupts one of the gradients responsible for the proton-motive force (the pH gradient isunaffected), ATP synthesis will decrease. So, choice B is correct and choice C is wrong, as the movement ofprotons into the intermembrane space is unaffected.

Choice A is incorrect because K+ would be less likely to compete with protons at the active site ofATP synthetase than they were before treatment with valinomycin. Assume that K+ and H+ did compete forATP synthetase’s affection (they don’t). If anything, the presence of a K+ channel would divert the K+ fromany possible competition with H+. Choice D is wrong because ATP synthesis will decrease, not increase, dueto the disruption of the electric potential gradient established by the pumping of protons out of themitochondrial matrix during electron transport.

Passage VI (Questions 176–180)

Passage VI is a typical organic chemistry passage since it describes an experimental technique. Inthese types of passages, you will be asked questions about the apparatus, so you need to understand how itworks. The apparatus is described and also drawn, so chances are you will be asked the following: whycertain components are used in the apparatus, how the experiment can be modified, and what possibleproblems might be encountered when using this apparatus.

Let's briefly discuss the principal behind fractional distillation. Fractional distillation is used toseparate liquids that differ in their volatility, and therefore, their boiling points. Obviously, a liquid that has alow boiling point and is very volatile will be removed from a mixture before a liquid with a higher boilingpoint is removed. This is achieved by heating the distillation flask; the more volatile component then movesup the fractionating column and goes through a series of condensations (when the vaporized component hits acool region of the fractionating column) and revaporizations (when the condensed liquid drops down thefractionating column, into a hotter region). The result is the accumulation of an enriched, more volatilecomponent at the top of the fractionating column, and the accumulation of an enriched, less volatilecomponent at the bottom of the column. Once the more volatile component is removed from the receivingflask, the heat applied to the distillation flask is increased to remove the second component.

A problem associated distillation in general is that when the mixture is heated to its boiling point, thecomponents of the mixture may break down, oxidize, rearrange, or undergo other reactions. To overcome this

Page 53: Explanations to AAMC Practice Test IIIdocshare02.docshare.tips/files/14524/145247660.pdfAAMC MCAT PRACTICE TEST III VERBAL REASONING EXPLANATIONS Passage I (Questions 1-6) This passage

Kaplan explanations to AAMC Practice Test III Biological Sciences

Copyright © 2001, Kaplan, Inc. 51None of this material may be reproduced by any means or incorporated into any information retrieval system, without written permission.

problem, vacuum fractional distillation is employed; reducing the pressure results in a lower boiling point foreach component, so the entire separation can be carried out at a lower temperature, and there is less risk ofthe components reacting in some way.

The two compounds to be separated in this experiment are carvone and limonene—fairly similarcompounds. Applying what we know about vacuum fractional distillation, let's start looking at the questions.

176. Difficulty = Easy; Topic = Hydrocarbons;The correct answer is choice A. Recall that the purpose of fractional distillation is to separate

components in a liquid mixture that possess different boiling points, and that the component with the lowestboiling point—which is the most volatile—will be removed from the mixture first. Since limonene andcarvone—two components of caraway seed oil—are separated by vacuum fractional distillation, they musthave different boiling points.

A liquid boils when its individual molecules overcome the intermolecular forces that hold themtogether. These intermolecular forces can be hydrogen bonds (if the molecules contain a proton attached to anelectronegative atom), dipole-dipole interactions (if the molecules possess polar functional groups), ordispersion forces—induced by the interaction of electron clouds. Hydrogen bonds and dipole-dipoleinteractions are very strong and can significantly increase the boiling point of a compound.

Carvone and limonene are structurally similar except that carvone possesses a carbonyl group. Incarbonyl groups, the oxygen is negatively polarized and the carbon is positively polarized. Therefore, thisfunctionality can undergo dipole-dipole interaction with another carvone carbonyl group. As a result, carvonehas stronger intermolecular attractions which are harder to break; therefore, carvone will possess a higherboiling point than limonene. If limonene has the lower boiling point, it follows that it will be more volatileand will be removed from the distillation flask first; choice A, therefore, is the correct response.

Choice C is incorrect because we have just established that carvone has a higher boiling point thanlimonene since it possesses stronger intermolecular forces. Choice B is incorrect for two reasons. First,limonene has a lower boiling point than carvone; second, if limonene did have a higher boiling point, it wouldnot be the first component that is distilled. Choice D is incorrect because carvone will be distilled secondsince it has a higher boiling point. Again, choice A is the correct response.

177. Difficulty = Moderate; Topic = Separations and Purifications;The correct answer is choice C. Superheating is a phenomenon in which liquids are heated above

their boiling points. This can occur in vacuum distillation since, under reduced pressure, the volume of vaporproduced is much greater. As a result, large bubbles escape from the liquid, resulting in vigorous 'bumping' ofthe liquid. The insertion of an ebulliator ensures that very small bubbles are introduced into the liquid (this isstated in the passage), which provides nucleation centers for the vapor bubbles. The presence of thesenucleation centers ensures that the liquid is not heated above its boiling point and that the distillation processoccurs smoothly, hence, superheating is avoided. Choice C, therefore, is the correct answer.

Choice D may appear to be a viable answer since the ebulliator is open to the atmosphere. However,we have just discussed that the purpose of this piece of apparatus is to provide an inlet, so that small airbubbles can enter the distillation flask, not to provide an outlet to alleviate the pressure in the distillationflask. Choice D can be eliminated. As you can see from the apparatus in Figure 1, the ebulliator is farremoved from the receiving flask. Therefore, the insertion of the ebulliator has nothing to do with the workingsof the receiving flask, and choice A is incorrect. If the ebulliator is open to the atmosphere, there is no way itcan promote a high vacuum in the in system. In fact, it is the vacuum source that promotes the vacuum in thesystem. Therefore, choice B can be eliminated. Again, the correct answer is choice C.

178. Difficulty = Moderate; Topic = Separations and Purifications;The correct answer is choice A. This question addresses another problem associated with vacuum

fractional distillation: Since the volume of vapor produced is much greater at reduced pressure, the velocity atwhich the vapor enters the condenser attached to the fractionating column is greatly increased. This results inback pressure in the fractionating column; therefore, the measured pressure in the column is falsely low. If thedistillation flask is heated at a slower rate, the difference between the actual pressure (caused by backpressure) and the measured pressure is minimized, and the separation of limonene and carvone is moreefficient. Therefore, choice A is the correct answer.

The problems associated with vacuum fractional distillation stem from reduced pressure in theapparatus. Hence, lowering the pressure even further—as stated in choice B—would aggravate theseproblems, not alleviate them; choice B can be eliminated. Cooling the condenser with ice water would notaffect the efficiency of the separation process. By the time the vapor reaches the condenser, the componentsare separated; therefore, introducing cooler water into the condenser will not affect the efficiency of theseparation process, and choice C is wrong. Choice D is incorrect since using a shorter fractionating columnwill lessen the degree of separation between carvone and limonene. In the fractionating column, a number ofsimple distillations are being carried out; the vapor condenses and revaporizes, and the more often this isdone, the more enriched the component will be at the top of the column. If the length of the column is

Page 54: Explanations to AAMC Practice Test IIIdocshare02.docshare.tips/files/14524/145247660.pdfAAMC MCAT PRACTICE TEST III VERBAL REASONING EXPLANATIONS Passage I (Questions 1-6) This passage

Kaplan explanations to AAMC Practice Test III Biological Sciences

52 Copyright © 2001, Kaplan, Inc.None of this material may be reproduced by any means or incorporated into any information retrieval system, without written permission.

decreased the component at the top of the column will be less enriched and the separation less efficient;choice D is wrong. Again, the correct answer is choice A.

179. Difficulty = Moderate; Topic = Molecular Structure of Organic Compounds;The correct answer is choice B. This question tests your understanding of stereochemistry. You are

told in the passage that carvone can exist as either of two enantiomers: (+)-carvone and (–)-carvone.Enantiomers are stereoisomers that have opposite configurations at every chiral center, so you have to identifywhich of the carbons in carvone is chiral. Remember, a chiral center is one that is attached to four differentfunctional groups. If we look at carvone, we can see that the only carbon attached to four different functionalgroups is carbon 5; it is attached to –C(CH3 )CH2, –H, –CH2 CH, and CH2 CO. Choice B is, therefore, thecorrect answer.

Carbons 2 and 7 are achiral since they are sp2 hybridized, and are only attached to three substituents.Therefore, choices A, C, and D can be eliminated, and again, choice B is the correct response.

180. Difficulty = Moderate; Topic = Separations and Purifications;The correct answer is choice A. Although this is classed as a question about oxygen-containing

compounds in the booklet, you really need to understand the principals of separation and purification to answerthis question. Reducing the pressure results in a decreased boiling point. Therefore, if there is a leak in thedistillation apparatus, the pressure will increase slightly and the components will boil at a highertemperature—choice A. Choice B states that the boiling points of the components will decrease. The onlyway the boiling point of the components could decrease would be if the pressure was reduced further. ChoiceC is incorrect since a change in pressure always affects the boiling point of a liquid. Choice D is another wayof saying that the boiling point of one component increases, while the boiling point of the other decreases.However, a change in pressure only results in either an increase or a decrease in the boiling points of all thecomponents in the mixture.

Passage VII (Questions 181-185)

Passage VII is a problem-solving passage dealing with the mechanism of action of an unidentifiedhormone, Hormone X. According to the experimental results, in normal individuals, Hormone X binds toProtein R, initiating a series of reactions that ultimately results in an increase in the amount of Ca2+ enteringthe cell. The description of the interactions between Hormone X, Protein R, and Protein P suggest thatHormone X is either a peptide hormone or an amino-acid derived neurotransmitter; that Protein R is themembrane-bound receptor to which Hormone X binds extracellularly; and that Protein P is the protein mostdirectly responsible for inducing normal cells to admit more Ca2+. In fact, it has been found that, like cAMP,Ca2+ functions as a “second messenger” for certain extracellular signalling molecules.

The organization of this passage almost mimics that of the logic game format seen on standardizedexams like the GRE and the LSAT. You really had to understand the sequence of events and theirimplications. If you were able to draw the right conclusions from the experimental results, you would havebeen able to approach most of the questions already knowing what the correct answer choice had to look like.Here are the conclusions you were expected to draw:

Result #1: Cells of affected males in both Family 1 and Family 2 do not respond to treatment withHormone XResult #2: Hormone X–Protein R binding (hormone-receptor binding) is normal in cells of affected malesin both Family 1 and Family 2Result #3: Protein R is abnormally small in cells of affected males in Family 1 onlyResult #4: Protein P phosphorylates Protein R only in female (unaffected) cellsResult #5: Protein P is abnormal in cells of affected males in Family 2 onlyResult #6: Protein P must be phosphorylated to induce an increase in the admittance of Ca2+ in normalcells

181. Difficulty = 40 (difficult); Topic = Genetics and Evolution;Choice D is correct. The information in the first paragraph of the passage makes it quite clear that

inherited Hormone X deficiencies are X-linked recessive, since the deficiencies were present only in males inthe two unrelated families. In X-linked recessive traits, a male (XY) needs only one copy of the gene toexpress the trait, while a female (XX) must have two copies — one per X chromosome. Males can onlyinherit the gene from their mothers, since mom supplies their X chromosome. Females with X-linked traits arerare, since they must inherit the gene from both mom (carrier) and dad (affected). Females are typicallycarriers of X-linked traits, which means that one of their X chromosomes carries a copy of the gene thoughthey do not express the trait themselves.

Page 55: Explanations to AAMC Practice Test IIIdocshare02.docshare.tips/files/14524/145247660.pdfAAMC MCAT PRACTICE TEST III VERBAL REASONING EXPLANATIONS Passage I (Questions 1-6) This passage

Kaplan explanations to AAMC Practice Test III Biological Sciences

Copyright © 2001, Kaplan, Inc. 53None of this material may be reproduced by any means or incorporated into any information retrieval system, without written permission.

Although the question stem tells you to assume that a defective Hormone X response is X-linked (andthat it is recessive, based on the inheritance pattern described in the passage), to get this one right you alsohad to assume that the affected male is having children with a female who is not a carrier. If you didn’t makethat assumption, then you couldn’t have gotten the right answer. Frankly, this is sloppy question-writing on thepart of the test-writers. If h = defective Hormone X allele, then the genotype of the affected male is XhY andthe genotype of the unaffected, non-carrier female is XX. Since boys inherit their X chromosome from mom,and since mom has two normal X’s, then none of the boys will be affected. In other words, none of the malesproduced by this couple will be unable to respond to Hormone X. Since girls get one normal X from mom andone affected X from dad, all girls will be carriers. But since carriers do not themselves express the disease,none of the girls will be affected. In other words, none of the females produced by this couple will be unableto respond to Hormone X. Thus, choice D is correct and choices A, B, and C are wrong.

182. Difficulty = 60 (moderate); Topic = Molecular Biology;Choice B is correct. If you were able to make the correct deduction from Result # 4, then you knew

that, of the three players here — Hormone X, Protein R, and Protein P — only Protein R most likely usesATP. According to Result #4, in female (unaffected) cells, Protein R phosphorylates Protein P after HormoneX binds to Protein R. In other words, Protein R activates Protein P by phosphorylating it; the most likelysource of this phosphate is ATP. An enzyme that phosphorylates is known as a kinase; thus, it’s most likelythat in addition to binding Hormone X, Protein R has kinase activity. Thus, choice B is correct, since it is theonly choice that contains Protein R. Choices A, C, and D are wrong because the information in the passagedoes not make it seem likely that either Hormone X or Protein P uses ATP for their actions, and because noneof these choices includes Protein R, which most likely does use ATP.

183. Difficulty = 60 (moderate); Topic = Generalized Eukaryotic Cell;Choice C is correct. Given Result #6, which implies that phosphorylated Protein P is most directly

responsible for the increase in Ca2 + influx, it can be deduced that phosphorylated Protein P most likelyfunctions as a membrane channel. Membrane channels are typically formed by transmembrane proteins.Therefore, choice C is correct.

The Na+/K+ pump is also a transmembrane channel, actively transporting 3 Na+ out of a cell for every2 K+ it transports in. However, the Na+/K+ pump does not also transport Ca2+ in its spare time, and so choiceA can be eliminated. Choice B is wrong, because a phosphatase is an enzyme that removes phosphate fromcompounds. It is highly unlikely that a phosphorylated Protein P would remove phosphate from othermolecules. Choice D is wrong because it is Protein R that most likely functions as a hormone receptor bybinding Hormone X.

184. Difficulty = 65 (moderate); Topic = Molecular Biology;Choice D is correct. According to Result #6, it the phosphorylation of Protein P that is most directly

responsible for the increase in the rate of Ca2+ entry in female (unaffected) cells. Therefore, it is logical toconclude that the influx of Ca2+ should stop immediately when Protein P is dephosphorylated. In fact, this isthe most likely mechanism by which Hormone X activity is regulated. Thus, choice D is correct.

The binding of Hormone X to Protein R, the synthesis of Protein P, and the possible entry of thishormone-receptor complex into the cytoplasm are all clearly necessary for normal cellular response toHormone X. However, the interruption of these processes would not result in the immediate shutdown of Ca2+

influx, since all of these events occur several steps before the phosphorylation of Protein P. For example,while terminating synthesis of Protein P means that no new Protein P will be synthesized, that won’t preventexisting Protein P from being phosphorylated and letting Ca2+ in. Remember, it’s the phosphorylation ofProtein P that is responsible for the influx of Ca2 +, so it makes sense that its dephosphorylation wouldimmediately stop the influx. The key here is the word immediate. Therefore, choices A, B, and C can all beeliminated.

185. Difficulty = 45 (difficult); Topic = Molecular Biology;Choice B is correct. If you drew the correct conclusion from Result #5, then you know that affected

males in Family 2 produce an abnormal form of Protein P; otherwise, the activated Protein R isolated from thefemale cells would have been able to phosphorylate the Protein P isolated from the male cells. This meansthat the DNA coding for Protein P found in the cells of affected Family 2 males has a mutation, most likely apoint mutation. Without going any further than that, you could have correctly picked choice B and eliminatedchoices A, C, and D, since none of them deals with Protein P. Choices C and D can also be eliminated on thetechnicalities of language: Proteins and protein hormones are not composed of codons; they are composed ofamino acids. Premature stop codons can only be introduced into mRNA. It is unclear whether thisterminology mismatch was intentional or not (either way, C and D are wrong).

A little info about mutations: Inherited deficiencies, such as the ones exhibited by Family 1 andFamily 2, are the result of genetic mutations. Synthesis of an abnormal protein is the result of a mutation inthe DNA that codes for the synthesis of the protein. A change as seemingly minor as the switching of one

Page 56: Explanations to AAMC Practice Test IIIdocshare02.docshare.tips/files/14524/145247660.pdfAAMC MCAT PRACTICE TEST III VERBAL REASONING EXPLANATIONS Passage I (Questions 1-6) This passage

Kaplan explanations to AAMC Practice Test III Biological Sciences

54 Copyright © 2001, Kaplan, Inc.None of this material may be reproduced by any means or incorporated into any information retrieval system, without written permission.

nitrogen base (adenine, thymine, cytosine, guanine) for another can make the difference between a normalprotein and an abnormal one.

During transcription, the mutated segment of DNA is transcribed along with the normal sequences,since the resultant mRNA is composed of the sequence of nitrogen bases complementary to the DNA. Thelocation of the transcribed mutation determines whether or not the mutation will be expressed. If it lies in anoncoding (intron) region of the mRNA, then the mutation will wind up on the cutting room floor duringmRNA processing. If the mutation lies in a coding (exon) region, then it stands a better chance of beingexpressed. mRNA is “read” by a ribosome as a sequence of codons (nucleotide triplets), each of whichspecifies a particular amino acid. During translation, the ribosome matches mRNA codons with theappropriate amino acids. Translation terminates when the ribosome encounters a stop codon in the mRNA. Ifthe DNA mutation results in the introduction of premature stop codons in the mRNA, then a shorter, possiblynonfunctional protein, will be synthesized. If the DNA mutation results in the replacement of a single aminoacid, then resulting protein may or may not be affected; some mutations do not produce any noticeable effect(due to the redundancy of the genetic code), while others can be lethal.

***Bonus Kaplan Question***Based on the passage, the mutation in the DNA of affected males from Family 1 most likely results in:

A. the replacement of a single amino acid inProtein R.

B. the replacement of a single amino acid inProtein P.

C. the introduction of a premature stop codonin the mRNA coding for Protein R.

D. the introduction of a premature stop codonin the mRNA coding for Hormone X.

According to Result #3, the cells of affected males in Family 1 produce an abnormally small ProteinR. An abnormally small protein is the result of a premature stop codon in the mRNA that codes for thatprotein. A stop codon in the mRNA signals the ribosome to stop synthesizing the protein. A premature stopcodon in the mRNA prematurely signals the ribosome to stop synthesizing the protein. Thus, a premature stopcodon in the mRNA coding for Protein R in the cells of affected males in Family 1 would result in thesynthesis of abnormally small Protein R. Thus, choice C is correct. You might have been tempted by choiceA; however, the replacement of a single amino acid implies that the Protein R produced in affected cells ofFamily 1 males would be the same size as the Protein R produced in female, unaffected cells. This clearlycontradicts the information in Result #3, so choice A is wrong.

Passage VIII (Questions 186–190)

Passage VIII tests your understanding of reaction mechanisms. Presented in Reaction 1 is a schemein which Compound 1 is converted to two different compounds when heated with sodium ethoxide. Wheneveryou are presented with reactions, always observe them closely. Notice the reaction conditions, and look at thefunctional groups in both the reactants and the products. Notice that Compound 2—which appears to havebeen formed through an elimination reaction—is the major product, and Compound 3—which appears to havebeen formed through a substitution reaction—is the minor product. Two proposed mechanisms for theformation of Compound 2 are also drawn. Although these mechanisms may look complex, do not be perturbed;the MCAT only tests basic organic chemistry. To support Mechanism A or Mechanism B, the chemists carryout a labeling experiment, shown in Scheme 1.

186. Difficulty = Moderate; Topic = Hydrocarbons;The correct answer is choice C. To answer this question, you need to understand Mechanism A and

Mechanism B. In Mechanism A, a symmetrical carbene (Intermediate A) is formed, which then rearranges toform Compound 2. Although the arrows—which indicate the movement of electrons—are drawn in aclockwise direction when Intermediate A forms Compound 2, they can easily move in a counterclockwisedirection. This would have no effect upon Compound 2 since this molecule is symmetrical, but what ifMechanism A was applied to the formation of Compounds 6 and 7 from Compounds 4 and 5? In this case, wewould expect a 50/50 mixture of products (since, according to Mechanism A, the electrons in the intermediatecan move in a clockwise or counterclockwise direction). However, since there is an uneven distribution ofproducts in Scheme 1, the reaction cannot occur by Mechanism A, making choice C the correct answer.

If compounds 4 and 5 rearranged to form Compound 6 only (Choice A), or Compound 7 only (ChoiceB), the formation of an asymmetrical intermediate would be supported, as shown in mechanism B. Therefore,choices A and B can be eliminated. Choice D is incorrect since the formation of neither Compound 6 norCompound 7 is contradictory to Reaction 1. Since an elimination product (Compound 2) is formed in

Page 57: Explanations to AAMC Practice Test IIIdocshare02.docshare.tips/files/14524/145247660.pdfAAMC MCAT PRACTICE TEST III VERBAL REASONING EXPLANATIONS Passage I (Questions 1-6) This passage

Kaplan explanations to AAMC Practice Test III Biological Sciences

Copyright © 2001, Kaplan, Inc. 55None of this material may be reproduced by any means or incorporated into any information retrieval system, without written permission.

Reaction 1, it follows that elimination products such as Compound 6 and Compound 7 will be formed inScheme 1. Again, the correct answer is choice C.

187. Difficulty = Easy; Topic = Hydrocarbons;The correct answer is choice B. The reaction of Compound 1 with sodium ethoxide to form

Compound 3 may look familiar: if the double bond in Compound 1 were absent, this reaction would becharacteristic of a nucleophilic substitution reaction. Since a mechanism for the reaction is not given, wecannot tell for certain if the reaction is nucleophilic substitution, but we can establish that the reaction is atype of substitution reaction since the bromide in Compound 1 is replaced by ethoxide to form Compound 3.Therefore, choice B is the correct response.

Elimination, choice A, involves the formation of a multiple bond with the loss of a small moleculelike a hydrogen halide or water. It is the conversion of Compound 1 to Compound 2 that is an example ofelimination—HBr is eliminated to form a triple bond. Therefore, choice A is incorrect. Cyclization is theprocess by which a straight-chain compound is converted into a compound that contains a ring. There is noring formation in the conversion of Compound 1 to Compound 3, so choice C is incorrect. Finally, choice D isincorrect because halogenation involves the addition of a halogen to a molecule. Again, this does not occur inReaction 1, and choice B is the correct answer.

188. Difficulty = Easy; Topic = Oxygen-Containing Compounds;The correct answer is choice A. The conversion of Compound 1 to Compound 2 proceeds by an

elimination reaction, i.e., HBr is eliminated from Compound 1 to form a triple bond. In the first step of theelimination reaction, sodium ethoxide dissociates into Na+ and CH3 CH2O–. The ethoxide anion then abstractsa proton to form ethanol (this is shown in Mechanisms A and B). The bromide ion that is eliminatedassociates with Na+ to form NaBr—sodium bromide. Therefore, the by-products of the elimination reaction areethanol and sodium bromide. In the reaction of Compound 1 to form Compound 3, bromide is substituted byethoxide. Again, the bromide combines with Na+ (obtained from the dissociation of sodium ethoxide), so theby-product of this reaction is also NaBr. Therefore, choice A is the correct response.

Since the ethoxide ion is a strong base, it would rather accept a proton to form ethanol than exist asits sodium salt. Therefore, choice B and C can be eliminated. In addition, choice C is incorrect becausemolecular bromine is not produced in either reaction. Choice D suggests that an alkyl halide and sodiumhydroxide are by-products of Reaction 1. If this was the case, ethanol—obtained in the elimination reaction—would have to undergo a nucleophilic substitution reaction with bromide. This is unlikely since the hydroxylgroup is a poor leaving group and, therefore, will not be easily substituted by bromide. Again, choice A is thecorrect answer.

189. Difficulty = Moderate; Topic = Molecular Structure of Organic Compounds;The correct answer is choice D. The specific rotation is the magnitude by which an optically active

compound rotates plane-polarized light. In basic organic chemistry, optically active compounds are defined asmolecules that contain at least one chiral center and do not possess an internal plane of symmetry.Compounds 2 and 6 are achiral molecules—they do not contain chiral centers and they do possess an internalplane of symmetry. Therefore, these molecules will not exhibit optical activity and will not rotate plane-polarized light, so they cannot be distinguished from each other by specific rotation, and choice D is thecorrect response.

Choice A can be eliminated because Compounds 2 and 6 CAN be distinguished by infraredspectroscopy. Unlike Compound 2, Compound 6 contains a benzene ring substituted at the para position byBr. Therefore, the spectrum of Compound 6 will show a C–Br stretch; this will be absent in the spectrum ofCompound 2. Both compounds can also be distinguished by proton NMR. Since bromine is electronegative,the aromatic protons of Compound 6 will give a signal further downfield. In addition, the pattern produced byeach compound will differ. Therefore, choice B is wrong. Gas-liquid chromatography is used to separatecompounds based on differences in their polarity. The compounds to be separated are carried in a mobilephase, and are passed over a stationary phase. Depending on the polarity, each compound will vary in itsaffinity for the stationary phase—hence, the compounds are separated on different regions of the stationaryphase. Compound 6, because of its Br group, is more polar than Compound 2. Therefore, if both compoundswere subjected to gas-liquid chromatography, they would be separated, and choice C can be eliminated.Again, choice D is the correct answer.

190. Difficulty = Moderate; Topic = Hydrocarbons;The correct answer is choice B. In Compounds 4 and 5, the Br substituent could theoretically attach

to three different positions with respect to C(C6 H5)CHBr: the para position, the meta position, and the orthoposition. The question stem asks why the para position is preferred over the ortho position. In order for anelimination reaction to occur, a bulky base (which in this case is tertiary butoxide—t-BuO– ) must abstract anacidic proton from the molecule. If bromine were in the ortho position, it would sterically hinder the incomingbulky base more so than if it were in the para position. Since steric interactions must be minimized, choice Bis the correct answer.

Page 58: Explanations to AAMC Practice Test IIIdocshare02.docshare.tips/files/14524/145247660.pdfAAMC MCAT PRACTICE TEST III VERBAL REASONING EXPLANATIONS Passage I (Questions 1-6) This passage

Kaplan explanations to AAMC Practice Test III Biological Sciences

56 Copyright © 2001, Kaplan, Inc.None of this material may be reproduced by any means or incorporated into any information retrieval system, without written permission.

Choice A is incorrect because the chemists do not want to the maximize the steric effect of thesubstituent since this would interfere with the bulky base and, hence, the elimination reaction. Choices C andD are wrong because electronegativity is not an issue here: it is steric effects that determine whether thesubstituents are ortho or para to each other. Again, the correct answer is choice B.

Discretes

191. Difficulty = 55 (moderate); Topic = Circulatory, Lymphatic, and Immune Systems;Choice D is correct. An increase in the plasma concentration of albumin will result in an increased

osmotic force pulling fluid into the capillaries from the tissue fluid. Remember that any time we havedifferent solute concentrations across a semipermeable membrane, an osmotic pressure gradient will drive themovement of fluid from the region of lower solute concentration to the region of higher solute concentration.This situation exists in the relationship between the fluid in the capillaries (plasma) and the tissue fluid(interstitial fluid) surrounding them. The capillary membrane contains pores that permit the passage of waterand small molecules, but prevents the passage of large proteins, including albumin, a major plasma protein.An increase in the serum albumin concentration will increase the difference in solute concentration across thecapillary wall, resulting in an influx of tissue fluid into the bloodstream. Thus, choice D is correct.

Choice A is incorrect because albumin is an osmoregulatory protein and plays no role in the immuneresponse. Choice B is incorrect because albumin is a plasma protein; it is not found in the tissues.Remember, the capillary membrane is impermeable to albumin. Choice C is wrong because the increasedserum albumin will pull fluid into the capillaries, not push fluid out into the tissues.

192. Difficulty = 80 (easy); Topic = Genetics and Evolution;Choice B is the correct answer. Answering this question depends on knowing the hierarchy of

taxonomic classification. From broadest to narrowest classification, the hierarchy is: Kingdom, Phylum,Class, Order, Family, Genus, and Species. As pointed out in the Biology Home Study Notes, one helpfulmnemonic is: King Phil Came Over For Good Sushi. Remember, the narrower the classification, the closerthe relationship among organisms in the group. Looking at the answer choices, the closest possiblerelationship would be among animals in the same genus, and so choice B is correct. All of the other choicesappear further up in the hierarchy than genus, and therefore include animals that are not as closely related toeach other.

193. Difficulty = 80 (easy); Topic = Genetics and Evolution;Choice A is correct. The situation described is the classic example of imprinting. Imprinting is a

rapid form of learning that occurs during a very short critical period in early life during which animals learn torespond to a specific stimulus. The ducklings in the example “imprint” the stimulus of the large, movingobject as “mom”, and will follow it throughout their early development. Choice A is the therefore correct

Choice B is incorrect because habituation refers to the general phenomenon of an organism becoming“used to” a stimulus, such that subsequent responses to the same stimulus are dampened compared to theoriginal response. For example, a watchdog that barks in response to a noise will stop barking if the noisecontinues for a prolonged period. Choice C is incorrect because conditioning refers to the training of ananimal to respond in a certain way to a specific stimulus, which can be accomplished through reinforcement orthrough pairing of stimuli. Two classic examples of conditioning: training rats to push levers in their cages byrewarding them with food afterwards, and Pavlov’s dog experiment, in which a dog was trained to salivate inresponse to a ringing bell by consistently ringing the bell immediately before feedings. Choice D is incorrectbecause discrimination refers to the ability to differentiate between stimuli; for example, distinguishing a high-pitched musical note from a low-pitched musical note.

194. Difficulty = Difficult; Topic = Hydrocarbons;The correct answer is choice C. The heat of combustion of an alkane is the enthalpy change when it

is converted to carbon dioxide and water. For example, the combustion of methane is as follows:

CH4 + 2O2 → CO2 + H2 O

The enthalpy change for this reaction is –803 kJ/mol; that is, when methane is converted to carbondioxide and water, it gives out 803 kJ/mol of heat. (By the way, you do not need to know values like this forthe MCAT.) Because heat is evolved from the combustion of hydrocarbons, the enthalpy change is oftenassigned a positive value. Since cycloalkanes are not isomers of each other, we cannot compare their heats ofcombustion directly. For this reason, the heat of combustion per –CH2 – group is compared.

The amount of heat evolved when a cycloalkane undergoes combustion is directly proportional to thestability of the molecule; the more unstable a cycloalkane, the greater the amount of potential energy each–CH2– group possesses, and the more energy that is evolved upon combustion.

Page 59: Explanations to AAMC Practice Test IIIdocshare02.docshare.tips/files/14524/145247660.pdfAAMC MCAT PRACTICE TEST III VERBAL REASONING EXPLANATIONS Passage I (Questions 1-6) This passage

Kaplan explanations to AAMC Practice Test III Biological Sciences

Copyright © 2001, Kaplan, Inc. 57None of this material may be reproduced by any means or incorporated into any information retrieval system, without written permission.

Cyclohexane—choice C—is the correct response since it is the most stable of the cycloalkanes listed.When this molecule assumes the chair configuration, it does not suffer from angle strain (it possesses the idealsp3 hybridized carbon bond angle of 109.5°), torsional strain, or nonbonded strain. As a result, it has the lowestpotential energy, and evolves the least amount of heat per –CH2 – when it undergoes combustion. Choice A—cyclopropane—will possess the highest heat of combustion per –CH2 – since this molecule has a high degree ofangle strain (recall that the C–C bond angle is forced to 60°), and a high degree of torsional strain (all thehydrogens in the molecule are eclipsed). Cyclobutane—choice B—will also have a higher heat of combustionthan cyclohexane since this molecule suffers from angle strain (the C–C bond angle is 88°) and torsionalstrain. Finally, choice D is incorrect because the carbon-carbon bond angle in cycloheptane is slightly largerthan 109.5°. Therefore, this molecule has slightly more angle strain than cyclohexane and will possess ahigher heat of combustion. Again, the correct answer is choice C.

195. Difficulty = Difficult; Topic = Oxygen-Containing Compounds;The correct answer is choice A. To answer this question, you need be familiar with carboxylic acids

and the reactions they undergo. Maleic acid and fumaric acid are both dicarboxylic acids, and if you draw outthe structures, you can see that there are two possible isomers of this compound: cis and trans. In the cisisomer, the carboxyl groups lie on the same side of the rigid carbon-carbon double bond. In the trans isomer,the carboxyl groups lie on opposite sides of the rigid double bond. It is obviously easier for the cis isomer tolose water and form an acid anhydride than it is for a trans isomer to do so since the carboxyl groups in the cisisomer are in close proximity. (Remember, an acid anhydride linkage (C–O–C) is formed when water is lostfrom two carboxyl groups.) The ease with which the cis and trans isomers dehydrate is related to thetemperature; the cis isomer, because of the proximity of the carboxyl groups, will dehydrate at a lowertemperature. It follows, therefore, that maleic acid is the cis isomer, and fumaric acid is the trans isomer. Atlow temperatures, the carboxyl groups in fumaric acid cannot rotate around the double bond to form ananhydride. At higher temperatures, however, fumaric acid can isomerize to form maleic acid, which candehydrate. Therefore, choice A is the correct answer.

Choice B is incorrect because the conversion of fumaric acid to maleic acid involves isomerization,not decarboxylation. Remember, maleic acid and fumaric acid are geometric isomers—compounds with thesame molecular formula and atomic connectivity, but with a different spatial arrangement of atoms.Therefore, loss of carbon dioxide, as choice B suggests, would result in the formation of a different compound.A chiral center is defined as an atom attached to four different functional groups. Fumaric acid (and maleicacid) does not contain any chiral centers, so choice C is incorrect. Choice D is incorrect because keto-enoltautomerism is only associated with aldehydes and ketones, not carboxylic acids. Again, choice A is thecorrect answer.

Passage IX (Questions 196-202)

Passage IX is a research-study passage dealing with the concepts of renal plasma clearance andtubular transport maximum. The general approach that is most useful here is to read through the passage once,fairly carefully, trying to pick up the big ideas but not getting too hung up on the details. This applies to tablesand figures, too, as we’ll discuss in a minute.

The first paragraph introduces the concept of plasma clearance. Plasma clearance is a means ofexpressing the capacity of the kidney to remove a substance from the plasma. It is a function of the rate atwhich a substance is cleared from the plasma and appears in the urine.

The second paragraph introduces the concept of the tubular transport maximum (Tm ), the maximumreabsorption rate possible for a substance. Substances initially enter the kidney tubule through filtration fromthe glomerulus into Bowman’s capsule. Many valuable nutrients and minerals, such as glucose, sodium,potassium, and chloride, are filtered into the kidney tubule in large amounts. It is to our advantage, however,to keep the excretion of these substances to a minimum. This is accomplished by reabsorption of materialback into the circulation from the lumen of the kidney tubule. However, if the rate at which a substance isfiltered exceeds its Tm , then even though the tubule is reabsorbing as fast as it can, excess substance will endup being excreted in the urine. We are given the specific Tm for glucose. Circle it for later reference.

The rest of the passage is concerned with two experiments done to study the plasma clearance ofSubstance A, a hexose sugar. The first experiment involved measuring the concentrations of glucose andSubstance A in the plasma and the urine as these substances were continuously injected intravenously. Theresults are shown in Figure 1. Some quick advice about dealing with MCAT graphs: 1) check out the axes; 2)look for general trends; 3) annotate if appropriate and 4) move on. A common and dangerous pitfall is tobecome obsessed with understanding everything presented in a graph before moving on to the questions. Youcan always refer back if necessary and often times the questions are based on a rudimentary understanding ofthe data presented. Here, we see that the plasma concentration is on the X-axis, and the urine concentration ison the Y-axis. At a plasma concentration of just over 6 mg/mL, Substance A begins to appear in the urine,while glucose first appears in the urine when its plasma concentration exceeds 10 mg/mL. A secondexperiment was conducted to determine the effects of ADH on plasma clearance. A subject was given two

Page 60: Explanations to AAMC Practice Test IIIdocshare02.docshare.tips/files/14524/145247660.pdfAAMC MCAT PRACTICE TEST III VERBAL REASONING EXPLANATIONS Passage I (Questions 1-6) This passage

Kaplan explanations to AAMC Practice Test III Biological Sciences

58 Copyright © 2001, Kaplan, Inc.None of this material may be reproduced by any means or incorporated into any information retrieval system, without written permission.

separate does of ADH. We are not told how urine output was ultimately affected; however, we are told that ahigh dose of ADH increased blood pressure.

196. Difficulty = 80 (easy); Topic = Digestive and Excretory Systems;Choice C is correct. The key to answering this question lies in paragraph two where Tm is defined.

Tm is the maximum rate of transport at which a substance can be reabsorbed by the kidney. In other words,when the rate of plasma filtration reaches/exceeds the Tm of a substance, the substance will begin to appear inthe urine. This is described in choice C.

Choices A and B are incorrect because Tm is defined in terms of the reabsorption rate, not in terms ofthe rate of concentration of a substance in the filtrate or the urine. Choice D is incorrect as bladder capacity isindependent of Tm .

197. Difficulty =75 (moderate); Topic = Digestive and Excretory Systems;Choice A is correct. You are told that the tubular load of glucose under normal conditions is equal to

approximately 125 mg/min. According to the passage, the Tm for glucose is 320 mg/min. In other words, thenormal tubular load of glucose is far less than the Tm of glucose, so all of the glucose will be reabsorbed andthere will be no glucose in the urine.

Note: A little outside knowledge could have gotten you to the right answer instantaneously. Youmight have known that under normal conditions, glucose does not appear in the urine. Glycosuria (glucose inthe urine) is a hallmark of diabetes mellitus.

198. Difficulty = 55 (moderate); Topic = Circulatory, Lymphatic, and Immune Systems;Choice C is correct. Plasma clearance describes the kidney’s ability to clear a substance from the

blood and excrete into the urine. Plasma clearance will be zero unless the rate of filtration of a substance isgreater than the substance’s Tm . The glomerular filtration rate is directly proportional to blood pressure. Ifblood pressure is lower than normal, then fluid will be filtered into the tubule at a slower rate and urineproduction will slow down. This affords the kidneys more time for reabsorption of Substance A. The moreSubstance A that is reabsorbed back into the plasma, the lower its plasma clearance, since less Substance Awill be removed from the plasma and excreted in the urine. Thus choice C is correct and A and B are wrongas they suggest that plasma clearance of Substance A would increase. Choice D is wrong because increasedADH secretion is a compensatory response to decreased blood pressure, not the driving force behind thedecreased plasma clearance of Substance A. Remember, ADH (vasopressin) is a hormone secreted by theposterior pituitary in response to a decrease in blood pressure. It increases blood volume by enhancing waterreabsorption in the kidneys.

199. Difficulty = 55 (moderate); Topic = Circulatory, Lymphatic, and Immune Systems;Choice B is correct. Answering this question involves interpreting Figure 1. The question tells us that

a volunteer’s plasma has 8 mg/ml of both glucose and Substance A. According to Figure 1, at 8 mg/ml ofsubstance A in the plasma, there is approximately 2 mg/ml of substance A in the urine. At this concentration,the plasma is being cleared of substance A, therefore we know that its Tm has been exceeded. We don’t needto know the value of Substance A’s Tm; we just know that if there is substance A in the urine, its Tm has beensurpassed. In comparison, at the same plasma concentration, there is no glucose in the urine; its Tm has notyet been reached and no glucose is being cleared from the plasma. So, choice B is correct and C and D arewrong.

Choice A is incorrect because the slope of the clearance line for Substance A is actually lower thanthat for glucose, and more importantly, at 8 mg/ml, the is no clearance line for glucose.

200. Difficulty = 55 (moderate); Topic = Circulatory, Lymphatic, and Immune Systems;Choice B is correct. We are told in paragraph 2 that if the filtration rate of a substance exceeds its

Tm, then the substance will appear in the urine. According to Figure 1, glucose first begins to appear in theurine when the plasma concentration just exceeds 10 mg/mL. Therefore, the Tm for glucose is reached atapproximately 10 mg/ml; any increase in plasma concentration above this level will result in the presence ofglucose in the urine, as shown in Figure 1.

201. Difficulty = 30 (difficult); Topic = Circulatory, Lymphatic, and Immune Systems;Choice A is correct. As discussed in the explanation for question 198, the filtration of plasma is

driven mainly by blood pressure in the glomerular capillaries. An increase in blood pressure, whether from ahigh ADH concentration or from any other cause, will result in an increase in the glomerular filtration rate andtherefore an increase in urine output. Thus, choice A is correct.

Choice B is incorrect because Tm is unaffected by ADH, or any other factor for that matter, since Tm isa constant for each substance. Choice C is a bit tricky. (I bet most of the 70% of those who got this questionwrong chose C.) ADH does indeed increase water reabsorption from the tubules. However, you were askedspecifically about the effects of ADH (increased blood pressure) on urinary output, not on the actions of ADH.

Page 61: Explanations to AAMC Practice Test IIIdocshare02.docshare.tips/files/14524/145247660.pdfAAMC MCAT PRACTICE TEST III VERBAL REASONING EXPLANATIONS Passage I (Questions 1-6) This passage

Kaplan explanations to AAMC Practice Test III Biological Sciences

Copyright © 2001, Kaplan, Inc. 59None of this material may be reproduced by any means or incorporated into any information retrieval system, without written permission.

Choice D is incorrect because the concentrating ability of the loop of Henle is driven by solute concentrationgradients, not by blood pressure.

202. Difficulty = 50 (moderate); Topic = Digestive and Excretory Systems;Choice B is correct. The main function of the proximal tubule is the reabsorption of filtered nutrients

such as amino acids, glucose, and minerals. Thus, choice B is correct. Choice A is incorrect because theglomerulus is the capillary bed through which plasma is filtered into the Bowman’s capsule of the nephron. Noreabsorption occurs in the glomerulus. Choice C is wrong because the distal tubule is the segment of thenephron where sodium is reabsorbed and potassium is excreted due to the action of aldosterone. The latedistal tubule (along with the collecting duct) is the site of water reabsorption driven by the presence of ADH.Choice D is wrong because the loop of Henle is responsible for maintenance of the gradient that allows for theexcretion of urine with a wide potential range of concentrations.

Passage X (Questions 203-207)

Passage X is an information-presentation passage, although, quite frankly, most of the informationpresented is not actually necessary to answer the majority of the questions. The main point of the passage isthat although both men and women experience some degree of osteoporosis after age 40, the rate of bone lossis accelerated in postmenopausal women due to estrogen deficiency. The average question difficulty = 61,making this a moderate passage.

FYI: The scope of Passage X and its questions are eerily similar to a Kaplan-written passage:Passage 1 of Biology Lesson 1. Yes, it’s true: Kaplan writers are also psychic. It’s a pre-requisite of the job.

203. Difficulty = 70 (moderate); Topic = Muscle and Skeletal Systems;Choice A is correct. The average adult female height and weight are significantly lower than the

average adult male height and weight. This is because adult women have less bone mass than men. So, inaddition to estrogen deficiency, a lower bone mass predisposes postmenopausal women to osteoporosis. Thus,choice A is correct. Choice B is wrong because all human beings have the same number of vertebrae (withthe possible exception of my grandmother). Choices C and D are wrong, because although aging results inless efficient uptake of calcium, these effects are felt equally by men and women, as specifically stated inparagraph 2.

204. Difficulty = 65 (moderate); Topic = Reproductive System and Development;Choice C is correct. Estrogen and progesterone are the two primary hormones responsible for the

monthly build-up of endometrial tissue and subsequent sloughing off (menstrual flow) in premenopausalwomen. Therefore, it is logical that postmenopausal women receiving estrogen and progesterone therapy inthe treatment of osteoporosis would experience a return of periodic menstruation (no pun intended), and sochoice C is correct. Even though a postmenopausal woman does not ovulate, her uterus can be “fooled” intopreparing itself for the implantation of a fertilized egg through the administration of the proper hormones.Without the presence of a fertilized egg, the endometrium sloughs off, giving rise to the menstrual flow. Inessence, this is what “the pill” (oral contraceptive) does in premenopausal women. The pill contains estrogenand progesterone analogues, which together inhibit the onset of ovulation by inhibiting the hormone GnRH.Inhibition of GnRH inhibits LH and FSH secretion, thereby preventing the maturation of ovarian follicles.Women on the pill do not ovulate; however, they still menstruate on a monthly basis.

In addition to the increased rate of bone loss, the loss of estrogen due to menopause is accompaniedby atrophy of breast tissue, uterine tissue, and mucus-secreting vaginal epithelium. Thus, postmenopausalwomen receiving estrogen and progesterone therapy would be expected to experience a reversal of thesesymptoms, and so choices A and B are wrong. Since the hormones prolactin and oxytocin are responsible forinducing lactation, estrogen therapy would neither induce nor inhibit lactation, and so choice D is wrong.

205. Difficulty = 50 (moderate); Topic = Nervous and Endocrine Systems;Choice D is correct. According to the passage, parathyroid hormone and calcitonin are the two

hormones involved in the regulation of calcium homeostasis. That information alone should have been enoughfor you to realize that the right answer had to be either choice B or choice D. Parathyroid hormone is secretedin response to a decrease in plasma calcium concentration. Parathyroid hormone raises plasma calcium byincreasing bone resorption, mobilizing calcium ion stores, and increasing calcium ion reabsorption in thekidneys. Parathyroid hormone also increases calcium ion absorption from the small intestine by increasing theformation of metabolically active vitamin D. Parathyroid hormone production would therefore be inhibited bythe administration of dietary calcium, since this treatment would increase plasma calcium, not decrease it.Thus, choice D is correct.

Calcitonin is secreted by the thyroid gland in response to a rise in plasma calcium concentration.Calcitonin tones down plasma calcium by inhibiting bone resorption and increasing calcium ion excretion inthe urine. Therefore the administration of dietary calcium to prevent osteoporosis would stimulate the

Page 62: Explanations to AAMC Practice Test IIIdocshare02.docshare.tips/files/14524/145247660.pdfAAMC MCAT PRACTICE TEST III VERBAL REASONING EXPLANATIONS Passage I (Questions 1-6) This passage

Kaplan explanations to AAMC Practice Test III Biological Sciences

60 Copyright © 2001, Kaplan, Inc.None of this material may be reproduced by any means or incorporated into any information retrieval system, without written permission.

production of calcitonin, and so choice B is incorrect. Although one of the actions of growth hormone is tostimulate bone growth, the secretion of growth hormone is regulated by a feedback mechanism with thehypothalamus; plasma calcium concentration is independent of growth hormone secretion. So, choice A iswrong, since an increase in plasma calcium concentration would not affect growth hormone secretion. Thissame line of reasoning holds for choice C. Thyroid hormone secretion is regulated primarily by the circulatingconcentration of the pituitary hormone TSH (thyroid-stimulating hormone). Though its effects are widespread,the primary effect of thyroid hormone is to increase metabolic rate by increasing oxygen consumption(cellular respiration). Thyroid hormone helps regulate lipid metabolism and increases the rate of carbohydrateabsorption from the GI tract; it is also necessary for normal growth and neurological development in children.

206. Difficulty = 35 (difficult); Topic = Nervous and Endocrine Systems;Choice C is correct. According to the passage, osteoporosis is more significant in postmenopausal

women than in similarly-aged males because of the estrogen deficiency that accompanies menopause.Unfortunately, this is all the information about the link between estrogen deficiency and osteoporosis thatyou’re given, and it doesn’t help much in terms of answering this question. The key to answering this questionis deducing that the low doses of estrogen used to treat an estrogen-responsive adrenal tumor would not havean impact on the concentration of estrogen circulating in the male patient.

Treating an estrogen-responsive adrenal tumor with low doses of an estrogen analogue will notappreciably increase the amount of estrogen available for binding to osteoblasts (and stimulating them to buildbone) because almost all, if not all, of the exogenously-supplied estrogen will bind directly to the tumor cells.In other words, there won’t be any extra estrogen available for warding off osteoporosis. Therefore, a mantreated with low doses of an estrogen analogue will have approximately the same chances of developingosteoporosis as an age-matched control (a man not being treated with low doses of estrogen). Thus, choice Cis correct, and choices A, B, and D are wrong.

For the Curious Only: A crash course on the differences in estrogen secretion in males and females, and howit relates to osteoporosis:

While the primary source of testosterone in males is the testis and the primary source of estrogen infemales is the ovary, the testis also produces estrogen and the ovary also produces testosterone. In addition,the adrenal glands produce both testosterone and estrogen, though in much smaller quantities. Of the smallquantity of estrogen produced per day on average in normal young men, 15% comes from direct secretion bythe testes, while 85% comes from the peripheral aromatization of secreted androgens (androstenedione andtestosterone). In other words, testosterone is converted to estrogen. And rather than decreasing with age, theamount of circulating estrogen in males actually increases slightly with advancing age.

In women, the primary source of estrogen is the ovary, with the adrenals supplying the rest. The onsetof menopause and the decline in ovarian function results in a significant loss of estrogen. Without anexogenous supply of estrogen, the only source of estrogen in postmenopausal women are the adrenals. In otherwords, a 55-year-old man has more circulating estrogen (and a greater bone mass) than a 55-year-old,postmenopausal woman. Because osteoblasts (the cells that build bone) have been found to have estrogenreceptors, current belief holds that estrogen deficiency results in a decrease in osteoblast activity, causingosteoporosis. The fact that men experience a slight increase in estrogen production with advancing age, intandem with their higher bone density, most likely accounts for why women are more susceptible toosteoporosis than are men.

207. Difficulty = 85 (easy); Topic = Muscle and Skeletal Systems;Choice D is correct. The key to dealing with questions like this (where you’re given a lot of info in

the question stem), is to avoid getting tripped up by the similarity of answer choices. If you look at thesechoices too quickly, they all seem to say the same thing. Know what you’re looking for in an answer beforeyou even look at the choices the test-writer has supplied you with. That way, you can avoid sloppy test-takingand the loss of easy points.

According to the question stem, continuous bone remodeling is a function of the activity ofosteoblasts, which form (build) bone, and osteoclasts, which resorb (degrade) bone. You probably alreadyknew this, since it was covered in Biology Lesson 1. Osteoporosis is defined as a pathological decrease in aperson’s bone mass. Therefore, a decrease in bone mass would most likely be characterized by a decrease inbone building and an increase in bone degradation. In other words, osteoporosis would most likely becharacterized by a decrease in osteoblast activity and an increase in osteoclast activity. Therefore, choice Dis correct and choices A, B, and C are wrong.

Page 63: Explanations to AAMC Practice Test IIIdocshare02.docshare.tips/files/14524/145247660.pdfAAMC MCAT PRACTICE TEST III VERBAL REASONING EXPLANATIONS Passage I (Questions 1-6) This passage

Kaplan explanations to AAMC Practice Test III Biological Sciences

Copyright © 2001, Kaplan, Inc. 61None of this material may be reproduced by any means or incorporated into any information retrieval system, without written permission.

Passage XI (Questions 208–213)

Passage XI gives a brief description of an experiment carried out by a chemist, as well as the resultsof the experiment. The compound obtained is shown as Product A. This compound contains a hydroxyl groupand a carbonyl group, so it is classed as a β–hydroxy ketone, or an aldol. You are told that acetone undergoesa self-condensation reaction to form Product A. (You should now start to think about the mechanism of thisreaction since you may need to apply this mechanism to a compound other than acetone.) You are thenpresented with five different observations that tell you about the nature of Product A. Finally, you arepresented with another reaction in which Product A is dehydrated in the presence of hot sulfuric acid to formProduct B. Product B is not identified in the passage, so you will probably have to work out the structure ofthis molecule.

208. Difficulty = Moderate; Topic = Oxygen-Containing Compounds;The correct answer is choice B. There are two ways to answer this question. If you did not remember

the mechanism of the aldol condensation, you could have taken the easy route by noticing that the molecularweight of acetone is 58 g/mole, and the molecular weight of the product it forms, Product A, is 116 g/mole.Therefore, in the self-condensation reaction, the molecular weight of the product is double that of the reactant.The question states that the product has a molecular weight of 144 g/mol, so the molecular weight of thestarting material must be half this value—72 g/mol; choice B is the correct response. A more difficultapproach to the correct answer is to draw out the mechanism of the condensation reaction. The first step in thereaction is the abstraction of an alpha proton from acetone by OH– . This alpha carbon is now nucleophilic, andcan attack the carbonyl carbon of another acetone molecule; at the same time, pi electrons in the carbonylbond of the second acetone molecule are donated to the oxygen. The aldol molecule is now formed, exceptthat the oxygen of the hydroxyl group is not protonated. By picking up a proton from water (which forms whenOH– abstracts an alpha proton), oxygen becomes protonated, the base is regenerated, and aldol formation iscomplete. Although acetone initially loses an alpha proton, the second acetone molecule retrieves that protonwhen the aldol is formed. Hence, no species are lost in the reaction—the molecular weight simply doubles.Again, choice B is the correct response.

209. Difficulty = Moderate; Topic = Separations and Purifications;The correct answer is choice C. This question has a slight general chemistry bias; it tests your

understanding of Le Chatelier's principle. Le Chatelier's principle states that an equilibrium subjected to stressmoves to alleviate that stress. Therefore, in the equilibrium between acetone (the reactant) and Product A(the product), removing Product A as it is formed will place a stress upon the system since the product isdepleted. To alleviate the depletion of product, the equilibrium will shift to the right. Therefore, if Product Awas desired over acetone, a viable experimental procedure would be to remove Product A as it was formed—choice C.

Catalysts are species that speed up the rate of a reaction without being changed themselves. In anequilibrium reaction, catalysts affect the forward and reverse rates of reaction, i.e., the speed at which theequilibrium is reached; however, they do not affect the position of the equilibrium, which is what we are tryingto achieve in this question. Therefore, choices A and B are incorrect. In addition, choice B is incorrect sinceacetone does not act as a catalyst in the condensation reaction—it is a reactant. Increasing the reactiontemperature to the boiling point of acetone would result in its evaporation. As a result, it would be acetone,not Product A, that is removed from the equilibrium mixture. Applying Le Chatelier's principle, removing thereactant would shift the equilibrium to the left, not the right, so choice D is incorrect. Again, the correctanswer is choice C.

210. Difficulty = Moderate; Topic = Hydrocarbons;The correct answer is choice D. Observation 1 states that Product 1 consists of carbon, hydrogen, and

oxygen only. Observation 2 states that Product A has a molecular weight of 116 g/mole. Based onObservations 1 and 2 only, choice D would be a possible product since this molecule only contains carbon,oxygen, and hydrogen, and it does have a molecular weight of 116 g/mole.

Choice A can be eliminated since this molecule is an alkane, and therefore, does not contain oxygen.Although choices B and C contain oxygen, hydrogen, and carbon, they do not possess molecular weights of116 g/mole. The molecular weight of choice B is 130 g/mole, and the molecular weight of choice C is 100g/mole, so choices B and C can be eliminated. Again, the correct answer is choice D.

211. Difficulty = Moderate; Topic = Hydrocarbons;The correct answer is choice B. Acetone, in the presence of base, self-condenses to from an aldol,

which is shown as Product A. Aldols undergo further reaction when heated in the presence of acid (or base) toform an α -β unsaturated aldehyde or ketone. In the presence of the acid, the –OH group of Product A isprotonated, which makes it a very good leaving group. In addition, the protons alpha to the carbonyl group areespecially acidic (due to the ability of the molecule to undergo keto-enol tautomerism). The combination of a

Page 64: Explanations to AAMC Practice Test IIIdocshare02.docshare.tips/files/14524/145247660.pdfAAMC MCAT PRACTICE TEST III VERBAL REASONING EXPLANATIONS Passage I (Questions 1-6) This passage

Kaplan explanations to AAMC Practice Test III Biological Sciences

62 Copyright © 2001, Kaplan, Inc.None of this material may be reproduced by any means or incorporated into any information retrieval system, without written permission.

good leaving group and acidic alpha protons makes the dehydration reaction favorable, and an α − βunsaturated product forms:

+ H3O+

Product A

CH3 C

O

CH2 C

OH

CH3

CH3

H3O+

CH3 C

O

CH C

OH2

CH3

H

CH3

+

Enol

CH3 C

O

CH C CH3

CH3

α-β unsaturated ketone) Product B

(

As stated in Observation 5, a test for carbon-carbon double bonds is the addition of a solution ofbromine in carbon tetrachloride; Br2 adds to carbon-carbon double bonds and the Br2 /CCl4 solution turns fromred to colorless. Product A has no carbon-carbon double bonds, accounting for Observation 5; however,Product B does have a carbon-carbon double bond. Therefore, when a drop of Br2 in CCl4 is added to ProductB, the red color will disappear—choice B is correct.

Choices A and C are incorrect because Product B does contain a double bond. In addition, choice Ais contradictory to the Br2 test. If a compound did not contain a carbon-carbon double bond, the Br2 solutionwould remain red. Finally, choice D is incorrect since this statement is also contradictory to the Br2 test. If acompound contains a carbon-carbon double bond, the solution will turn colorless. Again, the correct answer ischoice B.

212. Difficulty = Moderate; Topic = Spectroscopy;The correct response is choice A. The 1 H NMR spectrum of Product A would consist of 4 signals or

peaks; a singlet corresponding to the –CH3 protons next to the C=O group, a singlet corresponding to the –CH2protons, a singlet corresponding to one –OH proton, and a singlet corresponding to six –CH3 protons next to thehydroxyl group. Acetone would give only one NMR signal —a singlet corresponding to the six equivalent–CH3 protons. A singlet is observed since the methyl protons in acetone are not next to any other protons, sothey cannot undergo spin-spin coupling. The acetone signal would be observed close to, but not identical with,the signal generated by the methyl protons adjacent to the carbonyl group in Product A. Therefore, if ProductA were contaminated with acetone, one extra signal would be observed and, again, choice A is the correctresponse.

213. Difficulty = Difficult; Topic = Oxygen-Containing Compounds;The correct answer is choice D. The iodoform test is used to identify carbonyl groups attached to at

least one methyl group. A positive test is signified by the formation of a bright yellow precipitate,corresponding to iodoform—CHI3. Product A, Product B, and acetone all contain –COCH3 functionalities, sothey all give positive iodoform tests; choice D is the correct response.

Discretes

214. Difficulty = 70 (moderate); Topic = Microbiology;Choice D is correct. Bacteria do not have any membrane-bound organelles, therefore they cannot

carry out oxidative phosphorylation in an organelle like the mitochondria. Instead, the enzymes of oxidativephosphorylation are located in the plasma membrane of the bacterium. (Ring a bell? I bet you noticed thatthis question could be answered from the information found in Passage V. Don’t count on this happening ontest day — this is extremely atypical.)

Choice A is wrong because the cell wall functions to protect the bacterium and help maintain itsshape; it has no enzymatic properties. Choice B is wrong because while bacteria do have ribosomes, they areinvolved in protein synthesis, like in eukaryotes, and play no role in oxidative phosphorylation. Choice C iswrong because bacteria do not have nuclear membranes. In fact, the absence of a nuclear membrane is onedefining features of prokaryotes.

215. Difficulty = Moderate; Topic = Hydrocarbons;The correct answer is choice C. The preferred bond angle of an sp3 hybridized carbon is 109.5°.

However, in cyclopropane, the bond angle is forced to 60°; as a result, the molecule suffers a great deal ofangle strain and is highly unstable. Due to its instability, the cyclopropane ring would much rather open toform straight-chain propane. In the presence of hydrogen, the ring actually does open and propane is formed.Hence, the addition of hydrogen to cyclopropane is a direct consequence of angle strain, and choice C is thecorrect response.

Page 65: Explanations to AAMC Practice Test IIIdocshare02.docshare.tips/files/14524/145247660.pdfAAMC MCAT PRACTICE TEST III VERBAL REASONING EXPLANATIONS Passage I (Questions 1-6) This passage

Kaplan explanations to AAMC Practice Test III Biological Sciences

Copyright © 2001, Kaplan, Inc. 63None of this material may be reproduced by any means or incorporated into any information retrieval system, without written permission.

Choice A is incorrect since resonance effects only apply to multiple bonds, i.e., double and triplebonds. Since cyclopropane and cyclopentane are fully saturated, choice A can be eliminated. Althoughcyclopentane has a higher molecular weight than cyclopropane, the difference in molecular weight does noteffect the reactivity, and choice C is incorrect. Choice D is irrelevant since both molecules do not suffer fromsteric hindrance. Steric factors only apply when the cycloalkane ring is larger than 7 carbons, or when thereare multiple substituents on a ring of any size. Since cyclopropane and cyclopentane contain fewer than 7carbons, and they are both unsubstituted, choice D can be eliminated. Again, the correct answer is choice C.

216. Difficulty = 50 (moderate); Topic = Molecular Biology;Choice A is correct. As soon as you saw uracil, and UTP, you should have thought: RNA. So you

are essentially being asked where RNA is synthesized. DNA is transcribed into mRNA in the nucleus, and sochoice A is correct. None of the other choices is involved in nucleic acid synthesis, so choices B, C, and Dcan be eliminated. Ribosomes synthesize proteins, which are typically transported through the endoplasmicreticulum, and then packaged in the Golgi bodies for secretion.

217. Difficulty = 60 (moderate); Topic = Molecular Biology;Choice D is correct. You are told that pepsin functions best at a pH of 1.5. Almost all enzymes, in

fact, function best in specific pH ranges. To appreciate why this is so, you need to know a little bit about howenzymes work. Remember that the first step in enzyme catalysis is the binding of the substrate to theenzyme’s active site, and that a good fit between substrate and active site is very important for proper enzymefunctioning. Therefore, any factor that changes the three-dimensional shape, or conformation, of an enzyme,will affect the functionality of the enzyme. One such factor is pH. (Temperature is another.) Enzymes areproteins, and one important determinant of a protein’s three-dimensional structure is the hydrogen bondingbetween different amino acid side chains within the protein. Altering the pH can alter the hydrogen bondingoccurring between the side chains, and thus change the enzyme’s shape. Choice D is therefore correct.

Choices A and B are wrong because a change in pH affects pepsin at the structural level; it does notaffect the regulation of its synthesis. Choice C is wrong because it implies that pepsin would functionoptimally at a pH of 1.0. However, this contradicts what you were told in the question.

218. Difficulty = Moderate; Topic = Biological Molecules;The correct answer is choice C. The diagram in the question represents three different amino acids

joined by peptide bonds (–CONH– bonds). Peptide linkages form when the amino group of one amino acidcondenses with the carboxyl group of another amino acid. Since each amino acid contains an amino groupand a carboxyl group, and there are three amino acids in the tripeptide, the number of different tripeptides thatcan be produced is 3 × 2, or 6. Choice C is, therefore, the correct response.

219. Difficulty = 60 (moderate); Topic = Generalized Eukaryotic Cell;Choice B is correct. This question assesses your understanding of the function of spindle

microtubules. You should know that spindle microtubules play an important role in both mitosis and meiosis.They are responsible for pulling the chromosomes to opposite poles of the dividing cell during anaphase. Whatwould happen, then, if spindle microtubules were prevented from forming? The chromosomes would not bepulled to opposite poles, and would instead remain at the equatorial plate. It is likely that some cells wouldend up with more than the normal complement of chromosomes (this is called polyploidy), while other cellswould end up with no chromosomes (this is called aneuploidy). Thus, choice B is correct.

Choice A is wrong because no new genetic material is introduced into the resulting plants or plantcells as a result of tubulin inhibition. In other words, the daughter plants have the same genetic variability asthe parent plants. In addition, Choice C is incorrect for two reasons. 1) Because the question stem suggeststhat there would be an inhibition of tubulin function, not an excess of tubulin; and 2) because tubulin is not acomponent of the cell wall. Tubulin is, however, a component of the cytoskeleton, so if anything, disruptingtubulin function would compromise the cell’s ability to maintain its shape. Choice D is wrong for the samereason as choice C: The question suggests an inhibition of tubulin function, not an excess of tubulin.